Classified Criminal Appeals Bulletin 2000 Part 3 - Department of Justice [PDF]

Dec 8, 1999 - [English digest of MA 793/99 above]. Pang J. (15.10.99). *L Ho. #I/P. YU King-wai. Hospital order/Power of

4 downloads 5 Views 2MB Size

Recommend Stories


the Department of Justice Criminal Resource Manual
No amount of guilt can solve the past, and no amount of anxiety can change the future. Anonymous

texas department of criminal justice vehicle fleet
Ask yourself: Is romantic love important to me? Next

PDF The Criminal Justice System
Where there is ruin, there is hope for a treasure. Rumi

Criminal Justice
We must be willing to let go of the life we have planned, so as to have the life that is waiting for

Criminal Justice
When you talk, you are only repeating what you already know. But if you listen, you may learn something

PdF Essentials of Criminal Justice Full Book
Forget safety. Live where you fear to live. Destroy your reputation. Be notorious. Rumi

Research Bulletin 2014 Part 3
Don't fear change. The surprise is the only way to new discoveries. Be playful! Gordana Biernat

Testimony: Texas Department of Criminal Justice Legislative Appropriations Request
You often feel tired, not because you've done too much, but because you've done too little of what sparks

Department of Justice
If you feel beautiful, then you are. Even if you don't, you still are. Terri Guillemets

Untitled - Department of Justice
What we think, what we become. Buddha

Idea Transcript


CCAB 2000

Sentence (Principles)

Sentence (Principles) MA 1082/99

NGAI Yuk-ka

Gall J (8.12.99)

Training centre order/No power to impose consecutive orders of detention in training centre 教導所令 - 無權判處教導所羈留令分期執行 The Appellant pleaded guilty at North Kowloon Magistracy to a charge of theft. He was sentenced to detention in the training centre, and the magistrate stated that the sentence was to be consecutive to the sentence he was then serving in the training centre. On appeal

*Grace Chan #I/P

Held : The training centre order made by the magistrate was entirely appropriate. However, there was no provision in the Training Centres Ordinance which allowed a training centre order to be consecutive to an earlier order that a person be detained in a training centre. Result - Appeal allowed. Sentence of consecutive detention quashed and substituted with order for detention in a training centre. [see Sentencing in Hong Kong, 2nd ed, at 265: Ed]

FACC Nos 35 & 36/99 Li CJ Litton & Bokhary PJJ (31.1.2000) *IG Cross SC P Chapman & Wong Sze-lai #AC Macrae SC & P Duncan

YIP Kai-foon

Medical grounds seldom basis for reducing sentence for crimes of gravity/Serious injuries suffered in course of criminal activity can reduce sentence/All circumstances to be considered 就嚴重罪行而言,健康理由難以成為減刑的理據 - 在犯罪活動中嚴 重受傷可縮減刑期 - 需考慮所有情況 The Applicant was convicted in October 1985 of two counts of handling stolen jewellery and two counts of possession of firearms. He received a total of 18 years’ imprisonment on those counts, which was reduced to an overall sentence of 16 years on appeal. In 1989, he escaped from Queen Mary Hospital where he had been sent for medical attention while serving that sentence at Stanley Prison. When making his escape, he commandeered a van and kidnapped the van driver and his son. That gave rise to the counts in the indictment HCCC 271/96, being one of escaping from legal custody and two of kidnapping, contrary to common law. From August 1989 to May 1996 the Applicant was at large. In the early hours of 13 May 1996, the Applicant was among a group of men who were spotted by patrolling police officers. He exchanged shots with the police and all members of the group, except him, escaped. He suffered a gunshot wound which left him paralysed from the waist down. That incident gave rise to the counts in HCCC 270/96, which were possession of firearms, use of firearms with intent to resist arrest, possession of explosives with intent to endanger life or property and an alternative count of possession of explosives simpliciter. HCCC 271/96 The Applicant pleaded guilty to the three counts in this indictment and was sentenced as follows: (1)

Escape from lawful custody - 2 years (the statutory maximum was 2 years);

(2)

Kidnapping - 3 years; and

(3)

Kidnapping - 3 years. 300

CCAB 2000

Sentence (Principles)

The judge ordered that counts 2 and 3 be served concurrently but consecutive to count 1, making a total of 5 years. He also ordered that that sentence be consecutive to the overall sentence of 25 years, imposed on the counts in HCCC 270/96, and to the pre-existing sentence of 11 years and 3 months. That made a total of 41 years and 3 months. HCCC 270/96 The Applicant pleaded not guilty to the three counts but was convicted after trial and sentenced to 13 years (maximum 14 years) on the possession of firearms count, to 20 years (maximum life imprisonment) on the use of firearms count, and to 18 years (maximum 20 years) on the count of possession of explosives with intent to endanger life or property. Counts 1 and 3 were ordered to be concurrent but 5 years of those concurrent sentences were ordered to run consecutively to count 2, making a total of 25 years. In the Court of Appeal, the sentence of 41 years and 3 months was reduced to one of 36 years and 3 months on the basis that the totality was excessive. That reduction of sentence did not take into account the Applicant’s physical condition of being paralysed from the waist down with his life expectancy shortened. As to that, the Court of Appeal said: We are asked to bear in mind that the Applicant is a paraplegic who must endure grave hardship which is not the lot of an ordinary prisoner and that his life expectancy has been substantially reduced. We acknowledge that it is in our discretion to give weight to this ad miseracordiam plea, but are satisfied, as was the judge, that any such considerations should be left for the Executive. The sentencing judge had made no reference to the Applicant’s paraplegia, beyond saying to the Applicant: I am satisfied that you no longer pose any danger to the public …. I am persuaded that it would not be appropriate to pass a life sentence. The Applicant sought leave to appeal out of time against sentence to the Court of Final Appeal. The Court of Appeal earlier certified the involvement of a point of law of great and general importance, namely: Whether serious injuries received by an accused in the course of criminal acts can be a factor which lies for consideration by the sentencing court as to the length of the term of imprisonment to be imposed for those criminal acts. The Applicant contended (i) that the Court of Appeal erred in failing to take his paraplegia into account, and (ii) that such failure had resulted in substantial and grave injustice. In the course of argument, counsel for the Applicant amended the certified question by substituting the word ‘should’, for ‘can’. Held : (1)

In Re C W Reid [1994] 2 HKLR 14, 24, it was said: The responsibility for the proper punishment of offenders rests with the courts. This is done by the application of guidelines and principles to the facts of the particular case: guidelines and principles which the courts themselves have evolved over the years. … As a matter of principle … the practice of leaving it to the 301

CCAB 2000

Sentence (Principles)

executive to deal with the proper discount under Article XV of the Letters Patent can only be justified if there is no other course the courts can themselves properly take, since the remission of sentence under Article XV is done behind closed bureaucratic doors and is the exercise of a purely executive function. Article 48(12) of the Basic Law now provided that the Chief Executive had the power and function to commute the penalties of prisoners convicted of criminal offences; (2) Medical grounds would seldom, if ever, be a basis for reducing the sentence for crimes of gravity: HKSAR v Tong Fuk-sing [1999] 3 HKC 332, R v Ho Mei-lin [1996] 4 HKC 491. However, in those cases the defendant’s physical condition did not result from injuries received in the course of criminal acts; (3) Where injuries were received in the course of criminal acts, it was held in R v Chak Shui-chung [1989] 2 HKLR 81 that, although in most cases the injured person had brought the misfortune upon himself, and it was the criminal act which attracted the punishment, a punishment not lessened by virtue of damage to the perpetrator of that act while in the course of it, there were exceptions where the injuries were very serious. In other words, exceptionally, very serious injuries, even when suffered in the course of criminal activity, could be taken into account to reduce the sentence. There was no question of ‘must be’. In deciding whether the sentence should be reduced for that reason, all the circumstances of the case would have to be considered; (4) The guidelines and principles for sentencing evolved in these cases were plainly sound and should be applied by the courts. The cases, and in particular Chak Shui-chung, provided the clear answer to the certified question; (5) The Court of Appeal erred in not considering the question at all and in leaving it to the Executive. However, the Applicant had to establish an arguable case that the sentencing court considering the question would give weight to his condition and reduce his sentence for that reason, and, in the circumstances of this case, such a case had not been made out. Although the injuries suffered by the Applicant were very serious, the offences were very grave and he suffered his injuries in a gun battle between his gang and the police. Although no police officers or members of the public were injured they were put to terrible risk. The actions of the Applicant and his gang came very close, as the Court of Appeal rightly stated, to declaring war on society and ‘a court would be failing in its duty to the public if it did not impose heavy deterrent sentences in circumstances such as this’. Result - Application dismissed, by a majority.

302

CCAB 2000

CA 543/99 Stuart-Moore VP Leong & Wong JJA (13.1.2000) *Cheung Waisun #I/P

Sentence (Principles)

CHOY Po-lan

Hospital order without limit of time/Doctors unable to predict date of release/Review of position from time to time/Absence of insight into gravity of offence 沒有時限的入院令 - 醫生不能預計申請人的出院日期 - 不時檢討申 請人的狀況 - 申請人不知道罪行嚴重 The Applicant pleaded guilty in the High Court to a count of throwing corrosive fluid with intent, contrary to s 29(c) of the Offences Against the Person Ordinance, Cap 212. Following an adjournment for updated psychiatric reports, the judge made a hospital order under s 45 of the Mental Health Ordinance, Cap 136, committing the Applicant to Siu Lam Psychiatric Centre for an unspecified period.

The Applicant sought leave to appeal out of time against the order, stating in effect that because of her mental illness she was unable to appeal at the proper time. She believed she was now getting better and her application was to have the order limited to a specified period of time. The facts revealed that at the time of the offence the Applicant and her daughter were psychiatric out-patients at different hospitals. The Applicant was dissatisfied with her daughter’s treatment and her own treatment which was being given at their respective hospitals. On 23 October 1996, the Applicant went to the clinic where her daughter received medication. She went up to the desk in the clinic to make a request to the member of staff who happened to be there that her daughter should no longer be ‘forced’, as she put it, to take medication. Just before going to the hospital, the Applicant had been into a shop where she bought a cleaning liquid which contained concentrated sulphuric acid. In due course, the Applicant became angry with the staff in the hospital. She went to a plastic bag which she had left under a bench. She then opened the bottle of acid which was inside the bag and threw it over the unfortunate victim, who was the male nurse on duty. He was seriously injured with 22% burns to his body. In hospital later, he had to undergo two skin transplant operations. The judge, in passing sentence, considered psychiatric reports. The latest of those was prepared by Dr C H Yuen, the visiting consultant psychiatrist at Siu Lam Psychiatric Centre, and dated 8 July 1997. He described the condition of the Applicant thus : I noticed that although Madam Choy has been given drug and psychological treatment, she continues to have no insight. She invariably would become angry whenever the present offence is mentioned. She always says it is extremely unfair for her for having to stay in Siu Lam whereas the one who had been persecuting her daughter for more than 4 years could resume having his job though she had poured corrosive onto him. She knows it to be wrong to throw corrosive onto someone but she has little remorse for having done so. She does not like Siu Lam. She hates taking drugs. She also does not like the doctor who is treating her. She even has the idea that the doctor is persecuting her. When I examined her on 7 July 1997, she was quite calm initially but immediately became agitated when she talked about the alleged persecution by the male nurse whom she had wounded. She absolutely denied that her daughter has any mental illness. I told her it was the doctors who diagnosed her daughter having mental illness and not the nurse. She said those people were not real doctors. They were at most interns. She said after discharge she would try all the means she could find to stop her daughter from receiving psychiatric treatment. She did not 303

CCAB 2000

Sentence (Principles)

elaborate further as to what she would do. On herself, she denied being mentally ill and stated clearly that she would not attend follow-up after release. In my opinion, this patient is a dangerous schizophrenic patient. She has not made any improvement despite treatment in the past few months. The fixed delusion that she has is a real danger to anyone whom she considered to be related to her daughter’s (and possibly she herself as well) psychiatric treatment. The risk that she would act dangerously again is high, even in restricted environment. She needs to be placed in a secure setting and Siu Lam Psychiatric Centre which is the only suitable place for her. As to the duration of in-patient treatment required, in view of the uncertain prognosis, I would recommend to court to give her an unspecified period Hospital Order sentence and then it would be for the Mental Health Review Tribunal to decide in the future when and under what conditions could she be discharged. Held : (1) The order made by the judge was the only order which it was appropriate to make. As the judge said to the Applicant at the time of sentencing, the period of time she spent in hospital would be open to review from time to time, following an assessment of her response to the treatment she was being given. The doctors were not able to be specific about when it would be possible to release the Applicant back into the community, and it was because of this that the judge had no realistic option to the order he made; (2) If it was considered by the doctors that she no longer posed a danger to the community, no doubt she would have been released; (3) This was a sad case, and it appeared the Applicant still had no insight into the gravity of her offence, saying only that what she did was not seriously wrong. The order made would stand. Result - Application dismissed.

304

CCAB 2000

CA 520/99 Stuart-Moore VP Mayo JA & Woo J (18.1.2000)

Sentence (Principles)

CHU Shi-kung

Discount of one-third usual for timely plea of guilt/Defendant caught redhanded/Comments on departure from customary discount 及時認罪通常可獲減免三分一刑期 - 被告人當場被捕 - 就偏離慣常 的減刑提出意見 The Applicant pleaded guilty to a domestic burglary. He was sentenced to 2½ years’ imprisonment for that offence, and to a consecutive term of 12 months for a second offence, to which he also pleaded guilty, of remaining unlawfully in Hong Kong.

*J To #J Marray

The Applicant sought leave to appeal against the sentence for the burglary offence, the facts of which revealed that he was caught red-handed inside the flat he had burgled at night by the occupants who were disturbed by him. The police were called and the Applicant was found in possession of money he had stolen in the flat and the tools which he had used to enter it. He admitted having sneaked into Hong Kong from China, and said he had broken into the flat in order to find something to eat. Although the judge appeared to have adopted 3 years as the proper starting point for a domestic burglary, he then said : You pleaded guilty just now, but it carries no further weight than just a guilty plea because the evidence against you is overwhelming. You are one of those classic cases and I do not find any mitigation factors in your favour. On compassionate grounds I give you some allowance on your young age which will be reflected in the total sentence I now pass on you. On charge 1 of burglary I sentence you to 2½ years. Held : (1) Quite what the judge meant by ‘it carries no further weight than just a guilty plea because the evidence against you is overwhelming’, was something that was difficult positively to discern. It seemed, however, that the judge was intending to convey to the Applicant that because he had been caught in the act, he would not receive the full benefit usually accorded to those who pleaded guilty;

(2) This was an example of an accused being deprived of the full discount of one-third for his plea of guilty where no valid reason, having regard to current sentencing practice, had been provided for the course taken. The court had said repeatedly that when a person was caught red-handed this did not provide a good reason for a failure to give the normal discount; (3) There were sometimes reasons for not giving a full discount on a plea of guilty, such as a failure to enter a timely plea where, for example, the accused had unsuccessfully contested a voire dire and had only then pleaded guilty, or in some cases where he had absconded on bail. And that was not an exhaustive list of situations where, following a plea of guilty, a failure to accord the full discount or, in extremely rare cases, any discount at all, would be applicable; (4) Being caught ‘red-handed’ had not for a considerable time been one of the criteria for a failure to give the normal discount for a timely plea. Counsel must know how to advise their clients when they were minded to plead guilty and, if a wide discretion was given to judges in deciding whether or not to give a full discount for plea, counsel would understandably be unable to give sensible advice and defendants would be left with a justifiable grievance if the discount they expected to receive was not in the event given to them.

305

CCAB 2000

Sentence (Principles)

Result -

Appeal allowed. Sentence on Charge 1 reduced from 2½ years to 2 years. Overall sentence reduced by 6 months from 3½ years to 3 years.

香港特別行政區訴余勁慧 HKSAR v YU King-wai 高等法院原訟法庭 – 裁判法院上訴1 9 9 9 年第7 9 3 號 *何眉語 L Ho #上訴人自辯 I/P

高等法院原訟法庭法官彭鍵基 聆訊日期:一九九九年十月十五日 宣判日期:一九九九年十月十五日 COURT OF FIRST INSTANCE OF THE HIGH COURT MAGISTRACY APPEAL NO. 793 OF 1999 PANG J Date of Hearing : 15 October 1999 Date of Judgment : 15 October 1999 入院令 - 裁判官頒布不符建議的住院期的權力 上 訴 人 被 控 兩 項 傷 人 罪 名 。 根 據 當 時 情 況 顯 示 , 上 訴人患有 精神分裂病,常有幻覺。由於她是精神病人,原審裁判官根據香港 法例第136 章《精神健康條例》要求兩位精神科醫生提供報告。兩 位醫生在報告中都建議上訴人應該接受為期四至六個月的住院治 療。 裁 判 官 考 慮 兩 位 精 神 科 醫 生 報 告 的 時 候 , 基 於 上 訴 人情況相 當嚴重,認為四至六個月治療期並不適合,因此頒令了一個無限期 的入院治療令。上訴人為此提出上訴。 裁決: (1) 裁 判 官 有 權 拒 絕 接 納 或 不 受 醫 生 意 見 約 束,但本案中,除了醫 生的建議之外,根本沒有其他理由足以支持裁判官頒發的無限期住 院令; (2) 再 者 , 如 六 個 月 後 上 訴 人 仍 未 完 全 康 復 的話,院方有權根據第 136 章第36 條,由醫生簽署病人需要繼續留院的證明,因此病人的 健康情況仍有充分保障,並應該由院方醫生經檢驗才作決定。 改頒一個為期六個月的住院令。

306

CCAB 2000

[English digest of MA 793/99 above] Pang J (15.10.99) *L Ho #I/P

Sentence (Principles)

YU King-wai

Hospital order/Power of magistrate to impose a period contrary to that recommended The Appellant was charged with two charges of wounding. The circumstances of the case showed that she was suffering from schizophrenia and often had hallucinations. As she was a mental patient, the magistrate called for two psychiatric reports, in accordance with the Mental Health Ordinance, Cap 136. Both doctors recommended that the Appellant should receive medical treatment in hospital for a period of four to six months. The magistrate, upon considering the psychiatric reports, believed that four to six months treatment would not be appropriate in view of the Appellant’s rather serious condition. She therefore made a hospital order for an unspecified period. On appeal Held : (1) A magistrate did have the power to reject or to refuse to be bound by the medical opinions. However, in this case, apart from the doctors’ recommendations, there was virtually no other ground to justify the magistrate in making a hospital order for an unspecified period; (2) In addition, if the Appellant still had not fully recovered after six months of treatment, the hospital could invoke section 36 of Cap 136, under which a patient could be detained for further treatment upon medical practitioners’ certifications. Hence, the patient’s health could be adequately safeguarded. Nevertheless, the decision relating to such an issue should only be made after the medical practitioners of the hospital had examined the patient. Result - Hospital order for a period of six months substituted.

307

CCAB 2000

CA 66/2000 Mayo VP & Wong JA (18.4.2000)

Sentence (Principles)

LEE Kam-kei

Guilty plea/Basis for denial of customary one-third discount 認罪 - 不給予慣常三分一刑期減免的理據 The Applicant pleaded guilty in the District Court to two charges of robbery. On the first charge, he was sentenced to 1½ years’ imprisonment, and on the second charge the judge imposed a sentence of 3 years’ imprisonment to be served consecutively making a total sentence of 4½ years’ imprisonment.

*Edmond Lee #I/P

The facts showed that on 8 September 1996, the Applicant snatched a gold necklace and two gold bracelets from a woman inside a lift. On 24 October 1996, he again snatched from another woman, also inside a lift, a gold bracelet, a gold ring and $600 in cash. The woman was carrying a baby and on this occasion a knife was displayed. In passing sentence, the judge said: The defendant is 42 years old and has 10 previous court appearances, seven of which resulted in convictions for robbery and one for possession of an offensive weapon. In mitigation it is urged that he has a wife and two children and that his brother died prematurely recently. These are separate offences committed more than a month apart and I am of the view that the sentences must be consecutive. I will take into account the totality principle in assessing the sentences and I will give the defendant a discount for his pleas of guilty. I select starting points of 2 years on charge 1 and 4 years on charge 2, respectively. I discount each of these figures on account of the pleas of guilty. In the result, the defendant is sentenced on charge 1 to 1½ years’ imprisonment; on charge 2 to 3 years’ imprisonment consecutive to that, making in effect a total of 4½ years’ imprisonment. On appeal, it was submitted that the Applicant had not been given a full discount of one-third of the sentence for his guilty plea and as a result he received 6 months more than he should otherwise have. Held : (1) Mathematically, the Applicant was quite correct. It was also correct that a discount of one-third would normally be given for early pleas of guilty. There were, however, circumstances that might warrant a discount of less than onethird, such as a late plea or that the accused had absconded from bail during trial. But this case was not one of them. The judge had not given any reason as to why he did this; (2) The Applicant pleaded guilty at the outset of the trial and there were no circumstances to justify denial of a full discount; (3) The sentence of 18 months on the first charge would be reduced by 2 months to 16 months and the sentence of 3 years on the second charge would be reduced from 4 years to 2 years and 8 months. The sentences would be served consecutively making a total sentence of 4 years. Result - Appeal allowed.

308

CCAB 2000

MA 966/99

Sentence (Principles)

MAN Man-tai

Assistance to authorities/Extent of discount/Whether accused a ‘supergrass’ 協助當局 - 減刑幅度 - 被告是否‘告密人’

Nguyen J (16.12.1999) *David Chan #S M Chan

The Appellant pleaded guilty to a charge of trafficking in 34 packets of heroin and was sentenced to 12 months’ imprisonment. He pleaded guilty on the basis that two of the 34 packets of heroin, which were the subject matter of the charge, were for onward distribution to other drug users, and the rest were for his own personal consumption. He was sentenced by the magistrate on that basis. The Appellant gave evidence against another person on the charge of trafficking in dangerous drugs. That person had been arrested by the ICAC for cigarette smuggling offences. The ICAC had intelligence that another person had also been involved in drug trafficking offences, but they had no admissible evidence to substantiate that intelligence. After that other person was arrested by the ICAC, the Appellant gave information to the ICAC which resulted in that person being charged in addition to the cigarette offences, and with drug trafficking offences. For the cigarette offences, that person pleaded guilty and was sentenced to 18 months’ imprisonment. For the drug trafficking offences, the Appellant gave evidence at that person’s trial, his evidence was accepted by the trial judge, and the other person was convicted and sentenced to 5 years’ imprisonment, consecutive to the 18 months for the cigarette offences. The magistrate set two years as the starting point and reduced that by eight months to recognise the guilty plea. He also further reduced the sentence by four months to reflect the Appellant’s assistance to the prosecution, and ultimately sentenced the Appellant to 12 months’ imprisonment which amounted, in effect, to a 50% discount of the magistrate’s starting point of two years. The Appellant submitted that as a supergrass a discount of two-thirds was proper. It was said that, on the authorities, a discount of 50% was usually only for information supplied to the authorities, without the information being coupled with the Appellant appearing in court as a prosecution witness. Reliance was placed upon R v Siu Chun-chung & Another Cr App 741/95, where the appellant received a discount of 60% and the other one of 55%. Held : (1) The Appellant would not fall into the category of a supergrass only because he assisted the prosecution in only one case as opposed to a normal supergrass who would assist the prosecution in a number of cases; (2) The Appellant should receive a discount of 60% for his assistance. If the same starting point as that used by the magistrate was adopted, then a discount of 60% would produce a sentence of 9 months and 18 days. But following that done in Siu Chun-chung, the sentence would be rounded down to one of 9 months’ imprisonment. Result -

Appeal allowed. [For further consideration of the approach to the sentencing of a ‘supergrass’, see Sentencing in Hong Kong, 3rd ed, at pp 39-41: Ed.]

309

CCAB 2000

CA 606/99 Mayo VP Leong & Keith JA (19.5.2000) *Ian McWalters #D MackenzieRoss

Sentence (Principles)

LEE Sing-ho

Recidivist/Effect of criminal record upon sentence/Persistent criminality an aggravating factor/Protection of public 積犯 - 犯罪記錄對判刑的影響 - 屢次犯罪是加重刑罰因素 - 保護市 民 The Applicant pleaded guilty in the District Court to one charge of theft, two charges of common assault, one charge of blackmail and one charge of claiming to be a member of a triad society. The judge, having studied DATC and probation reports, and having given a discount of one-third for the pleas of guilty, sentenced the Applicant to two months’ imprisonment for the theft; on one of the two charges of common assault, to two months’ imprisonment; to 16 months’ imprisonment on the blackmail charge, all consecutive, making a total of 20 months’ imprisonment. On the remaining charge of common assault, the Applicant was sentenced to 4 months’ imprisonment, and on the charge of claiming to be a member of a triad society, to 16 months’ imprisonment, concurrent with the sentence of 4 months but consecutive to the sentence of 20 months for the theft, assault and blackmail charges. The total was thus 36 months’ imprisonment. On appeal, it was submitted, inter alia, that the judge attached undue weight to the Applicant’s past record. When considering sentence, the judge described the Applicant’s criminal record as ‘startling’. At the time of sentence, the Applicant was 23 years old. He had a number of previous convictions on charges of blackmail, assaults occasioning actual bodily harm, common assault and offences relating to triad membership and theft. His latest convictions were in June 1997 and these were the same as those in the present case for which he was sentenced to a total of 24 months’ imprisonment. In respect of the sentence of 24 months, the Applicant was released in May 1999. This meant that the Applicant repeated his offences just five months after he was released from prison.

Held : (1) The judge was conscious of the principle that the Applicant should only be sentenced for the offences he had been convicted of and not for his previous records. He reminded himself of that said in HKSAR v Chan Pui-chi [1999] 2 HKLRD 830, in relation to the sentencing of a repeat drug trafficker: ‘….. the constant repetition of this Applicant’s trafficking in dangerous drugs had itself increased the gravity of these offences, at least so far as sentence is concerned. The sentences he had received in the past had proved to be no deterrent at all, and prevention of further repetition was demonstrably in the public interest.’ The judge considered that this principle was equally applicable in sentencing the Applicant. He adopted as starting points 3 months and 6 months for the two assault charges, 2 years for the blackmail and for the claiming to be membership of a triad society offence. Bearing in mind the principle of totality and giving credit for the plea, the judge arrived at the sentences he imposed; (2) In Chan Pui-chi, the Court of Appeal also said ‘the starting point which would have been adopted for a man of good character will sometimes need to be increased to take into account the aggravating features of persistence and the failure of previous sentences to deter on the one hand, and the court’s duty to protect the public on the other…..’ Result - Application dismissed.

310

CCAB 2000

MA 209/2000 Gall J

Sentence (Principles)

TSUI Ching-wai

Possession for purpose of trade goods to which forged trade mark applied/Offender to be sentenced according to current guideline 管有應用偽造商標的貨品作商業用途 - 應按照犯案時適用的指引把 違犯者判刑

(10.5.2000) *K P Zervos #F C Whitehouse

The Appellant was convicted in February 2000 of one charge of possession for the purposes of trade of goods to which a forged trade mark was applied and, secondly, of a charge of possession for sale of goods to which a false trade description was applied. She was sentenced to concurrent sentences of imprisonment of 3 months on each charge. The facts showed that in July 1999, customs officers went to a store room of a flat in Bonham Strand West, which was operated as the ‘Ka Fung Ginseng Hong’, a company jointly owned by the Appellant and her husband. The Appellant was found in possession of, and packaging ginseng. Also in the premises were found various canned ginseng which were the subject of each of the two charges. Some of the ginseng had a forged trade mark, and the rest a false trade description. The Appellant admitted that she possessed the ginseng which came from China, and that it would be passed on to a processor who would put it into tin cans. The cans would then be returned to her and her husband for sale. Her husband had purchased the ginseng from the Mainland and had it shipped to the premises. She was responsible for packaging, arranging for it to be canned, and for sale. She knew the counterfeit nature of the goods and made full admissions. The magistrate, in sentencing, said: The Court of Appeal in Secretary for Justice v Lam Chi-wah [1999] 4 HKC 343, stated that hawkers and small traders in counterfeit goods should face heavy fines whereas wholesalers should face imprisonment unless there were special circumstances. It added that financial difficulties and a clear record would be unlikely to be regarded as special circumstances. Taking into account all the circumstances of this case, I considered a sentence of 6 months to be appropriate and I reduced that to 3 months in view of the guilty plea and the cooperation with the prosecution. On appeal Held : (1) As the offence occurred in July 1999, and the judgment in Lam Chi-wah was handed down in October 1999, the magistrate had made use of a judgment not in existence at the time of offence to determine sentence. In Sentencing in Hong Kong, 2nd Ed, at 130, it was stated: The sentence for an offence should be within the bracket prevailing at the time when the offence was committed. If there has been a change in the guidelines between the dates of offence and sentence, and this invariably involves an upward revision, then this should not operate to the detriment of the accused: R v Chu Chi-yat and Others [1993] 2 HKCLR 1, 4. Equally, if the Court of Appeal , in disposing of a particular appeal, takes the opportunity to issue new guidelines, this should not be to the disadvantage of the particular accused who happens to be before the court: R v Cheng Yeung [1989] 2 HKLR 258, 265. This approach achieves statutory recognition in section 8, Article 12 of the Bill of Rights Ordinance. 311

CCAB 2000

Sentence (Principles)

The appeal would be allowed on the basis that the magistrate followed an authority which was not in existence at the time when the offence took place; (2) The leading authority prior to Lam Chi-wah was that of Secretary for Justice v Yip Chi-tung [1998] 3 HKC 214. Had the magistrate considered that authority, the sentence he would in all likelihood have passed would have been a fine rather than imprisonment. The level of fine at that time was 30% of the value of the goods. Result - Appeal allowed. Custodial sentence set aside. In respect of charge 1, a fine of HK$4,000 would be substituted, and, in respect of charge 2, a fine of HK$35,000.

CA 20/2000 Stuart-Moore VP Leong & Wong JJA (27.6.2000) *P S Chapman #Paul Wu

YEUNG Kin-man

Discount for guilty plea/One-third discount appropriate in absence of good reason/Lesser discount not warranted where prosecution case overwhelming 承認控罪獲減刑 - 若無更充分的理由,減刑三分一是恰當的 - 法官 不應因控方證據確鑿而縮短可減免的刑期 The Applicant pleaded guilty to two counts of trafficking in a dangerous drug. He was sentenced to 20 years’ imprisonment. The judge took a global approach to the total quantity of narcotics involved and adopted a starting point of 27 years’ imprisonment. He considered that substantial credit was due to the Applicant for his guilty plea and full cooperation but that, as the evidence was ‘overwhelming’, he would give a discount of 7 years only. He saw no reason for further discount and imposed the sentence of 20 years on each count. On appeal, it was submitted that the judge erred in refusing to give the Applicant the usual one-third discount for his plea of guilty because the evidence against him was ‘overwhelming’. It was said that there were two lines of decision in respect of discount for a plea of guilty. One line (see, eg. AG v Wong Kwok-wai [1991] 2 HKLR 384), indicated that if a defendant was caught ‘red-handed’ or faced overwhelming evidence, the plea was little more than a recognition of the inevitable and he ought not to be given the usual one-third discount despite having pleaded guilty at an early stage. The other line of authority indicated that even in those circumstances a full one-third discount should still be given (see, eg, HKSAR v Wong Ka-kuen & Another Cr Ap 35/98). The Applicant contended that the present position was represented by the latter line of authority, and that a plea of guilty showed remorse, saved court time and prosecution resources. It was also said that if the question of how much discount would be given for a plea was clear, defence counsel would be in a better position to advise a defendant on plea. (Counsel for the prosecution did not dispute this analysis.) Held : The settled practice was reflected in Wong Ka-kuen and Another, (above), where Stuart-Moore JA said: This court has stressed on frequent occasions that, in the absence of good reason, where a timely plea has been entered, a defendant is entitled to his full one-third discount. Being caught ‘redhanded’ is not, in the view of this court, a sufficient reason to disallow the full discount …. The judge was wrong to have given a lesser discount solely by reason of the overwhelming evidence against the Applicant.

312

CCAB 2000

Sentence (Principles)

Result - Appeal allowed. Sentence of 18 years’ imprisonment on each count concurrent substituted. [For further consideration of the discount attaching to guilty pleas, see Sentencing in Hong Kong, 3rd ed at pp 247-256:Ed.]

CA 604/99

DANIEL John Peter

Stuart-Moore VP Leong & Woo JJA

First offender/Clear record and age not automatic basis for discount/Unlikelihood of re-offending only affected issue of deterrent sentence 初犯者 - 無犯罪紀錄和年齡不一定是獲給予減刑的理據 - 不大可能 再犯案這一點,只會在考慮應否判處阻嚇性刑罰時才起作用 The Applicant was convicted after trial of one charge of attempting to obtain property by deception. He was sentenced to 2½ years’ imprisonment.

(21.6.2000) *Eddie Sean

On appeal, it was submitted, inter alia, that the judge refused to give the Applicant any discount despite his age (63 years) and his clear record, and despite the fact that the offence was out of character and the chance of reoffending was remote.

#E L McGuinniety

Held : The clear record and age did not automatically entitle the Applicant to a discount, and the unlikelihood of re-offending only meant that it would not be necessary to consider a deterrent sentence. The ultimate question was whether the sentence imposed was manifestly excessive having regard to all the circumstances of the case. If not, the sentence should not be disturbed. Result - Application dismissed. Two months loss of time ordered.

CA 29/2000 Stuart-Moore VP Leong & Wong JJA (14.6.2000) *Albert Wong #Paul Fok

CHAN Ka-po, Joseph

Handling stolen motor vehicles/Co-accused sentenced differently by another judge/Whether legitimate grievance based on disparate sentencing 處理被盜取的汽車 - 同案被控人給另一名法官判處不同的刑期 - 基 於判刑有別而感到受屈是否合理 The Applicant pleaded guilty in the District Court to a charge of handling two high value motor cars, which were respectively a Mercedes Benz and a BMW. He was sentenced to 4 years’ imprisonment. On appeal, it was submitted, inter alia, that the Applicant had a grievance in that the other members of his gang had been sentenced to a lesser sentence of 3½ years’ imprisonment by a different judge. Held : There was no error of principle or in approach by the judge who sentenced the Applicant. Although the Applicant might well have received a shorter sentence if he had been sentenced by the other judge, the function of the court was not to criticise the starting point adopted by that other judge but to consider whether the judge in this case was justified in imposing a term of 4 years’ imprisonment upon the Applicant. Result -

Application dismissed. [See also Hong Kong, 3rd ed, at pp 167-168: Ed].

313

Sentencing

in

CCAB 2000

MA 751/1999

Sentence (Principles)

LEUNG Chun-choi

Gall J

Policeman convicted of AOABH/Propriety of custodial sentence/Position of police officer in prison/Effect of loss of position and pension 警員被裁定襲擊他人致造成身體傷害罪名成立 - 監禁刑罰是否恰當 - 警務人員入獄的處境 - 失去職位及退休金的影響

(1.6.2000) The Appellant, a serving police officer who was off-duty at the material time, was convicted after trial of an offence of assault occasioning actual bodily harm, and sentenced to 3 months’ imprisonment.

*Sin Pui-ha #J Chandler

On appeal, it was submitted that although a sentence of 3 months was appropriate, the term should be suspended. It was said that the magistrate should have given greater credit for the exemplary background of the Appellant: his 20 years service with the police force and the potential loss of his pension and position, and the particular difficulty that a police officer might face in prison. Held : (1) The loss of pension and loss of position were matters which it was open to the court to take into account: AG v Poon Ping-kwok & Another [1992] 2 HKCLR 231; (2) Although the position of the Appellant in prison could be taken account of, it had also to be taken into account that special provisions were made in prison for the security of prisoners; (3) As against the mitigation, the public had expectations of its police force, and where there was actual bodily harm a prison sentence usually followed. Result - Appeal dismissed.

AR 2/2000 Stuart-Moore VP Leong & Wong JJA (11.8.2000) *M Blanchflower & G Goodman # Stephen Chan (1, 4, 6, 7 & 8) Desmond Keane SC (2) Peter Wan (3) Nelson Lam (5)

SJ v (1) AU Man-kei, Alex (2) SHUM Hing-on, Geoffrey (3) CHEUNG Lai-kuen (4) CHU Doi-ying (5) NGAI Lai-hung (6) YIU Ka-man (7) CHOY Ka-ki (8) NG Kaming

Theft/‘Model agency fraud’/Effect of multiple charges/Delay as mitigation/Thefts not to be equated with credit card fraud sentences/Sentencing to be based upon charges preferred 盜竊罪 - ‘模特兒公司詐騙案 ’ - 多項控罪的影響 - 以案件拖延 過久作為求情理由 - 盜竊與信用卡詐騙在判刑方面並不相同 - 須基 於提控的罪行判刑 The Respondents were convicted after trial of a number of theft offences. R1 – R3 were each convicted of 31 charges of theft, and were sentenced, respectively, to terms of imprisonment of 3 years and 9 months; 3 years; and 18 months. R4 and R5 were each sentenced to 8 months’ imprisonment in respect of 6 charges of theft and 7 charges of theft respectively. R6 was sentenced to 9 months’ imprisonment for 10 charges of theft. R7 was sentenced to 2 months’ imprisonment for three charges of theft, and R8 was sentenced to 4 months’ imprisonment, suspended for 2 years, in respect of 4 charges of theft. The judge also made a compensation order against R1 – R3 in the sum of $654,000 in favour of the victims. The facts showed that the victims were induced to believe that they were being recruited as ‘models’ by employees of a business named ‘Star Art’ which effectively purported to be a model agency but was, in reality, a dishonest ‘front’ to persuade victims to part with their money. R1 – R3 were the directors of Star Art. The managers were R4 – R8. There was one group of people involved in the dishonest scheme, who were best described as ‘recruiters’, whose job it was to entice members of the public to go into Star Art’s premises for the purpose of being interviewed by the managers. Substantial sums of money were promised to those who successfully passed their 314

CCAB 2000

Sentence (Principles)

interviews. The victims were told that these sums could be offset against an initial deposit, referred to as a ‘membership fee’, they had been persuaded to pay in order to be taken on by Star Art. In the event that no work was forthcoming within twelve months of recruitment, victims were told the so-called membership fee would be reimbursed. Victims were asked to sign documents which gave their apparent consent to the conditions imposed by Star Art. In sentencing, the judge made an enhancement of the sentences under section 27(11) of the Organised and Serious Crimes Ordinance (‘OSCO’), Cap 455 in the cases of R1 and R2. It was apparent that he also gave credit to the ‘good character’ of the Respondents, and took into account that the matter had been ‘hanging over the defendants’ heads’. On review, it was submitted that the overall sentences imposed on the defendants failed to reflect the seriousness and overall effect of the criminality involved. Held : (1) This was an instance of ‘model agency fraud’, and Star Art milked a great deal of money from its victims in the months during which it operated dishonestly. It was abundantly plain that all the sentences were lenient and, in all cases where discounts were awarded to the Respondents, these were not only generous but contrary to normal sentencing policy and practice. The discounts were awarded despite the constant repetition of offences which had been committed, and after a heavily fought case lasting for over 60 days in court where not a hint of remorse had at any stage been shown by any of the Respondents. Star Art was a bogus company which existed purely to steal money from unsuspecting persons recruited by the staff. R1 and R3 were responsible for the planning, organisation and general supervision of the staff. R2 acted as a banker and recruiter of managers and people to recruit ‘models’. R4 – R8 were all managers who played important but lesser roles compared to the directors. Nevertheless, they were essential to the success of the scheme which deprived many who could least afford it of their money. A one-third discount was usually reserved only for cases where there had been a timely plea of guilty; (2) In Shum Ming & Others v R [1978] HKLR 41, it had been said that it was difficult to regard as first offenders persons who, although appearing before the court for the first time, were charged with a number of offences. That approach remained the one to adopt; (3) That the matter had been ‘hanging over the defendants’ heads’ should not have been considered as a mitigating circumstance. The case was complicated and difficult to unravel, witnesses were not easy to trace and it was not a case where any cooperation had been given by a single defendant. If a defendant had cooperated and had been kept waiting for three years before he or she had been able to plead guilty because others were fighting the case, that would have been a compelling circumstance in mitigation. The long delay before being sentenced was of the defendants’ own making; (4) Although these thefts were mean offences, an analogy could not be drawn, for sentencing purposes, with the tariffs imposed for fraudulent use of credit cards, since such offences were particularly serious as they affected Hong Kong’s commercial success; (5) Although it might be the case that in reality these offences were specimen offences providing merely examples taken from many hundreds, that was one factor amongst others which enabled the prosecution, in presenting an overall picture of their case, to apply for enhanced sentences under the OSCO, having conclusively established that Star Art was a wholly bogus company; 315

CCAB 2000

Sentence (Principles)

(6) The starting point for sentencing purposes had to be determined upon the basis of the charges on which the Respondents had been convicted : R v Chow Tat-ming [1997] HKLRD 353 considered. The prosecution could have proceeded on the charge of conspiracy to defraud, which would have enabled the judge to sentence them on a far wider basis. Despite the leniency of these sentences, they could not properly be described as manifestly inadequate bearing in mind the type of theft alleged in the case and the losses incurred by the victims. Result - SJ’s application refused. Per cur -

CA 564/99 Stuart-Moore VP Leong & Wong JJA (21.7.2000) *G Di Fazio #D TollidayWright (1) Lau Yiu-nam (2)

(1) LO Chi-yip (2) CHOI Ping-wing

There was no power to make orders of compensation or costs from the monies which had been made the subject of a restraint order under section 10 of OSCO, as those monies could only be applied towards confiscation under section 19(2) of OSCO.

Guilty plea/Rationale for discount of one-third/Comments on withholding of full discount 承認控罪 - 三分一刑期減免的理據 - 就法庭不給予十足刑期減免這 點作出評論 The Applicants were jointly charged with an offence of blackmail and an offence of wounding with intent. On the first day of trial, A1 pleaded guilty to the second charge of wounding. After trial, A1 was convicted of blackmail and A2 was convicted of both charges. They were each sentenced to 6 years’ imprisonment on the wounding charge and to 2 years’ imprisonment on the blackmail charge, one year of which was to be consecutive to the 6 years, making 7 years in toto. A1 submitted, inter alia, that he should have been given the full one-third discount for his timely plea, and that the starting point for the wounding charge was too high having regard to the injuries not being life-threatening. It was also said that the sentences should have been concurrent. A2 contended that the sentence was manifestly excessive, and that concurrent sentences were appropriate. Held : (1) It was a cause of great dismay to the court that it was again confronted by a judge in the District Court who had failed to accord a defendant the customary one-third discount for a plea of guilty where no good reason was shown for a departure from the practice, despite the number of times the court had had occasion to deal with the topic; (2) Although the fact that A1 had been arrested ‘blatantly red handed’ was once considered to be a good ground for reducing the discount that otherwise would have been given, that had not been so for some time; (3) Although the reduced discount was also sought to be justified on the ground that the late plea was entered on ‘the very first day of the trial’, the court did not wish to encourage the view that in some circumstances this might justify a slightly reduced discount. This depended entirely on all the circumstances of individual cases, particularly when considering whether a plea at court on the first day of trial had been a technical manoeuvre resulting in frightened or reluctant witnesses being forced to come to court, or resulting in a prolongation of an investigation that could have been curtailed far earlier if a plea had been indicated at the first opportunity. Those were examples and the list was not exhaustive; 316

CCAB 2000

Sentence (Principles)

(4) In this case, the blackmail charge was to be contested so that, in any event, it made no difference that the plea to the first charge was entered on the day of trial as the private witnesses were to an extent common to both charges. A1 had not jumped bail or contested a voire dire and, for all practical purposes, this was, in the particular circumstances of the case, a timely plea in the broadest sense; (5) It appeared that the principle that being caught red-handed was not a sufficient reason to disallow the full discount where an offender pleaded guilty needed to be stated again. Counsel had to be able to advise their clients with full confidence as to this aspect of sentencing, and it was neither fair nor sensible that this principle should be dependent upon the whim of the judge before whom the offender appeared as to whether the full discount or less than the full discount should be given merely because an offender had been caught in the act. Counsel could not properly advise his client that he would receive a discount of one-third if, before one judge, it might be decided that being caught red-handed deserved a discount of short of one-third and yet, before another judge, it might be decided that the full discount was appropriate. That was an important rule of practice in order to achieve fairness and parity towards all defendants. In any event, the ingenuity of some defences had shown that there was no such thing, in terms of practical reality, as a case that was so open and shut that no defence could ever be advanced. The obvious example was that of the drug trafficker caught holding a bag of drugs who could always allege a fabricated account had been given against him. In the present case, although A1 could easily have concocted defences in an attempt to avoid justice, he pleaded guilty and should have received the full discount for his plea; (6) The sentencing practice which used to allow for reduced discounts in cases where the offender had been caught red-handed had long since been disapproved: HKSAR v Wong Ka-kuen and Another Cr App 35/98, Sentencing in Hong Kong, 3rd ed, at p 253. Result - Appeal of A1 allowed. Sentence of 2 years on the blackmail charge to run consecutively to 4 years and 8 months’ imprisonment on the wounding charge, making a total of 6 years and 8 months’ imprisonment. Application of A2 dismissed.

317

CCAB 2000

CA 195/2000 Stuart-Moore VP Leong & Wong JJA

Sentence (Principles)

HOANG Thi Thu Huyen

Offence committed whilst accused on bail for earlier offence/ Totality not relevant to sentencing exercise/Consecutive sentence appropriate 被告於早前所犯罪行的保釋期間犯罪 - 量刑時不涉及整體刑期原則 - 刑期分期執行是恰當的 On 2 May 2000, the Applicant pleaded guilty to a charge of trafficking in a mixture containing 47.24 grammes of heroin hydrochloride. She was sentenced on 16 May 2000 to 4 years’ imprisonment.

(4.8.2000) *Louisa Lai

When he sentenced the Applicant, the judge observed:

#I/P

Defence counsel also urges me to take into account the totality principle, insofar as the defendant has just commenced a sentence of imprisonment for theft. For the present offence I select from the guidelines a starting point of 7 years and discount that by one-third for the plea of guilty, leading to a result of 4 years and 8 months. I discount that by a further 8 months in accordance with the totality principle, and in the result I impose 4 years’ imprisonment, that sentence to be consecutive to the sentence at present being served. On appeal, it was submitted that the sentence was too heavy and that leniency ought to be shown to her because her mother was seriously ill in Vietnam. Held : The offence was committed whilst the Applicant was on bail for a theft offence which had been committed on 13 September 1999. It was very difficult to see why totality played any part at all in the sentencing exercise undertaken by the judge. The present offence was entirely separate to the other matter and was, in any event, committed whilst the Applicant was on bail. In such circumstances it was almost inevitable that the sentence for this offence should have been ordered to run consecutively. If the overall effect produced a sentence which was considered to be too long then a part of the sentence could have been ordered to run concurrently. The Applicant was remarkably fortunate to have had the benefit of a somewhat unusual approach taken by the judge. Result - Application dismissed.

CA 200/2000 Leong, Wong & Keith JJA

(1) CHEUNG Chi-wai (2) FUNG Hiu-chong

Training centre/Whether community service order appropriate alternative for grave offence/Comments on suitability of pregnant offender for training centre 教導所 - 就嚴重罪行而言,社會服務令是否適當的替代刑罰 - 就是 否適宜判懷孕罪犯進入教導所一事作出評論

(5.9.2000) *Sin Pui-ha & Tam Sze-lok #Michael Lunn SC & Freddy Woon (A1) A2 I/P

The Applicants were convicted after trial of offences of blackmail and theft. As they were aged, respectively, 19 years and 17 years, the judge called for pre-sentencing reports. They were subsequently sentenced to detention in a training centre. It was the case for the prosecution that the Applicants had been engaged in an unpleasant scheme to exploit the vulnerability of those who engaged the services of prostitutes. When a client requested the services of a prostitute, she would visit his home or hotel room and demand payment of a sum greater than had been agreed for her services. The client would be threatened with violence if he refused to pay. Those threats would be made over the telephone or by a man in the street when the client went to a cash dispenser to obtain the money demanded. The Applicants were each involved in such a scheme on two occasions. 318

CCAB 2000

Sentence (Principles)

The judge took the view that the offences committed by the Applicants were so serious that despite their age they had to receive custodial sentences. However, by ordering their detention in the training centre he apparently concluded, in the light of the language of s 4(1) of the Training Centres Ordinance, Cap 280, that it was expedient for their reformation and for the prevention of crime that they should undergo a period of training in a training centre. A1 submitted that the judge was in error in not having considered a community service order as an alternative to a custodial sentence. The judge was said to have fallen into the same error as had the judge in HKSAR v Chow Tak-man [1999] 2 HKC 659, who was said, at 663, ‘not to have addressed the particular matter and to have regarded himself as simply precluded from imposing a community service upon the basis [that] burglary was a serious offence and that a custodial sentence was inevitable’. A2 submitted that she came from a stable working-class, and that she got into the wrong crowd after failing to achieve at secondary school. As she was also four months’ pregnant, the pre-sentence assessment panel officer at the institution at which she was held pending sentence considered her unsuitable for detention in a training centre. However, the judge concluded otherwise and said: Pregnant women are regularly imprisoned or found to be pregnant after imprisonment. The authorities have medical facilities and tailor work programmes accordingly. The original concept of training centre was that they were to be institutions short of imprisonment, to which young persons could be sent with a hope of reformation, and where they could be kept apart from more hardened adult prisoners. That principle, in my view, remains paramount and the authorities should be prepared to tailor physical activities and medical care to accord with pregnancy, and not the other way round. Held : (1) Although it was true, as A1 submitted, that the judge did not refer to the community service option in his sentencing remarks, that did not mean that he did not consider it. The judge had to be assumed to have concluded that community service was not, in this case, a viable alternative to a custodial sentence, and he was right. In Chow Chak-man the Court of Appeal approved the guidance given by the English Court of Appeal in R v Brown [1981] 3 Cr App R (S) 294 as to those offenders who could be said to be best suited for a community service order. Some of the features identified in that case could be said to apply to A1, but the nature of his offences, in particular his involvement at a professional level with vice activities and organised crime, took away much of the benefit to be derived from his lack of previous convictions hitherto and his previous work record; (2) The judge did not err in sentencing A1 to detention in a training centre. The nature of the offences he committed made a custodial sentence inevitable, and although he was a real candidate for the strict regime to be experienced in the training centre, on one view he was lucky to avoid a sentence of imprisonment; (3) Although detention in a training centre was also the most appropriate way dealing with A2, her pregnancy meant that such a disposal was less appropriate than would otherwise have been the case for two reasons. First, the regime in the training centre was physically demanding, and her condition would, for the first few months at any rate, prevent her from firstly participating 319

CCAB 2000

Sentence (Principles)

in the activities of the training centre. Second, as said by Cross & Cheung, in ‘Sentencing in Hong Kong’, 3rd ed, at p 465: A court ….. should think long and hard before sending a pregnant woman to the training centre. Such an institution has no proper facilities for babies, and the separation of mother and child at some stage will be inevitable; (4) The inability of A2 to take part in the physical activities of a training centre would be over within a few months after she had given birth and confinement; (5) The separation of mother and child caused greater concern. Rule 21 of the Prison Rules - which applied to training centres as well as to prisons, by virtue of s 8(1)(b) of the Training Centres Ordinance – enabled the authorities to permit a mother to keep her baby with her in prison until it reached 9 months, and apply thereafter for the baby to remain with her until the age of three - the difficulty was that this facility was not available in training centres. One view of the matter was that this should not deter the court from sentencing a pregnant offender to detention in a training centre if that was the correct sentence to pass. The court should not be deflected from passing the appropriate sentence simply because of the lack of facilities for mothers and babies. If the sentence was an appropriate one, it was for the authorities to make those facilities available. On the other hand, if mother and child did not have to be separated because such facilities were made available, the point of sending A2 to a training centre would be defeated. She would have to devote so much time to nursing and caring for her baby that she would not receive the training which she was sent to the training centre to receive; (6) The only viable sentencing options - if mother and child were not to be separated - were to convert A2’s sentence to one of imprisonment or to place her on probation. It might be that the 4 months during which A2 had been undergoing training might have had a salutary effect upon her, and provided her with the insight into her behaviour which she was lacking. A further report would therefore be needed for the purpose of obtaining up-to-date information (a) as to whether her insight into her behaviour had improved to such an extent as to enable probation to provide her with the counselling she needed to lead a productive and crime-free life, and (b) as to what were her future plans for the child. It was also necessary to know from the Correctional Services Department whether it was feasible for A2 to keep her baby in the training centre, and what facilities would be available to her there. Result - Application of A1 dismissed. Application of A2 adjourned to a date to be fixed.

320

CCAB 2000

CA 315/2000 Stuart-Moore ACJHC Rogers VP (11.10.2000) *M Blanchflower #I/P

Sentence (Principles)

CHEUNG Ka-lon

Training centre/Drug addict sent to training centre/Comments on desirability of legislative amendment to allow addicts to complete DATC 教導所 - 吸毒者被判進入教導所 - 就是否宜修訂法例以容許吸毒者 在戒毒所完成治療的問題作出評論 The Applicant, aged 19 years, pleaded guilty in the District Court to a charge of burglary. He was sentenced to detention in a detention centre. On appeal, the Applicant expressed the hope that the court would impose a drug addiction treatment centre (DATC) order because, at the time when he was sentenced, he was already subject to such an order. The Applicant also hoped to be given a concurrent order at DATC because his girlfriend was pregnant and because at the time of the offence he was so affected by the drug known colloquially as ‘cross’ that he really did not know what he was doing. Held : As the circumstances which confronted the judge amounted to a familiar sentencing problem which confronted judges in the District Court, it was appropriate to repeat what the judge had to say in his Reasons for Sentence: In January of this year, you committed an attempted theft. It was not very serious. You were bailed until 5 April. On that day you were put on probation. In the meantime in February, you had committed a further attempted theft, a more serious one, going into premises and looking in showcases in the place where mobile telephones were kept. You were bailed in respect of that. In the early hours of the morning before your appearance on 5 April, you were out burgling, trying to get into a shop with a crowbar. I have got reports on you. It reveals that you take almost any drug that you come into contact with, that you have been leading what is called a loafing lifestyle, that you have turned your back upon and ignored the good advice of your parents, and the history reveals that being caught, arrested, brought to court and the like has absolutely no effect upon your criminal conduct. One only has to recite those facts to make it absolutely apparent that you are a young man - because you are only 19 - who needs training. After those crimes that I have described, you committed the further offence of being in possession of drugs. You are in DATC at the moment. I am asked to leave you there but I cannot do that. The crime you committed of seeking to burgle commercial premises, has a starting point of up to 2½ years’ imprisonment. If you were not addicted to drugs and behaved in the same way, you could have expected, with your record, to have gone to Training Centre. The idea that the extra vice of being addicted to drugs should result in you being locked up for less time, only has to be stated to be seen to be unreasonable. I appreciated the report from the Correctional Service says that you are unsuitable for Training Centre, but the sole reason for that is your drug dependency. But I regret to say that, for the reason I have just given, is not sufficient reason to leave you in DATC and not give you the proper sentence for the crime you committed. The problem of what to do with young men who are drug addicted but need to go to Training Centre is one that has vexed the courts for a long time now. It is a matter of regret that the DATC element of your incarceration has to finish today because I 321

CCAB 2000

Sentence (Principles)

send you to Training Centre. The same authority runs both institutions but one has to treat them, under the present rules and law, completely separately. That is a matter of regret. It is as long ago as 1987 the Court of Appeal, in the case of Lee Yuen-yee, which was Appeal Case No. 346 of that year, was complaining that the court did not have the power to do the sensible thing, which was to let you do your DATC, then go for training. That is what ought to happen. But despite the weight of that suggestion from the Court of Appeal in 1987, no one has done anything about it and we are faced again with this problem that we have now. I can walk out of my chambers this morning, putting my hands on two judgments from the Court of Appeal which happened to be cases from me, that is why they were in my chambers, they are 1997/746 Criminal Appeal and 1998/765 Criminal Appeal, where I was faced with virtually the identical situation that I am faced with you today. In both of those cases, I said much what I have said today, and the Court of Appeal were kind enough to say that they agreed entirely with what I said. But despite that, the rules have not changed. If it be that having a drug addict in their Training Centre programme causes the Correctional Services some extra problems, I regret it, but in the end, since they can prompt change in legislation, it is their fault, not ours. And I have seen enough cases like this for people to know that once they have been caught once and they are drug addicts, they know they are going to go to DATC. They think that it is a licence then to commit further crime because they will not go to Training Centre. Anyone who has that perception must know it is wrong. Please do not think that I have forgotten the positive things. I have remembered you co-operated. You were sensible enough to plead guilty and the like, but even with those in the scales, Training Centre, in my judgment, is obviously the right sentence. The Court fully agreed with the judge’s sentence. Result – Application dismissed.

MA 853/2000 Beeson J

NG Wan-lung

Compensation order/Comments on use of compensation order/ Combination of penalties/Duration of payments 補償令 - 就補償令的使用作出評論 - 判刑包含了不同類型的刑罰 還款的期限

(12.9.2000) *Ian McWalters

The Appellant pleaded guilty to a charge of theft. He was sentenced to detention in a detention centre. A compensation order was also imposed under s 98 of the Magistrates Ordinance, requiring the Appellant to compensate the victim in the sum of $70,960.

#I/P The facts showed that the Appellant tricked his way into the home of a friend and stole jewellery, valued at $70,960, from the friend’s mother. The jewellery was sold for about $9,000 to be melted down, and the money was used to buy a second hand mobile phone and a pager which were later sold. The Appellant had a clear record, although he had been under a Care and Protection Order, and the magistrate sought various reports. The Appellant was allowed to remain on bail pre-trial for a longer period, so that he could make an attempt at restitution. However, by the time of sentencing, some 21 months after the offence, no restitution had been made. The magistrate, with the consent of 322

CCAB 2000

Sentence (Principles)

the Appellant, took $10,000 from the bail as part-payment of the compensation to the victim and then made a compensation order, which was formulated to take effect after the Appellant had completed his period in the detention centre. On appeal Held : (1) As to whether the compensation order was realistic, s 98 did not preclude the magistrate from making such an order together with a detention centre order, nor for making it in such a sum, as the maximum for such an order was $100,000; (2) The Appellant had the opportunity to make recompense to the victim but chose not to do so. The magistrate made proper enquiries into his means and the type of work he was doing before making the order and before making the monthly repayments. The victim had lost a substantial amount of property and it was incumbent upon the Appellant to meet the compensation order and to face the consequences of what his crime meant to the victim; (3) Although the Appellant said he was not alone in the crime and he proposed to testify against the person who committed the offence with him, he could be ordered to pay compensation of the whole amount and he had the opportunity, if he so wished, to ask his companion in crime to make a due contribution; (4) The magistrate was aware of the fact that an order for compensation should not stretch over too long a period, and set the instalments to last for a period of 12 months starting from 10 April 2001. Result - Appeal dismissed. [For a consideration of the use of the compensation order, see Sentencing in Hong Kong, 3rd ed, at pp 80-86:Ed]

MA 1233/1999 Suffiad J

CHAN Chong-chi

Appellant sentenced to 3 months’ imprisonment/Accomplices received suspended sentence/Disparity in sentence and other mitigating factors/Whether exceptional circumstances 上訴人被判處監禁3個月 - 各從犯獲准緩刑 - 判刑差異與其他減刑 因素 - 本案是否有涉及特殊情況

(11.10.2000) *Alex Lee #Juliana Chow

In November 1999, the Appellant pleaded guilty to three charges (being charges 4, 5 and 6) of offering an advantage to an agent, contrary to s 9(2)(a) and s 12(1) of the Prevention of Bribery Ordinance, Cap 201, and was sentenced to three months’ imprisonment on each charge, all the sentences to be served concurrently. He appealed against sentence. The facts showed the Appellant was, at the material times, a plumber who was contracted by the Incorporated Owners of Carado Garden (IOCG) to carry out plumbing maintenance works at Carado Garden. He paid money to Wong, Chan, Chak and Yu (being estate managers, assistant estate manager and/or manager of IOCG) for their assistance in obtaining plumbing works. Chan was subsequently charged with two offences of accepting advantages from the Appellant. He pleaded guilty to the charges in September 1997 and was sentenced to one month’s imprisonment on each charge, suspended for 12 months, and fined $4,000 in total. Wong was charged with similar offences. He was tried and found guilty in December 1997 and was sentenced to one month’s imprisonment on each charge, suspended for 12 months, and fined $4,000 in total. 323

CCAB 2000

Sentence (Principles)

Chak pleaded guilty to charges of accepting advantages from the Appellant and was sentenced in May 1999 to four months’ imprisonment, on each charge, suspended for 12 months. In July 2000, the Appellant gave evidence at the trial of Yu, who was convicted and sentenced to five months’ imprisonment, solely on the evidence of the Appellant. On appeal, it was submitted, inter alia, that there was delay in bringing these charges against the Appellant and the magistrate had failed to pass a sentence which was consistent with the sentences passed on the other assistant estate managers and the contractor. Held : (1) The magistrate dealt with the issue of disparity by relying on the decision in R v So Hung Lee [1986] HKLR 1049. In so doing, the magistrate erred in principle in that he failed to take into consideration another well established principle of sentencing that where a sentence on an appellant was so disparate to the sentence of another participant in the same or related offences that he might be considered to have a justified sense of grievance, then an appellate court might reduce that sentence even though it was not in itself excessive: R v Potter [1977] CLR 112; (2) The three other persons charged with similar offences which arose out of the same investigation had their prison sentences suspended. To ignore this matter completely was to shut out the reality of the situation; (3) Each one of the mitigation factors on its own might not have amounted to an exceptional circumstance, but when they were all considered together, their cumulative effect ought to have been such as to lead the magistrate to the obvious conclusion that there were exceptional circumstances present in this case; (4) The Appellant gave evidence for the prosecution against Yu in July 2000. It took place after he had been dealt with by the magistrate. Nevertheless, it was still a matter to which the appellate court could give consideration. Result -

Appeal Allowed. Sentence imposed to be suspended for a period of 12 months.

324

CCAB 2000

Sentence (Principles) 香港特別行政區訴陳隆池 HKSAR v CHAN Lung-chi 高等法院上訴法庭 – 刑事上訴2 0 0 0 年第2 3 2 號

* 李紹強及 王詩麗 R S K Lee & Wong Sze-lai # 陳嘉信 W Chan

高等法院上訴法庭法官王見秋 高等法院上訴法庭法官胡國興 高等法院原訟法庭法官張澤祐 聆訊日期︰二零零零年十月十二日、十一月二日及九日 宣判日期︰二零零零年十一月九日 COURT OF APPEAL OF THE HIGH COURT CRIMINAL APPEAL NO. 232 OF 2000 WONG JA, WOO JA, CHEUNG J Dates of Hearing: 12 October and 2 & 9 November 2000 Date of Judgment: 9 November 2000 建 議 的 入 院 治 療 期 為 3 個 月 , 但法庭卻判以為期12個月的入院令 判令是否有理據 - 應用《精神健康條例》第36條 上 訴 人 經 審 訊 後 被 裁 定 一 項 有 意 圖 而 傷 人 罪 罪 名 成 立,違反 香港法例第212 章《侵害人身罪條例》第17 條。原審法官判處上訴 人1 2 個月的入院令。他不服判刑提出上訴。 據案情顯示,上訴人與受害人是同一幢大廈上下層的鄰 居。他們經常因噪音問題互相感到不滿。1999年11月22日,他們 又因噪音問題爭吵。受害人拿晾衣鐵到樓下找上訴人理論。 上訴人取出一條鐵水管,追受害人,並多次用水管打受害人。 原審法官在判刑時要求兩名精神科醫生提交醫療報告。兩 名醫生同樣建議,如法庭認為適當的話,應判處上訴人入院接受 治療3個月。經考慮兩名精神科醫生所擬備的報告後,原審法官 說︰“基於被告採用的武力及事主所受的損傷,本席應考慮判被 告監禁4年。……基於事主的挑釁行徑,本席可以將被告的刑期減 為3年。……本案的起因也基於被告的精神問題。如本席把被告收

監,雖然可以以儆效尤,防範重蹈覆轍之誤,但未能解決被告的 個人問題,對社會或被告未必是最佳的處理方法。……由於被告 對外界的言詞反應激烈,異乎社會一般巿民大眾所能接受,本席 深信被告需要一個較長的治療時間。” 上訴時,代表上訴人的律師提出:(1)原審法官既然已決定 用一種可以治療上訴人及可以令上訴人改過自新的方式來判刑, 理應採納兩位精神科醫生的建議,判上訴人一個為期3個月的入院 令;(2)考慮到這案件一切有關情況,原審法官判罰上訴人之12個 月入院令是犯了原則上的錯誤或是明顯地過重。 法庭准許陳醫生所擬備的兩份進度報告呈堂。該兩份報告 顯示留院治療不是必須的。為了解上訴人是否可以接受有效的院 外治療,以及他如不受羈留是否會對其他人構成危險,法庭於是 把案件押後,以便感化主任及兩名精神科醫生就上訴人的情況提 交報告。 恢復聆訊時,法庭考慮了感化主任及兩名精神科醫生的 報告。感化主任的報告認為上訴人需要接受住院治療,不適宜接 受感化。兩名精神科醫生同樣在報告中指出,上訴人需要在一所 普通的精神病院接受治療,建議法庭可作出第136章《精神健康 條例》第36條的命令,把上訴人留在葵涌醫院繼續接受治療。 法庭打算接納有關建議。案件再度押後,以便有關醫生及醫院有 325

CCAB 2000

Sentence (Principles) 時間辦妥所需手續及準備工作。法庭於恢復聆訊時說明裁決理 由。 裁決︰ (1) 兩名精神科醫生在他們擬備的心理報告中同樣建議如判 處入院令,該令應為期3個月,因此在原審法官席前,並無任何 證據支持需要一年長的精神治療及心理輔導。在此方面,原審法 官犯了在R v LAM Yuen-siu [1993] 2 HKC 272一案的同樣錯 誤; (2) 醫生的證據或專業意見應以醫學為據,而在考慮及指出被 定罪者需要接受留院治療的期間時,最好能夠指出最長和最短的 時間,而不應提及入院令的期間,因為入院令期間的長短是由法 庭而非醫生決定; (3) 法 庭 須 按 《 精 神 健 康 條 例 》 第 45(1)(c) 條 的 規 定,經“顧 及全部情況,包括罪行的性質,該人的品格及履歷,以及其他可 用的處置該人的辦法後”,才可決定入院令是否最合適處理案件 的辦法和入院令的合適羈留期間; (4) 上訴人已在小欖精神病治療中心接受了近6個月的羈留治 療。雖然原審法官判上訴人為期一年的精神治療及心理輔導入院 令是犯錯的,但為了照顧上訴人及確保他不會因患精神病未癒而 對受害人或社會其他人士作無理襲擊,法庭認為根據第36條的規 定,下令上訴人進入葵涌醫院或另一所合適的精神病院接受羈留 治療是適當的; (5) 一年的入院令予以撤銷,改判以第36條的命令,而這個命 令按該條執行所需的證明書,需由兩名註冊醫生簽署,並於同日 送交原審法官或另一名區域法院法官加簽。 上訴得直。入院令取消,改判第36條的命令。

[English digest of CACC 232/ 2000 above] Wong Woo JJA & Cheung J

CHAN Lung-chi

Hospital order of 12 months imposed after 3 months recommended/Whether order justified/Use of s 36 of Mental Health Ordinance The Appellant was convicted after trial of a charge of wounding with intent, contrary to s 17 of the Offences Against the Person Ordinance, Cap 212. The trial judge imposed a 12-month hospital order on the Appellant. He appealed against the sentence.

(9.11.2000) *R S K Lee & Wong Sze-lai #W Chan

The facts showed that the Appellant and the victim were neighbours living on adjoining floors of the same building. They were always unhappy with each other because of the noise problem. On 22 November 1999, they quarrelled again over the same problem. The victim took a lift-fork and went downstairs to argue with the Appellant. The Appellant took out an iron pipe, chased the victim and hit her several times with the pipe. The trial judge, upon sentencing, called for medical reports from two psychiatrists. Both doctors recommended that the court should, if it deemed fit, impose a hospital order for a period of three months. Having considered the reports prepared by the two psychiatrists, the trial judge said that ‘Given the violence of the defendant and the injuries suffered by the victim, I should have passed a sentence of 4 years imprisonment on the defendant. …Because of the act of provocation on the part of the victim, I could reduce the defendant’s terms of imprisonment to 3 years. … the defendant’s mental illness was also the cause of the incident. If I were to send the defendant to jail, it might be able to 326

CCAB 2000

Sentence (Principles) bring about deterrence and prevent others from committing the same offence, but still it could not solve the personal problem faced by the defendant, nor could it be the best way of handling the case in so far as the community or the defendant is concerned….The over-reaction of the defendant in response to other’s comments was hardly acceptable to the public. I am convinced that he would need a longer term of treatment’. On appeal, it was submitted that (1) having decided to sentence the Appellant based on treatment and rehabilitation, the trial judge ought to have adopted the recommendations of the two psychiatrists and imposed a 3-month hospital order on the Appellant; (2) in the premises, the 12-month hospital order imposed by the trial judge on the Appellant was wrong in principle and/or manifestly excessive. Two progress reports on the Appellant prepared by Dr Chan were produced. The reports indicated that hospital treatment was not a must. In order to find out whether the Appellant was suitable for effective out-patient treatment and whether he would remain a danger to others if not detained, the court adjourned the case for the probation officer and two psychiatrists to submit reports on the Appellant. At the resumed hearing, the probation officer’s report and the psychiatrists’ reports were considered. The probation report stated that the Appellant was in need of in-patient treatment, and open probation supervision was considered not suitable for the Appellant. Both psychiatrists stated in their reports that it was necessary for the Appellant to continue to receive treatment in a general psychiatric hospital and recommended the court to make an order under s 36 of the Mental Health Ordinance Cap. 136, to detain the Appellant in Kwai Chung Hospital for further treatment. The recommendation was readily accepted by the court. The case was further adjourned so that the doctors and the institutions concerned would have time to complete the necessary formalities and to make the required arrangement. The court gave the reasons for its ruling at the resumed hearing. Held : (1) It was to be noted that both psychiatrists had recommended in their reports that if a hospital order was to be imposed on the defendant, the order should be a 3-month one. Thus there was no evidence before the trial judge to support the sentence of psychiatric treatment and psychological counselling for a period of one year. In this regard, the trial judge erred in the same manner as occurred in R v Lam Yuet-siu [1993] 2 HKC 272; (2) The evidence or expert opinion given by a medical practitioner should be based on medical findings. In the course of considering and specifying the period of in-patient treatment of a convicted person, it would be desirable if a medical practitioner could specify the maximum and the minimum term of treatment and not the period of time of a hospital order. The duration of such an order was very much a matter within the jurisdiction of a court and not that of a medical practitioner; (3) The court should, in accordance with s45(1)(c) of the Mental Health Ordinance, have regard to ‘all the circumstances, including the nature of the offence and the character and antecedents of such person, and to the other available methods of dealing with him’ before it decided whether the imposition of a hospital order would be the most suitable method of disposing of the case and determined the appropriate period of time of the hospital order; (4) The Appellant had been detained for nearly 6 months in Siu Lam Psychiatric Centre for treatment. Although the trial judge had erred in sentencing the Appellant to a one-year hospital order for mental treatment and psychological counselling, for the well-being of the Appellant and for ensuring 327

CCAB 2000

Sentence (Principles) that he would not, during the recurrence of mental illness, launch attacks on the victim or any other member of the community, an order made under s 36 to detain the Appellant in Kwai Chung Hospital or any other mental hospital might suit him; (5) The one-year hospital order would be substituted by an order under s 36, and that order would be executed in accordance with that section in that the required certificate should be signed by two registered medical practitioners and then forwarded to the trial judge or other District Judge on the same day for countersignature. Result - Appeal allowed. Hospital order set aside. An order under s 36 of the Mental Health Ordinance substituted.

AR 5/2000 Stuart-Moore ACJHC Mayo VP Seagroatt J (29.11.2000) *Arthur Luk & Derek Lai #Phillip Ross

SJ v TSEUNG Mang-ka

Two incidents of robbery and rape/Concurrent sentences wrong in principle for robbery and rape of one victim/ Calculation of sentence/Approach to discount on successful review of sentence 搶劫及強姦罪各兩項 - 就搶劫和強姦同一受害人所判的刑期予以同 期執行在原則上錯誤 - 刑期的計算 - 覆核刑期獲准後的刑期扣減方 法 On 21 May 2000, the Respondent, aged 30, pleaded guilty in the Court of First Instance to two counts of robbery and two counts of rape. Having adjourned sentence to 30 May 2000 in order to obtain a background report, the judge sentenced the Respondent to terms of imprisonment totalling 5½ years. On 22 April 1999, the Respondent robbed and raped his first victim at the same premises in Nathan Road. On 4 May 1999, twelve days after the commission of the earlier offences, he robbed and raped a victim in Lockhart Road. In respect of the first counts of robbery and rape, the judge imposed concurrent terms of imprisonment of 3½ years respectively. On the second counts of robbery and rape, he sentenced the Respondent to 2 years 5 months and 3 years respectively. He then ordered that 2 years of the aggregate terms on the second pair of counts should run consecutively to the total of 3½ years he had imposed on the first pair of counts, making 5½ years’ imprisonment in all. The victim in each case was a prostitute. He effected the first robbery and rape in private premises after threatening the victim with a cutter. Through aggression he was able to commit the second robbery and rape. On review, it was submitted that the sentences imposed were manifestly inadequate.

Held : (1) In relation to the first count of robbery, there was no justification for departing from the starting point of 6 years suggested in Mo Kwong-sang v R [1981] HKLR 610. The offence was a classic case of armed robbery on private premises and the fact, as the judge put it, that the premises were used ‘substantially for non-domestic purposes’ was a wholly irrelevant consideration; (2) When a robbery victim was subjected to a sexual attack, there should inevitably follow a significant increase in sentence. The judge’s approach in passing wholly concurrent sentences for the rape and robbery involved in each incident was an error of principle; (3) It was well known that the practice of the court in regard to giving a downward adjustment of sentence when a review had resulted in an increase of 328

CCAB 2000

Sentence (Principles) sentence had altered since 1987. When a downward adjustment was made, which was not on every review, it was usually only a token one to take into account the element of double jeopardy to which the prisoner had been subjected. The 1987 case upon which the Respondent relied (AG v Dominic Cheung Kai-man [1987] HKLR 788) was irrelevant to the modern-day practice in Hong Kong. The token downward adjustment nowadays usually only applied where there had been a substantial increase in the sentence originally imposed or where the prisoner either had been or was about to be released: AG v To Ka-yin AR 3/96; (4) Each incident involved the robbery, followed by the rape, of a lone woman in private premises. The first of the incidents was additionally aggravated by the production of a knife which was pointed at the victim’s neck. In R v Chan Chi-wah [1989] 2 HKLR 135, it had been said that a 12-year sentence would have been appropriate following pleas of guilty to one count of rape and four counts of robbery. The four victims involved were all nightclub waitresses who were robbed, and in one case raped, as they made their way home in the early hours of the morning. The offences in that case were committed either with the use of a weapon with which to threaten, or with violence, or both; (5) The 5½-year sentence represented an appropriate sentence for the two robberies, where a starting point after trial of about 8½ years’ imprisonment could not have been criticised. The rapes made this far more serious than that. For two rapes of this kind, the Respondent should not have received a sentence of less than 10 years’ imprisonment after trial. Having regard to totality and taking the robberies also into account, the Respondent should have received a sentence of not less than 15 years after trial. Result

- SJ’s application allowed. imprisonment substituted.

Sentences

totalling



years’

Per cur - It was difficult to understand why in a case of this gravity the judge adjourned sentence for a background report to be prepared.

CA 503/99 Stuart-Moore ACJHC Leong & Wong JJA (7.11.2000) *Wong Wingsum #Wong Powing

(1) CHAN Man-chau (2) WONG Kingchuen

Guilty plea/Discount not to be reduced if accused disbelieved at trial of codefendant/Basis of discount/No discount appropriate for mere promise to assist 認罪 - 如被告人在同案被告人的審訊中所作證供不獲接納,被告人 所獲的減刑也不應扣減 - 刑期減免的準則 - 只是答允協助警方不會 獲得刑期減免 The Appellants were jointly charged with trafficking in a dangerous drug. The amount involved was approximately 12.58 kilogrammes of heroin hydrochloride and 156.69 kilogrammes of ice. A2 pleaded guilty. A1 pleaded not guilty. A2 gave evidence for A1 at his trial. A1 was convicted after trial and sentenced to 26 years’ imprisonment. A2, who was sentenced to 20 years’ imprisonment, sought leave to appeal against sentence. The facts which A2 admitted indicated that on the morning of 15 October 1998, police officers saw the two Applicants arrive in a car at the waterfront outside the Cheung Sha Wan Fish Market and wait outside the car. Sometime later, a small boat arrived and a man from the boat came ashore delivering to the two Applicants four carton boxes and five nylon bags of substances. The two Applicants loaded them into their car. Police officers subsequently intercepted the car and found A2 in the seat next to A1 who was the driver. The boxes and nylon bags were found to contain what was subsequently proved to be the dangerous drugs specified in the charge. 329

CCAB 2000

Sentence (Principles) In sentencing A2, the judge indicated that he adopted 27 years as the starting point. He reduced the starting point by 1 year because it would be the first time A2 was sent to prison. He then gave A2 the discount of 6 years for his plea of guilty, arriving at the sentence of 20 years’ imprisonment. The judge reduced the normal discount for plea because A2 gave evidence for A1 which evidence appeared to have been rejected by the jury. On appeal, it was submitted that the judge should not have penalised A2 by reducing his discount for giving evidence for his co-defendant even though his evidence was rejected by the jury, and that he should have been given some discount for his assistance to the police. Held : (1) The purpose of discount for plea was to give the defendant credit for showing remorse and saving court time in proceedings against him. Whether he gave evidence for his co-defendant should have no effect on the credit he would have been given. Even if he did give evidence which eventually was not believed and the trial of the co-defendant had thereby been unnecessarily prolonged, much to the dislike of the trial judge, that was still no justification to penalise him for giving evidence. The fundamental concept was that ‘no man should be discouraged from testifying in a court of law’: R v Yu Hang-kit Cr App 323/95; (2) In R v Lawless [1998] 2 Cr App R (S) 177, it was held to be wrong not to allow a defendant who had pleaded guilty a discount for his plea on the ground that he had given evidence for a defendant who was convicted. Thomas L J said at p 177: It is of course well recognised that unless there are special reasons to the contrary, which sometimes there are, a person who pleads guilty is entitled to credit for his plea of guilty and for his expression of remorse. This appellant did not get that credit. The reason that he did not get it was the fact that he gave evidence on behalf of the co-accused and that evidence was rejected by the jury. As I have said, in our judgment that approach is wrong in principle. Unwittingly the recorder was in truth sentencing this appellant for something in respect of which he had not been convicted, namely telling lies in the witness box. Needless to say a co-defendant is not to be encouraged to go into the witness box and tell a pack of lies. However, it is also important that a co-defendant should not be inhibited from giving evidence on the basis that if that evidence should be disbelieved he will lose the credit which otherwise he would get for his plea. There was no reason why the approach in Lawless should not be followed. The fact that the Applicant’s evidence had been disbelieved was not inconsistent with him being remorseful; (3) Although in R v Wu Pak-kun and Another Cr App 456/85, it had been said that a judge could not be criticised for reducing the discount attaching to the guilty plea because the defendant had no option to plead guilty and had given perjured evidence for his co-defendants at their trial, that case was decided before the now generally accepted principle of giving a one-third discount in sentence to a defendant who pleaded guilty had been established. That rule was necessitated to encourage those who were remorseful to come forth and plead guilty to the charge, thereby saving the time and expenses of a trial and to make clear to them the benefit normally to be derived from taking such a course. A different approach to that adopted in Wu was appropriate due to changes of circumstances;

330

CCAB 2000

Sentence (Principles) (4) The judge erred in giving less than a one-third discount for the plea because he gave evidence for his co-defendant which was disbelieved; (5) The Applicant was not entitled to any discount for assistance to the police. He did no more than promise to give evidence and he gave the police no useful information on which they could act: R v Leung So-lee Cr App 724/96. Result -

Appeal allowed. Sentence of 18 years’ imprisonment substituted.

Sexual Offence MA 479/99 Nguyen J (4.5.2000) *W Tam #G McCoy SC and R Yu

CHAN Wai-hung

Indecent assault/Whether consent vitiated by fraud/Victim not consenting 猥褻侵犯 - 欺詐行為是否令所作的同意無效 - 受害人並沒有給予同 意 The Appellant was convicted after trial of one charge of indecent assault. The prosecution case was that the victim (‘PW1’) was a helper of the School of Continuation of Professional Education of the City University situated at the Centre of Environmental Building (‘the Centre’). She knew that the Appellant was a staff member of the Centre. On 21 October 1998, the Appellant approached PW1 and obtained her telephone number. Two days later, the Appellant telephoned her and told her that he was recruiting models to pose as injured persons. As requested by the Appellant, PW1, wearing a one-piece short dress, went to the Centre for an interview. The Appellant led PW1 to a room inside the Centre and, after talking to her for a while, he told her that she was successful in the interview. The door was then closed and the ‘first aid training’ followed. During the training, the Appellant performed mouth-to-mouth first aid upon PW1; unbuttoned the top few buttons on her dress and asked her to unhook her bra and touched her breasts; massaged the sole of her foot and circled the inner side of her thigh with his hands. The Appellant went through the process again with PW1 and then she left. Later, the Appellant telephoned PW1 and told her that after he had interviewed other candidates, he had decided not to use her, but would pay her for attending the interview. When PW1 went to the Centre to see the Appellant for the payment, she refused to accept cash and insisted on receiving a company cheque. The Appellant replied that the company cheque could not be issued as the company did not organise the function. PW1 then asked the Appellant whether the style of pressing on her heart during the training by using a cross style really existed, and the Appellant insisted that such a practice did exist. A report was later made to the police. The evidence-in-chief of PW1 as regards what the Appellant had done to her during the training was not challenged at all. During cross examination, PW1 said that if she had known beforehand what would be involved during the training, she would not have attended the interview and the training because all that she had been told by the Appellant was that she would be required to pose as an injured person whose bone had been fractured. In the Reasons for Verdict, the magistrate accepted PW1’s evidence of what had happened. He found nothing in her evidence which suggested a 331

CCAB 2000

Sexual Offence probability that she consented or that the Appellant believed that PW1 had consented or would consent. He also found that the Appellant had no belief that PW1 would consent to her chest and inner side of her thigh being touched if she had been aware that the whole thing was a sham. In the Statement of Findings, there was a paragraph which was not in the magistrate’s oral Reasons for Verdict, namely: I am satisfied on PW1’s evidence that even if there was consent or submission on her part, it was obtained by fraud. [R v Lau Chun-hon MA 1499/94]. Therefore, I rejected the defence of consent in this case. On appeal, it was submitted, inter alia, that the reasoning in R v Lau Chun-hon was wrong and that the consent of PW1 was not vitiated by fraud. Held : (1) The proposition that consent obtained by fraud was no consent at all was not true. Fraud would vitiate consent to an act which would otherwise be an assault only where the fraud had induced a mistaken belief as to the identity of the person doing the act or as to the nature or quality of the act: R v Linekar [1995] QB 250 and R v Richardson [1999] QB 444 followed; (2) In this case, the victim was fully aware of what was being done to her by the Appellant. The magistrate found that in fact the victim did not consent to what was being done to her. The magistrate’s reference to the case of Lau Chun-hon was only in the context as it were of an alternative argument, namely, that assuming there was consent or submission, the magistrate held that it was obtained by fraud and therefore vitiated. Result - Appeal dismissed.

FAMC 11/2000 Li CJ Litton & Bokhary PJJ (28.7.2000) *G J X McCoy SC & Raymond Yu #Peter Chapman & William Tam

CHAN Wai-hung

Indecent assault/Consent of complainant given in error induced by lies/Vitiation of consent by deception as to identity of person doing act or as to nature of act/No belief by defendant that complainant consented 猥褻侵犯 - 原訴人被謊言所誘而錯誤地同意 - 作出作為的人的身分 或作為的性質具欺騙成份均使同意失效 - 被告人沒有相信原訴人同 意 The Applicant was convicted in the magistracy of indecent assault. The High Court upheld that conviction. The facts showed that the Applicant performed on the complainant a series of acts which would undoubtedly have amounted to an indecent assault on her by him unless she had consented to them. And such consent as she had given was given in the erroneous belief, induced by his lies, that what he was doing to her was a first-aid demonstration. That emerged from her evidence. He did not testify. It was contended that two questions of law of great and general importance arose. The first was formulated thus: Does a defendant commit the offence of indecent assault contrary to s122(1) Crimes Ordinance, where the defendant procures a complainant, by false pretences or false representations, to consent to an assault that would otherwise be indecent, in which the false pretences or false representations relate not to the nature of the assault but only to the purpose of the assault?

332

CCAB 2000

Sexual Offence The second was formulated thus: In deciding whether the absence of consent by a complainant in a case of indecent assault has been proved, is it correct in law (a)

to decide whether the complainant would have consented to the assault had she known the true purpose of the assault, and

(b)

to require the defendant to demonstrate a probability that he believed that the complainant was consenting, or would have consented, had the complainant known the true purpose of the assault?

Held : (1) It was to be noticed that both questions spoke of the purpose of the act. But was that to the point? The law was clear and must remain so. What vitiated consent to an act was deception either as to the identity of the person doing the act or as to the nature of the act. That was the principle. And the decided cases revealed no difficulty in their application. There was no deception as to identity in the present case. So the decided cases to be considered were those on the question of whether a deception went to the nature or quality of the act. In this area, the law was concerned with the nature of the act and not its purpose; (2) In R v Williams [1923] 1 KB 340, the Court of Criminal Appeal in England held that it was rape where the victim had been deceived into thinking that what was being done to her was not the ordinary act of sexual intercourse but was some medical procedure aimed at giving her relief from a disability from which she suffered. Such deception went to the nature of the act. In R v Harms [1944] 2 DLR 61, the Saskatchewan Court of Appeal held that it was rape where the victim had sexual intercourse with a man who had deceived her into thinking that it constituted medical treatment which she needed. Such deception went to the nature of the act. In Papadimitropoulos v R [1957] 98 CLR 249, the High Court of Australia held that it was not rape where the victim had sexual intercourse with a man who deceived her into thinking that he was her husband when, unknown to her, the ceremony which they had gone through was not a valid marriage ceremony. Such deception did not go to the nature of the act. In Bolduc and Bird v R [1967] 63 DLR (2d) 82, the Supreme Court of Canada held that it was not indecent assault where the complainant had consented to a man’s presence during an intimate examination of her by her doctor, the doctor having falsely represented to her that that man was a medical student. Such deception did not go to the nature of the act. In R v Linekar [1995] QB 251, the Court of Appeal (Criminal Division) in England held that it was not rape where a prostitute had sexual intercourse with a man who had promised to pay her but had never intended to do so. Such deception did not go to the nature of the act. In R v Lau Chun-hon [1995] 2 HKC 599, Wong J held that it was indecent assault where the victim consented to her breasts being fondled by a man who deceived her into thinking that he was conducting a medical examination of her. Such deception went to the nature of the act. In R v Richardson [1999] QB 444, the Court of Appeal (Criminal Division) in England held that it was not assault when a dentist who had been suspended from practice carried out dental treatment on patients who were unaware that she had been suspended. Such deception did not go to the nature of the act. 333

CCAB 2000

Sexual Offence In R v Tabassum 2000 Times Law Report 418, the Court of Appeal (Criminal Division) in England held that it was indecent assault where a woman consented to the touching of her breasts in the mistaken belief that the appellant was medically qualified. He had said that he had received medical training. In truth his medical knowledge, such as it was, had been gained only through his employment as a medical representative. The appellant’s interest was to prepare a database software package to sell to doctors. There was no sexual motive. It did not help the Applicant that Mr Tabassum was held to be guilty of indecent assault. It seemed, however, that even a decision in Mr Tabassum’s favour would not necessarily have helped this Applicant.

Although the decision of the Saskatchewan Court of Appeal in R v Harms was not followed in R v Mobilio [1991] 1 VR 339, it was not necessary to choose between those two cases. All that needed to be said about R v Mobilio in relation to the present case was this: At p.352 the court said: ‘In this case each of the women consented to the applicant introducing the transducer into her vagina in the performance of the act of conducting a transvaginal ultrasound examination. That is precisely what [Mobilio] did.’ In the present case conducting a first-aid demonstration was precisely what the Applicant did not do; (3) It was perfectly plain that the complainant in the present case was deceived as to the nature of the act, and there was simply no basis for suggesting that she might have consented if she had not been so deceived. Nor was there any basis for thinking that the Applicant might have believed that she had consented. It was to be noted that the Applicant did not give evidence. The magistrate found that the Applicant ‘had no belief that [the complainant] would consent to the touching of her chest, the inner side of her thigh, etc., had she been aware it was a sham.’ Reduced to its essentials, that was a finding that the Applicant knew that there was no consent. Such finding was amply supported by the evidence. Result – Application refused.

MA 171/2000 Gall J

LAU Kwai-chung

Indecent assault/Defendant kissing schoolgirl against will/ Whether indecent in circumstances 猥褻侵犯 - 被告人在違反女學生的意願下吻她 - 就情況而言是否猥 褻行為

(15.6.2000) The Appellant was convicted of one charge of indecent assault. *P K Madigan #F C Whitehouse

The facts, which in the main were not in dispute, were that the Appellant was 40 years of age and worked as an office assistant at the Hong Kong Arts Centre in Wanchai. The victim was a girl of 12 years who studied music at the Hong Kong Arts Centre. In the afternoon of 21 September 1999, she went to the Arts Centre to attend a music lesson and to practise. The Appellant, whom she knew as a clerical assistant but not well, took her from the practice room to a room used for the storage of musical instruments. Inside the room, he pushed her against the wall, held her arms and kissed her on the lips for some three to four seconds. She resisted and the Appellant released his grip and allowed the victim to pick up her musical instrument and leave. At some stage during the incident she was given a packet of sweets by the Appellant which he had already placed upon a shelf in the storeroom in advance.

On appeal, it was submitted that the evidence led during the trial did not amount to a sufficiency of evidence for the magistrate to convict the Appellant 334

CCAB 2000

Sexual Offence of indecent assault. It was also contended that the magistrate erred in drawing an inference that the Appellant intended to indecently assault the alleged victim. The Appellant relied on two cases to support the proposition that a kiss in itself was not an indecent act. They were R v Lam Chi-chee MA 783/92, and R v Fong Chi-wai MA 1096/95. Each was a case where a young woman was either kissed or an attempt was made to kiss her in circumstances where the court, in each case, held that there was no accompanying indecency. Whilst the Appellant accepted that he had assaulted the victim, he nonetheless submitted that a kiss without more did not amount to an act which a right-minded person would regard as indecent, and, further, that there was no evidence that the Appellant intended an indecency.

Held : (1) It was trite law that for an indecent assault to take place, there must be first of all an assault, and that it must be accompanied by circumstances which a right-minded person would consider to be indecent: R v Court [1988] 2 WLR 1071; (2) Although a kiss need not of itself be indecent, it was quite obvious, from the authorities, that a kiss itself could, in some circumstances, amount to indecency when there were circumstances which a right-minded person would find indecent. In R v Leeson (1968) 52 Cr App R 185, a kissing of a baby-sitter by her employer in circumstances where he asked for acts of a sexual nature and was rejected was sufficient for the court to find that the acts were indecent; (3) What the magistrate had to consider was a situation where a 40-year old man invited a 12-year old girl to meet him in circumstances which were indicative of a considerable amount of planning. It was a reasonable and proper inference to be drawn that the Appellant considered that the act which was to take place in that room was not one which he was prepared to be seen to be doing in the open and public gaze; (4) The magistrate was entitled to consider whether in all the circumstances a person viewing them of normal and upright mind would consider that to do those acts in the manner that they were done was repugnant and did amount to indecency and that the Appellant had that intention when he did so. Result - Appeal dismissed.

CA 308/99 Stuart-Moore ACJHC Wong & Woo JJA (30.10.2000) *A A Bruce, SC & Stanley Chan #Andrew Macrae, SC

HUNG Wai-tak

Rape/Whether ‘lurking doubt’/Admissibility of recent complaint evidence/Effect of threat on admissibility of complaint/All circumstances to be looked at 強姦 - 是否有‘潛在疑點 ’ - 早期投訴的證據可否獲得接納 - 受 到恐嚇對投訴是否可獲接納這點有何影響 - 須考慮全部情況 The Applicant was convicted of rape after trial. The sole issue at trial was concerned with whether consent had been given to the act of sexual intercourse. The Applicant was sentenced to 5 years’ imprisonment. On appeal, it was submitted, first, that it could be realistically concluded that a lurking doubt existed as to the validity of the conviction. The uncorroborated evidence of the victim (PW1) was described as inherently weak, improbable, unreliable and inconsistent. Second, it was submitted that the judge had erred in ruling that the evidence given by PW2 as to ‘recent complaint’ was admissible. That was because, although the alleged rape of PW1, a domestic helper from the Philippines, occurred on 6 March 1998, she did not telephone PW2, her church leader, to tell her what had happened until 10 March 1998. She explained the delay on the basis that she had been threatened, that she was 335

CCAB 2000

Sexual Offence confused and did not know what to do. She finally complained after she had been told that the Applicant’s wife was to visit Australia, and she feared a similar event might occur in her absence. Held : (1) As regards the issue of ‘lurking doubt’, it was right to have regard to the provisions of s 83 of the Criminal Procedure Ordinance, Cap 221, in order to determine whether, in the circumstances of the case, the conviction should be set aside on the ground that it was unsafe or unsatisfactory. That followed Kwong Kin-hung v R [1997] HKLRD 15, where Lord Stein, in giving the judgment of the Privy Council, said that ultimately the ‘words of the statute must govern the position’. That said, had the court felt a ‘lurking doubt’ existed in relation to this conviction it would be virtually inconceivable that the conviction could be regarded as safe or satisfactory. This conviction could not, however, be regarded as either unsafe or unsatisfactory; (2) Although the complaint of PW1 was not as ‘recent’ as it might have been, the mere fact that there were earlier opportunities for her to have complained could not amount to sufficient reason in itself to render the evidence inadmissible. This aspect of the law was dealt with in R v Valentine [1996] 2 Cr App R 213, where the court, having traced the development of the law in relation to evidence of recent complaint, concluded that: The authorities establish that a complaint can be recent and admissible, although it may not have been made at the first opportunity which presented itself. What is the first reasonable opportunity will depend on the circumstances including the character of the complainant and the relationship between the complainant and the person to whom she complained and the persons to whom she might have complained but did not do so. It is enough if it is the first reasonable opportunity. Further, a complaint will not be inadmissible merely because there has been an earlier complaint, provided that the complaint can fairly be said to have been made as speedily as could reasonably be expected. The complaint has to be made within a reasonable time of the alleged offence and on the first occasion that reasonably offers itself for the complainant concerned to make the complaint that was made in the terms in which it was made. We now have greater understanding that those who are the victims of sexual offences, be they male or female, often need time before they can bring themselves to tell what has been done to them; that some victims will find it impossible to complain to anyone other than a parent or member of their family whereas others may feel it quite impossible to tell their parents or members of their family. That reflected the modern and proper approach to evidence of recent complaint; (3) In deciding whether to admit the evidence in this case the judge was bound to look at all the circumstances of the case, including PW1’s background and her relationship to the Applicant, before making his conclusion as to whether this complaint was made on the first opportunity which reasonably presented itself after the alleged rape. PW1 depended for financial support and her accommodation on the Applicant’s family. Her natural confusion about what to do only changed into positive action when the Applicant’s wife announced her intention to go to Australia; (4) The threat which PW1 had received was particularly unpleasant and was relevant to the admissibility of the evidence of recent complaint as it had made her frightened; (5) After the judge had decided on the admissibility of the evidence in accordance with what was stated in R v Cummings [1948] 1 All ER 551, the jury 336

CCAB 2000

Sexual Offence were entitled, if they regarded the complaint as unreasonably late, to give little credence or weight to the evidence; (6) The submission in relation to the lapse of four and a half days before the complaint was made by PW1 was of sufficient importance that it was proper to grant leave. Result - Appeal dismissed.

MA 656/2000 V Bokhary J

NG Chi-lok

Indecent assault/Defence of innocent contact/Lack of apology insufficient reason to reject defence evidence 猥褻侵犯 - 以無不軌意圖的接觸作為辯護 - 以被告沒有道歉而拒絕 接納辯方的證據,理由並不充分

(17.11.2000) *Simon Tam #Andrew Macrae SC

The Appellant was convicted after trial of an offence of indecent assault, contrary to s 122(1) of the Crimes Ordinance, Cap 200. The case for the prosecution was that after the Appellant had boarded a bus in Tuen Mun, he sat next to the complainant. On two occasions during the journey she felt something rubbing against her buttock. On the second occasion she came to the conclusion that the touching was deliberate and that the Appellant was responsible. She contacted the police through her mobile phone and caused the bus to stop and to await the arrival of police. The Appellant accepted that there might have been some innocent contact with the female but claimed that it was only the sort of contact that would usually occur during the course of a bus journey. He denied that he had deliberately touched the complainant and denied any indecent contact. On appeal, it was submitted, inter alia, that the magistrate’s reason for rejecting the Appellant’s evidence, namely, that he had failed to ‘acknowledge the intrusion, let alone apologise’, was unfair, unrealistic, and an insufficient reason for rejecting his evidence. In his statement of findings, the magistrate said: I considered the evidence of the Appellant. I found that it lacked credibility. On two occasions the Appellant said that he or his clothing had come into contact with PW1. However, on neither occasion had he seen fit to acknowledge the intrusion let alone apologise. This I found unworthy of belief particularly in the light of the evidence from the Appellant’s character witnesses that the Appellant was a caring and considerate person. Held : The complaint was well-founded. It was not safe to proceed on the basis that it was inherently improbable that an innocent person would apologise in such circumstances. The magistrate wrongly placed considerable weight on the lack of apology. That lack of apology was the only reason given by the magistrate for his view that the evidence of the Appellant lacked credibility. Result - Appeal allowed.

337

CCAB 2000

Statement/Admissions/Confession/Voire Dire

Statement/Admissions/Confession/Voire Dire CA 239/99 Stuart-Moore ACJHC Leong & Wong JJA

(1) KONG Siu-ming (2) HO Chi-wai (3) HO Wai-kuen

Cautioned statement/Approach to mixed statement/Exculpatory material capable of forming basis of defence 警誡口供 - 處理混合口供的方法 - 開脫罪責的資料可構成辯護的依 據 The Applicants were convicted in the District Court after trial of one charge of conspiracy to defraud.

(3.3.2000) *J R Reading SC & Sharman Lam #(1) I/P (2) I/P (3) A C Macrae SC

The Applicants were each interviewed by the police. The interview records were produced as part of the prosecution case. A3 submitted, inter alia, that the judge adopted the wrong approach in considering his interview when, in the Reasons for Verdict, she said: The exculpatory statements of the defendants are not evidence of the facts they have heard and show no more than the image or reaction of them when taxed with allegations. A3 submitted that his interview was a mixed statement consisting of incriminating admissions and exculpatory explanations and excuses. What the judge said would be correct if the statement was wholly exculpatory and selfserving but where the statement was mixed as in the present case, the judge was wrong to adopt that approach R v Aziz [1995] 2 Cr App R 478, following R v Duncan (1981) 73 Cr App R 359. The direction of Lord Lane CJ in Duncan, which was cited with approval by Lord Steyn in Aziz, was: Where a ‘mixed’ statement is under consideration by the jury in a case where the defendant has not given evidence, it seems to us that the simplest, and, therefore, the method most likely to produce a just result, is for the jury to be told that the whole statement, both the incriminating parts and the excuses or explanations, must be considered by them in deciding where the truth lies. It is, to say the least, not helpful to try to explain to the jury that the exculpatory parts of the statement are something less than evidence of the facts they state. Equally, where appropriate, as it usually will be the judge may, and should, point out that the incriminating parts are likely to be true (otherwise why say them?), whereas the excuses do not have the same weight. Nor is there any reason why, again where appropriate, the judge should not comment in relation to the exculpatory remarks, upon the election of the accused not to give evidence. Lord Steyn added that: Importantly, Lord Lane CJ added that all parts of a mixed statement ‘were evidence of the facts stated, although their weight as evidence must differ widely’ at p 365. Held : (1) The Applicants in their interview statements undoubtedly had made admissions of overt acts which the prosecution contended were pursuant to the conspiracy to run the company to swindle suppliers and from which it might be inferred that they were participants of the conspiracy. But these statements also contained their exculpatory explanations for these overt acts. The statements were therefore clearly mixed statements. Neither the judge nor counsel at the trial appeared to have appreciated that these statements were of a mixed nature. Although these were lengthy statements which required considerable time to go through, if the matter had been fully ventilated at the trial, the judge might not 338

CCAB 2000

Statement/Admissions/Confession/Voire Dire

have taken the wrong approach to treat them as wholly exculpatory and selfserving statements. That said, if the judge had in fact considered each statement as a whole, giving the incriminating parts as well as the exculpatory parts their due weight, the judge would have been entitled to have come to the verdicts she reached; (2) The judge did not entirely ignore the evidence provided in the statements. She referred to the denials and, in relation to A3’s interview, she said, to take the examples in her Reasons for Verdict: Although A3 in the interview denied that he had ever heard suppliers going up to the office of Cimax arguing that they did not want to accept post-dated cheques, it is not supported by any evidence … and later … A3 had explained in his interview that he wanted to set up another Cimax so that clients of D1’s Cimax would mistake this for the former Cimax. Those references, as well as similar references elsewhere in her Reasons for Verdict, did not show that the mixed statement of A3 had been properly considered by the judge as evidence. This was compounded by what she said after she found there was a case to answer: … it is clear that the court, faced with evidence from the prosecution which is unanswered by testimony from the accused, is not required to imagine possible defences of which there is no evidence. … and that towards the end of her Reasons for Verdict: … as the present case is one that a not incredible explanation given by the defendant in the witness-box might have created a reasonable doubt in the absence of credible explanation from the defendants, I am entitled more readily to draw inferences adverse to them. From those paragraphs, the judge had plainly not regarded the explanations given in the Applicant’s interview as evidence capable of being considered as forming the basis of a defence. This was a fundamental error in the judge’s approach to the evidence. The same wrong approach had been applied to all three of the Applicants’ interview statements. In these circumstances, there was a material misdirection by the judge and the verdicts against them had to be regarded as suspect. Result - Appeals allowed. Retrial ordered.

339

CCAB 2000

FACC 9/99 Li CJ Litton, Ching & Bokhary PJJ Sir Anthony Mason NPJ (26.6.2000) *A A Bruce SC I C McWalters & Maggie Yang #Andrew Macrae SC & Y C Yeung

Statement/Admissions/Confession/Voire Dire

SJ v (1) LAM Tat-ming (2) NG Sai-hing

Judge excluding evidence/Use of trickery to obtain confession to an earlier crime/Undercover operation involving use of questions/Approach to exercise of residual discretion to exclude/Principles applicable to exclusion re-considered 法官豁除證據 - 以哄騙手段獲得被告招認先前所犯的一宗罪行 -  底行動涉及使用提問方法 - 法官行使剩餘酌情權以豁除證據這個做 法的方針 - 重新考慮適用於豁除證據的原則 R1 and R2, both police officers, were charged with soliciting and accepting an advantage, contrary to s 4(2) of the Prevention of Bribery Ordinance, Cap 201. The ICAC used an undercover operation in which Ngau Wing, an informant, played the part of a senior triad society member and Heung Kai, an ICAC officer, played the role of his follower and minder. Ngau Wing had telephone conversations and meetings with R1 and R2, and these were tape-recorded. At the meetings, Ngau Wing and Heung Kai were wired with tape-recorders. During the conversations the Respondents incriminated themselves. The conversations in effect contained confessions. At the trial in the District Court it was submitted that while the use of undercover operatives to obtain evidence of on-going criminal offences was permissible, it was an illegitimate use of such tactics to attempt to gain evidence of confessions to past offences. It was argued that as the challenged evidence had been obtained as the direct result of trickery and deception, it was involuntary and therefore inadmissible. These submissions found favour with the judge who found that the Respondents had incriminated themselves as the result of the deception practised upon them and that, but for the trick which had been practised on them, they would in all likelihood not have incriminated themselves. In these circumstances he ruled that the evidence of the conversations of both Respondents with Ngau Wing and Heung Kai was inadmissible because the confessions which had been made were involuntary. On appeal by way of case stated, the substantial issue before the Court of Appeal was one of whether it was open to the judge to exclude the challenged evidence in the exercise of his discretion, based on unfairness, after he had found that manifest unfairness had been practised upon the Respondents by the undercover operatives such as to render the evidence involuntary. The Court of Appeal dismissed the appeal by way of case stated, holding that it was open to the judge to exclude the challenged evidence in the exercise of discretion based upon unfairness. [See Criminal Appeals Bulletin, August 1999, at pp 4-7: Ed] The Appeal Committee certified the following question of law and granted leave to the Secretary for Justice to appeal to the Court of Final Appeal: What is the scope of the discretion vested in a judge to exclude an admission made to a law enforcement officer by an accused person which is proved to be voluntarily made and where the admission was made in circumstances in which: (1) the accused was not under arrest or in custody at the time of making the admission; and (2) the accused was unaware at the time of making the admission that the person to whom it was made was a law enforcement officer and had the maker been aware of the true identity and status 340

CCAB 2000

Statement/Admissions/Confession/Voire Dire

of the officer or the agent of the officer he may not have made the admission. Held : (1) It was important to observe that by the time the undercover operation started, the alleged criminal offences had long been completed and there were suspects. At first R1 was a suspect and as the operation progressed R2 also emerged as a suspect. The purpose of the operation was to obtain confessions from them. This kind of undercover operation needed to be contrasted with undercover operations by a law enforcement agency conducted where the criminal activities were on-going and the offences had not been completed. Different considerations applied in that situation, where the purpose was to collect evidence of an on-going offence and the evidence usually concerned the commission of a crime, although, in the course of it, it was possible that evidence of past offences might emerge; (2) The judge had the overriding duty to ensure a fair trial for the accused. For that purpose he had a single discretion to exclude admissible evidence, including a voluntary confession, whenever he considered it necessary to secure a fair trial for the accused. The essential question was not whether the law enforcement agency had acted unfairly in a general sense. It was no part of the court’s function to exercise disciplinary powers over the law enforcement agencies or the prosecution as regards the way in which evidence they sought to adduce at trial was obtained by them. The court’s function was to consider whether it would be unfair to the accused to use the confession though voluntary against him at his trial. The test of unfairness was not that of a game governed by a sportsman’s code of fair play. Unfairness was to be judged against and only against what was required to secure a fair trial for the accused. It was important to observe that, in a just society, the conviction of the guilty was in the public interest, as was the acquittal of the innocent; (3) To ensure a fair trial for the accused, the court would exclude admissible evidence the reception of which would compromise the principles that no man was to be compelled to incriminate himself, and that no one could be convicted except upon the probative effect of admissible evidence; (4) The law recognised that the use of undercover operations was an essential weapon in the armoury of the law enforcement agencies. These unavoidably involved elements of subterfuge, deceit and trickery. Where those tactics resulted in a confession that was voluntary and therefore admissible, the court would have to consider whether to exclude it in its discretion in discharge of its duty to ensure a fair trial for the accused. Thus, the court might exclude the confession where the confession or the evidence of it was so unreliable that no jury properly directed might convict; for example, where the recording was so poor or in the absence of a recording, the recollection of the undercover agent was so poor that it should not be allowed to go before a jury; (5) The court might exclude the confession where the right of silence of the accused had been infringed. Where the undercover officer played a passive role and heard or overheard the confession or recorded it, there could be no basis for rejecting it. It had been freely volunteered without interrogation on the part of the officer even though he had misrepresented his identity thereby inducing the suspect to believe that he was speaking to a person who was not a person in authority; (6) The real problem arose where the undercover officer played an active role in procuring the confession from the suspect by asking questions, and it was in that area that the exercise of the residual discretion to exclude might be engaged. How it should be exercised depended on the following. If all the officer did was to draw attention to the incident which gave rise to the offence 341

CCAB 2000

Statement/Admissions/Confession/Voire Dire

charged, in the absence of any other relevant circumstances, there was no persuasive reason for excluding a confession made. The officer was doing no more than providing an opportunity for the suspect to speak. If however what the officer did amounted to interrogation, the discretion would ordinarily be exercised so as to exclude the confession. Such an interrogation would constitute a derogation of the accused’s right of silence and thus prejudice his fair trial. Whether the conversation amounted to an interrogation would have to be determined having regard to all the circumstances. Whether such an interrogation could feasibly be conducted in the context of an undercover operation without blowing the cover, would depend on the relationship between the suspect and the undercover officer in his posed capacity; (7) In the exercise of the residual discretion whether to exclude a voluntary confession, the courts might take into account as a factor the gravity of the offence. However, it would not be right to give great weight to this consideration because to do so would be to devalue the right to a fair trial to which all persons were entitled for all offences. But it was a relevant factor. The law’s approach to this matter had to take into account community interests, as well as the rights of suspects; (8) It was necessary to emphasise that the residual discretion was a judicial discretion which was to be exercised in a principled way. But the occasions where its exercise might have to be considered would involve a wide variety of facts and circumstances. Ultimately, the courts would have to apply the proper approach in a commonsense way; (9) Where a confession was obtained through an undercover operation where the crimes had already been committed and there was a suspect, the exercise of the residual discretion was likely to be engaged on the approach laid down in this judgment. That situation was to be contrasted with the undertaking of undercover operations to uncover on-going criminal activities. There, ‘the accusatory stage’ with a suspect would, generally speaking, not have been reached and different considerations would apply even where incriminating admissions were made of past offences; (10) There were a number of authorities in Hong Kong on the residual discretion. They were broadly consistent with this judgment and should henceforth be read subject to and consistent with this judgment. However, three matters arising from those authorities should be referred to: (i)

In R v Lam Yip-ying [1984] HKLR 419, the residual discretion was formulated in this way: If the judge is so satisfied [i.e. that the statement was voluntary] and the evidence suggests that it was obtained “unfairly” or that its use in evidence would be unfair, should he exercise his discretion to exclude it?

The first limb of the test of evidence having been obtained unfairly was inappropriate and should be discarded. The test should only be that in the second limb, whether its use in evidence against the accused at his trial would be unfair. And unfairness was to be judged against what was required to secure a fair trial for him. Further, in summarising the principles relating to voluntariness, Roberts CJ used the simpler word deception in substitution for fraudulent misrepresentation. This was unsatisfactory and might well have led the trial judge in this case astray. Fraudulent misrepresentation in this context related to the character of the document;

342

CCAB 2000

Statement/Admissions/Confession/Voire Dire

(ii) In R v Cheung Hon-yeung [1993] 1 HKCLR 292, Power JA, in discussing the residual discretion, treated the exclusion of statements not ruled to be voluntary as subject to the residual discretion. Power VP repeated this passage in R v Lam Ka-fai [1995] 1 HKCLR 155 at 161. It was not right to treat involuntary statements as subject to the residual discretion. They were legally inadmissible evidence and must be excluded; the court had no discretion in such a case. Power VP in these judgments referred to categories of the residual discretion. The better approach was to regard the residual discretion as a single discretion; (iii) In R v Lee Yi-choi [1985] 1 HKC 578, the applicant was convicted of murder. He was in custody having been charged. Evidence of a confession had been given by a police officer who after the applicant had been charged was introduced into the applicant’s cell under the guise of a fellow prisoner. The officer was instructed not to coax the prisoner to say anything but to listen to him and let him speak freely. The conversation was initiated by the applicant. It took about an hour after which they slept in the cell and the officer was released from the cell the following morning and he then made a note of the conversation. The Court of Appeal held that the confession was unfairly procured by a person, who was in fact a person in authority, as a result of a trick and the trial judge should have excluded it in the exercise of his discretion. The applicant was more than a suspect. He had been charged and was in custody. The Secretary for Security’s rules and directions provided that it was only in exceptional cases that questions relating to the offence should be put to the accused after he had been charged or informed that he might be prosecuted. It was unnecessary to consider in this appeal the correctness of that decision involving an accused who had been charged and was in custody. In any event, that decision might be distinguishable on the ground that the evidence of the confession was unreliable as the officer could only make a note of the conversation the next morning, although this aspect was not mentioned in the judgment. (11) The certified question of law had to be answered by reference to the common law in Hong Kong. In summary, in exercising the residual discretion to exclude a voluntary confession, the judge had to consider whether it would be unfair to the accused to use the confession against him at trial and unfairness was to be judged against what was required to secure a fair trial for him. The protection of his right of silence was a principle of a fair trial. In the context of an undercover operation, whether his right of silence had been jeopardised was relevant. Result - SJ’s appeal allowed. Acquittals set aside. Case remitted to District Court for resumption of trial. Trial judge to consider all the facts in light of this judgment in deciding the use to be made of the residual discretion.

343

CCAB 2000 MA 592/99 Gall J (8.6.2000) *Cheung Wai-sun #Henry Ma (1) J Acton-Bond (2)

Statement/Admissions/Confession/Voire Dire LEI Pei-chi (1) Yau Yick Knitting Factory Ltd. (2)

Discretion of magistrate to permit the re-opening of the prosecution case/Whether director authorised to make admission on behalf of the company/Whether admission implicated the company 裁判官行使酌情權准許控方重開案件提證 - 董事是否有權代表公司 招認 - 招認會否牽連公司 A1 was convicted of furnishing false or misleading information to an authorised officer, contrary to section 26(2) of the Import and Export Ordinance, Cap 60. A2 was convicted of giving or causing to be given false or incorrect information in a document that might be used in support of a certificate of origin application, contrary to section 7(2)(b) of the Protection of Non-Government Certificates of Origin Ordinance and of exporting prohibited articles without a licence, contrary to section 6D(1) and (3) of Cap 60 and importing prohibited articles without a licence, contrary to section 6C(1) and (2) of Cap 60. All the summonses faced by both Appellants involved 1,996 blouses, which were manufactured in A2’s knitting company in Kowloon, and were later exported to China without a licence. In China, the work of looping and stitching was done to the blouses and they were then exported back to Hong Kong without a licence. However, an application was made for the issue of a certificate of origin in respect of the blouses on the basis that all the work of manufacture had been done in A2’s factory (‘the application’). A2 was a limited company and A1 was at all relevant times a director of the A2. The application was chopped with A2 and signed by its other directors, Mr. Lau. When Customs and Excise officers attended A2’s factory, they saw A1, who produced documents to substantiate the claims made in the application. However, upon further enquiry, A1 admitted that part of the manufacturing work of the blouses was done in the Mainland and a cautioned statement was taken from him in the capacity as a director of A2, and, in that statement, he implicated A2. On appeal, it was submitted, on behalf of A1, that the magistrate wrongly exercised her discretion in permitting the prosecution to re-open its case after the defence had closed its case and delivered its final submission. On behalf of A2, it was submitted, inter alia, that the magistrate was wrong to rule A1’s statement was admissible against A2. Held : (1) The magistrate had complete discretion on whether a witness should be recalled and the court would not interfere with the exercise of the discretion unless it appeared that thereby an injustice had resulted. In the closing address of this case, the defence counsel argued that whilst the officer had given evidence of being authorised pursuant to section 4A of Cap 60, she had not given evidence that the authorisation was in writing. Out of an excess of caution, the magistrate permitted the prosecution to re-open its case and recall the witness to exhibit her warrant card. The magistrate considered that it was a technical matter to permit the recall of the witness and concluded that the exercise of the discretion would not cause any injustice: R v McKenna 40 Cr App R 65 followed; (2) A1 was a director of A2. Directors of a company were its officers. It was common sense that, on the face of it, when an investigating authority approach a company and was spoken to by a director of that company who spoke to the company’s business, that director spoke for the company : R v Lolly Queen Ltd. [1994] 2 HKCLR 51, Edwards v Brookes (Milk) Ltd. [1963] 3 All ER 62, Myers v DPP [1964] 2 All ER 881 considered; (3) It was clear from the evidence that A1, one of the three directors of A2, had an intimate working knowledge of the details of the company and that he 344

CCAB 2000

Statement/Admissions/Confession/Voire Dire had worked on the company production. The magistrate was satisfied that A1 was a full director of the company and knowledgeable as to its business. In view of the authorisation signed by Mr. Lau of A2, authorising A1 to give information to the Customs officers on behalf of A2, the magistrate was not obliged to look beyond this document. Two out of three of the directors of the company, prior to the making of the statement, were therefore in agreement that A1 should make that statement binding the company. Result : Appeal dismissed.

FACC 1/2000 Bokhary Chan & Ribeiro PJJ Sir Denys Roberts & Lord Nicholls of Birkenhead NPJJ (3.11.2000) *AA Bruce SC & Anna Lai #GJX McCoy SC RJ Pierce & Kennis Tai

(1) WONG Wai-man (2) HAU Kingyeung (3) LEE Kar-yeung

Confessions/Not permissible for prosecution to rely on differences between confessional statements of various accused to refute allegation that confession concocted/Dangers inherent in jury comparing out-of-court statements/Prejudicial effect of comparing statements outweighs probative value 供認 - 不容許控方倚賴各被告供詞中的差異,以反駁辯方指稱供認 是捏造的 - 陪審團把在庭外所作的供詞比較,這個做法有潛在危險 - 比較供詞造成偏見的影響超越其證案價值 The Appeal Committee, in granting leave to appeal, certified as a question of law of great and great importance this question: Is it permissible for the prosecution to rely on differences between the confessional statements of two or more co-accused for the purpose of refuting allegations that those confessions were concocted by the police? The Appellants were each convicted after trial of murder. A fourth accused was convicted of manslaughter. The Court of Appeal affirmed the Appellants’ convictions. As against each accused all that the prosecution had was his disputed confession to the police. The confessions were of killing the victim, whose body was never found, dismembering his body and disposing of his dismembered body. They came in the form of post-records of cautioned statements, written records of interview, video tapes of interview and video tapes of scene of the crime reconstructions. Each Appellant testified that his confession was untrue, had been concocted by the police and was made under torture. In summing-up to the jury, the judge said: There’s one other matter in relation to the statements that you can take into account. I have told you and I will be telling you again, that you cannot take into account in one defendant’s statement what he said against other defendants. You can’t look at the statements and compare them, to draw inferences of guilt against any of the defendants. However, for one purpose, you can compare the statements, and for one purpose only, and that is in assuming the question of how the statements were taken, and the question of the frame-up; because the comparisons are valid in that context and in that context alone. The judge then gave the jury an illustration of what she meant. She did so by reference to what each of the four accused had said in his confession as to the number of cartons used for disposing of the victim’s dismembered body. She pointed out that the 1st accused said that two cartons had been used while the 2nd, 3rd and 4th accused all said that three cartons had been used. She then directed the jury: So you have to consider why, if the police are doing this frameup, they allow such a major - or what might be a major difference to occur between the two statements, or amongst the three statements that they have been concocting. That direction gave rise to the certified question. 345

CCAB 2000

Statement/Admissions/Confession/Voire Dire Held : (1) The comparison exercise which the jury was invited to undertake appeared to be unprecedented. The judge did not forewarn counsel that she was going to take the novel step of inviting the jury to undertake such an exercise. Neither side had any opportunity to address the jury on what such a comparison did or did not suggest; (2) Although it was submitted by the Appellants that the comparison exercise which the judge invited the jury to undertake violated the rule against hearsay, that exercise stopped short of using out-of-court statements for the purpose of directly proving, as against anyone other than the statement-maker himself, that the facts were as asserted in them. Accordingly the rule against hearsay was not engaged; (3) Whether it was permissible to compare the out-of-court statements of coaccused did not depend on which side was seeking to make the comparison. It depended instead on the purpose for which the comparison was sought to be made. Where that purpose was to show that the out-of-court statements concerned were false, there was no question of any accused’s out-of-court statement being used against any other accused. And it was to be remembered that the prosecution too was entitled to rely on similarities between the out-ofcourt statements of co-accused for the purpose of showing that they were false: Mawaz Khan v R [1967] 1 AC 454; (4) It was in the intrinsic nature of things that differences in the statements of two or more persons did not point nearly as strongly to the absence of concoction as striking similarities in such statements pointed to concoction. Once the prosecution advanced the argument that the differences suggested that the confessions were not concocted by police, the opposite argument, that if officers were evil enough to concoct confessions, they might well be cunning enough to work in whatever differences would best hide their authorship, would automatically become available to the defence. The exercise would thus become circular; (5) An invitation to a jury to undertake the sort of comparison exercise in question would be incomplete, unfair and unsafe unless counter balanced by a cautionary admonition to consider whether the differences between the various confessional statements might have been artificially inserted in order to hide common authorship. No such admonition was given in this case. That omission made it difficult to sustain the convictions on any view as to what was the correct answer to the certified question. In answering that question it was necessary to proceed on the footing of the proper admonition being given. But then the prosecution’s difficulty was that once such an admonition was given, any probative value which the exercise might otherwise have had would be wholly negated or at least reduced to slight significance; (6) If the probative value was wholly negated, the exercise would not be probative at all and would therefore be impermissible as irrelevant. But if, on the other hand, the probative value of the exercise was not wholly negated but was nevertheless reduced to slight significance, it would be necessary to consider whether the exercise was attended by such prejudice as outweighed whatever probative value it might have. Once a jury was invited to compare the out-of-court statements of two or more co-accused for one purpose, there arose the inherent danger that they would stray into comparing those statements for other purposes. There were very strong standard directions designed to steer the jury away from treating what one accused said in his out-of-court statement about another accused as evidence against that other accused. Nothing should be said to a jury which jeopardised the effectiveness of such directions; (7) The comparison exercise which the jury was invited to undertake was at the very least fraught with danger. Even assuming that such an exercise had 346

CCAB 2000

Statement/Admissions/Confession/Voire Dire some probative value, it was attended by a measure of prejudicial effect which outweighed such probative value so as to render the exercise impermissible; (8) As it was not permissible for the prosecution to rely on differences between the confessional statements of two or more co-accused for the purpose of refuting allegations that those confessions were concocted by the police, the certified question of law was to be answered in the negative against the prosecution. Result - Appeal allowed. Retrial ordered. Per cur -

CA 53/2000 Stuart-Moore ACJHC Woo & Stock JJA (27.10.2000)

MA Yee-keung

It was for the legislature, not the judiciary, to create new exceptions to the hearsay rule: R v Blastland [1986] 1AC 41. The creation of a new exception to the hearsay rule ‘would require a general review of the hearsay rule, its history, purpose and operation’: Barron v R (1995) 185 CLR1. The Law Reform Commission appeared to be the body best suited to conduct such a general review.

Voire dire proceeding/Alternate procedure used in District Court/Defendant testifying on special issue only/Evidence given for determination of special issue not to be used for purposes of conviction/Application of proviso 案中案程序 - 區域法院使用交替程序 - 被告人就特別爭論點作證 就特別爭論點的裁決而作的證供不可用於作出定罪裁決 - 應用但書

*John Reading, SC & Wong Sze-lai

The Appellant (D2) was charged together with Chiu Shun-nin (D1) with an offence of trafficking in a dangerous drug, namely, 106.26 grammes of a mixture containing 38.67 grammes of heroin hydrochloride. As D1 had pleaded guilty, only the Appellant stood trial and, after conviction, he was sentenced to 5½ years’ imprisonment.

#Graham Harris & A Omar

On 14 July 1999, at about 8:45 p.m., on a road in Ngau Tau Kok, police officers stopped a light van with registration number GG2791, for which they had laid ambush. The Appellant was driving the van. Beside the Appellant was his wife, Madam Cheung, and sitting behind them was the D1. D1 got out of the vehicle and attempted to escape. He was stopped by the police and found to be in possession of a black plastic bag containing 6 packets of white powder, which was later found by the Government Chemist to be the heroin, which was the subject of the charge. These facts were not disputed by the Appellant before the judge. It was also agreed that on the Appellant’s fingernail clippings taken at the police station after the arrest and on the shoes he was wearing, traces of heroin were found. Further agreed was that the Appellant had borrowed from his cousin the van some days before the incident and had told the owner that it was for transporting goods. At trial, the prosecution sought to adduce certain confessions and statements made by the Appellant. Adopting the alternate procedure, the judge heard evidence from the prosecution and the Appellant himself in order to decide on the admissibility of such confessions and statements. At the end, he admitted into evidence exhibit P2, a post-record of a statement made by the Appellant upon his arrest at the scene, and P4, a record of a video-interview between PW1 and the Appellant. However, he ruled inadmissible the records of the interviews subsequent to the taking of exhibit P4, namely, prosecution exhibits P5, P7 and P9. The most important evidence shown in exhibit P2 and P4 was the statement made by the Appellant at the scene. The key issue before the judge was whether the Appellant knowingly participated in the transportation of the heroin found on D1 who had attempted to escape from the van when it was stopped by the police. 347

CCAB 2000

Statement/Admissions/Confession/Voire Dire PW1, PC 53202 and another police officer PW2, were in a police vehicle driven by a sergeant who did not give evidence. The evidence of PW1 and PW2 was that after stopping the van, PW1 approached the Appellant who was sitting in the driver’s seat of the van. PW1 told the Appellant that PW1 suspected him of possession of dangerous drugs and asked him to alight for a search. He cautioned the Appellant, searched him and found nothing suspicious. The sergeant then came and said that D1 who had been in the van had been found with white powder on him. PW1 then arrested the Appellant for possession of dangerous drugs and cautioned him. According to PW1’s testimony on the special issue of admissibility, the Appellant then said: “Ah Sir, my wife had nothing to do with this case. It was Ah Nin and I who ‘taw’ (transliteration of Cantonese) this for somebody.” The meaning of ‘taw’ in Cantonese was to drag or pull, and, as the judge put it, meant ‘moving something’. PW2’s evidence was that the Appellant at the scene said: “Ah Sir, my wife had nothing to do with the white powder. It was Ah Nin and I who ‘taw’ it for somebody”. This was also what was recorded in the post-record in exhibit P2. When PW1 subsequently repeated this to the Appellant in the first video-interview, the Appellant did not gainsay, as recorded in exhibit P4. These two exhibits were admitted into evidence after the judge heard evidence on their admissibility. Returning to the scene, upon the Appellant being further cautioned, he said nothing. The Appellant gave evidence only on the special issue of admissibility of his confessions and statements, but did not give evidence on the general issue. The judge evaluated the evidence and opined that PW1 and PW2 gave evidence in a straightforward manner. He said: Having heard PW1 and PW2, I am in no doubt that D2 (the Appellant) told PW1 that his wife had nothing to do with the matter and that D1 (the 1st Defendant) and he were transporting what had been found on D1 for someone else. Now, it is true that the words do not specifically contain an admission of knowledge that what was being transported was white powder. I use white powder there; obviously in Cantonese slang sense white powder means heroin, everybody knows that. However, on this I am entitled to look at the circumstantial evidence. There is no doubt that D1 was transporting heroin. D2 is himself a drug addict. He had to be taken for methadone whilst in police custody. D2 had himself handled heroin and got it on his shoes. He is not the kind of person who could be expected to be ignorant of drugs and drug-related matters. When stopped, his immediate reaction was to exonerate his wife, but that immediate reaction was in no way that which one would expect from an innocent man who was himself a drug addict. One would have expected an innocent man at least to say, ‘What white powder?’ Of course, he sought to exonerate himself later, but by then he had time to think matters over. It seems to me that the only inference which no reasonable person could fail to draw from what was said, and the circumstances under which it was said, is that D2 knew perfectly well that what he was transporting was white powder, i.e. heroin. I am satisfied beyond reasonable doubt that the defendant was assisting D1 to carry something for someone and that he knew that what was being carried was a dangerous drug. This amounts to trafficking. D2 is found guilty as charged and convicted accordingly. On appeal, it was submitted, inter alia, that in convicting the Appellant, the judge wrongly relied upon evidence which was before him for the purpose of determining the special issue only, and two pieces of evidence were identified in 348

CCAB 2000

Statement/Admissions/Confession/Voire Dire the Reasons for Verdict: (a)

The defendant himself said that he mentioned white powder; and

(b)

D2 is himself a drug addict. He had to be taken for methadone whilst in police custody.

Counsel referred to R v Wong Kam-ming [1980] AC 247, a murder case, where, at the start of the trial, the defence challenged the admissibility of the confessional statement on the ground that it had not been made voluntarily. The defendant gave evidence in the voire dire, in which he admitted that he had been present at the scene and involved in the attack of the deceased. The trial judge ruled the statement inadmissible but allowed the defendant’s said evidence to be proved in the main trial. On appeal from Hong Kong, the Judicial Committee of the Privy Council, at 258, held: Where the confession had been excluded, the argument against ever admitting such evidence as part of the Crown case must prevail ... the same exclusion of evidence regarding the voire dire proceedings from the main trial must be observed, regardless of whether the challenged confession be excluded or admitted. Held : (1) Wong Kam-ming was discussed by the Hong Kong Court of Appeal in R v Lai Chi-shing [1987] HKLR 422. Cons VP said: In a case of R v Wong Kam-ming (sic) [1980] AC 207 the Privy Council decided that where a confession was found upon a voire dire to be inadmissible what the defendant had said upon those proceedings should play no further part in the continued trial. That case, however, did not deal with what other witnesses, in particular those for the prosecution, had said. In Ho Yiu-fai and Others v R [1970] HKLR 415 (the) Full Court gave approval to what had by then become a common practice in the District and Magistrates Courts and is now usually called the ‘alternate procedure’. In that case the court does not stop and enter upon separate voire dire proceedings, but merely notes the objection that is taken to the proffered statement and continues with the trial. The evidence as to the admissibility of the statement is taken, as it were, in parallel with the evidence upon the general issue. At some suitable stage, usually the close of the prosecution evidence, the defendant is given a chance to give his evidence and to call his witnesses in relation to the proffered statements alone. The ruling of the court upon the admissibility of those statements will be made at least before the defendant is required to answer, if necessary, upon the general issue. In its judgment the Full Court expressly decided that although in the event the statements were not admitted the judge nevertheless would be entitled to rely upon the evidence given by the prosecution witnesses. Is the position then any different where the alternate procedure is not followed, but the single judge does enter upon a voire dire? In principle we can see no distinction. It seems to us to make no difference whether the evidence is given in what might be termed ‘parallel proceedings’ or in proceedings which are in one sense distinct and separate. The reason why there can be no repetition of the defendant’s evidence is that he is, in effect, forced to give evidence on the question of admissibility. If he were not given protection against repetition on the general issue he would be improperly deprived of his right to silence. But other witnesses have no right to silence. They do not need to be protected from 349

CCAB 2000

Statement/Admissions/Confession/Voire Dire the repetition of their evidence, nor can we see any reason why the defendant should be so protected. It would in our view be absurdly technical to exclude evidence which the judge has already heard, in some cases only a short while before. Even if viore dire proceedings are proceedings separate from the general issue, they are proceedings in the same litigation, between the same parties and before the same judge or magistrate. It was clear that, whether the alternate procedure or a voire dire had been adopted, insofar as the statement the subject matter of the proceedings had been held inadmissible, the evidence given by the defendant was inadmissible on the general issue. The dictum of the Judicial Committee cited above also extended that rule to a case where the statement had been held to be admissible. The rationale behind this was that a defendant was, when he faced the prosecution’s attempt to adduce his out of court statements as evidence implicating him, compelled to give evidence against their admissibility on the ground of involuntariness if he felt such to be the case, but his right to remain silent on the general issue had to be protected. However, the rule had no application to the prosecution’s evidence given on admissibility, which could be used and considered by the trial judge on the general issue; (2) The judge apparently used the evidence given by the Appellant in the alternate procedure, that the Appellant said he mentioned white powder at the scene, to help him decide that the discrepancy between the testimonies of PW1 and PW2 - PW2 said the Appellant used the words “white powder” while PW1 did not - was insignificant. He was wrong to have done that. However, from the evidence of PW1 and PW2 and in exhibits P2 and P4, which was admissible and had been admitted on the general issue, it was plain that the Appellant himself did mention white powder. The judge also erred in considering the evidence given by the Appellant in the alternate procedure that he was a drug addict and that he had to be taken for methadone whilst in police custody. The Appellant’s complaint on this ground was therefore substantiated. Notwithstanding, it was clear from the Reasons for Verdict, that the judge’s conviction of the Appellant was based on the following four important pieces of evidence: (a) the Appellant was driving the van containing D1 who was trying to escape but was found to be in possession of the heroin when searched; (b) the Appellant’s immediate reaction upon the police inquiry at the scene to exonerate his wife; (c) his admission in that immediate reaction that it was he and the 1st defendant who were transporting the heroin that had been found on the 1st defendant for someone else; and (d) traces of heroin were found on his fingernail clippings and shoes. Whether the Appellant himself used the words ‘white powder’ or merely ‘this case’ was, in the event, of scant comfort to him, because it was common ground that the words and circumstances of his arrest made perfectly clear that the police were challenging him about his connection with the dangerous drugs carried by D1, and it was the Appellant’s reaction which he chose to make which was not one to be expected from an innocent man, whether a drug addict or not. The evidence that the Appellant was a drug addict and having to be taken for methadone could only have been advantageous to him because that might proffer an explanation why traces of heroin were found on his fingernail clippings and shoes, though such an explanation was liable to be, and was in fact, rejected by the judge. In view of the fact that it was agreed that the traces of heroin were found on the Appellant’s fingernail clippings and shoes, and he exercised his undoubted right not to give evidence on the general issue to explain how those drugs came to be there, the erroneous admission of the Appellant’s evidence of his being a drug addict could not have operated and did 350

CCAB 2000

Statement/Admissions/Confession/Voire Dire not operate to his detriment; (3) Although it was complained that the judge had used the inadmissible evidence to conclude that the Appellant ‘is not the kind of person who could be expected to be ignorant of drugs and drug-related matters’, this was not entirely correct. What the judge said was: D2 is himself a drug addict. He had to be taken for methadone whilst in police custody. D2 had himself handled heroin and got it on his shoes. He is not the kind of person who could be expected to be ignorant of drugs and drug-related matters. The judge’s comment complained of was based more on the important pieces of evidence as enumerated in (2) above. Result -

Leave to appeal granted, but proviso applied and appeal dismissed.

香港特別行政區訴李彩彬 HKSAR v LEE Choi-bun 高等法院原訟法庭 – 高院裁判法院上訴2 0 0 0 年第9 3 9 號 * 萬德豪 J Man # 蘇惠德 W T So

高等法院原訟法庭暫委法官杜溎峰 聆訊日期︰二零零零年十月十四日及十一月十八日 宣判日期︰二零零零年十一月十八日 COURT OF THE FIRST INSTANCE OF THE HIGH COURT MAGISTRACY APPEAL NO. 939 OF 2000 TO DJ Date of Hearing: 14 October & 18 November 2000 Date of Judgment: 18 November 2000 沒有就口頭招認的自願性進行案中案程序 - 裁判官在辯方提證後方 對口頭招認作出裁斷 - 須進行案中案程序的情況 上 訴 人 被 控 一 項 在 沒 有 版 權 擁 有 人 的 許 可 下 , 為 交 易或業務 目的而管有該版權作品的侵犯版權複製品,以期作出侵犯版權的作 為 , 違 反 香 港 法 例 第528章《版權條例》第118(1)(d)及119(1)條。 經聆訊後,他被裁定罪名成立而被判處1 1 個月監禁。 據案情顯示,海關關員90142及8505到筲箕灣一間地舖執行 職務。當時舖內有許多顧客,而上訴人則站在收銀櫃後向一名男 顧各收取金錢及將一些光碟交給他。海關關員90142向上訴人表露 身份,道明來意,並問他在那處幹甚麼。上訴人回答說他是店主。 關員問他舖內的光碟有否任何版權證明。上訴人回答說他沒有版權 授權書及不能拿出來給關員查看。關員在店內搜出共3 ,9 6 5 隻光碟。 在審訊中,辯方律師提出:

主控官同我亦都頭先傾過,就係話控方呢單案件入面 需唔需要依賴個會面紀錄。基本上主控官話如果海關 關員證供可以畀得令佢滿意,佢可能未必需要呈遞 份會面紀錄。所以可唔可以同意我–如果真係控方呈 遞份會面紀錄時候,先至係… 當關員90142述及上訴人所作的口頭招認時,辯方律師沒有 提出爭議該口頭招認的自願性。控方也沒有引導關員把會面紀錄呈 堂。在盤問關員時,辯方律師向他指出上訴人沒有作出他所描述的 口頭招認。就關員的上司曾向上訴人作出恐嚇及利誘一事,該關 351

CCAB 2000

Statement/Admissions/Confession/Voire Dire 員被進一步盤問,但辯方律師卻沒有把恐嚇及利誘與關員所述及的 口頭招認扣上關係。 上 訴 人 針 對 判 罪 上 訴 時 提出多項理由,包括( i) 原審暫委裁判 官沒有循「案中案」程序裁斷上訴人的口頭招認是否自願,而把上 述招認納入證供。這剝奪了上訴人就這口頭招認的自願性作證的權 利 。 ( ii) 原 審 暫 委裁判官也沒有就這口頭招認自願性在控方結案作出 裁斷,而竟在辯方結案後在宣判時才作裁斷。所以他的程序不合符 常 規 。 上 訴 人 代 表 律 師 就  ( ii) 引 述 T ho ngj ai & Ano ther v R [ 1 9 9 7 ] HKLR 6 7 8 頁: It is sufficient to raise the issue of voluntariness for consideration by the trial judge if an allegation of ill-treatment is put to a police officer in cross-examination, even if the defendant does not give evidence in support of the allegation. This is what occurred in MacPherson where the defendant made allegations of ill-treatment in his cross-examination of the police officers, but gave no evidence in support of the allegations and made no reference to the ill-treatment in his statement from the dock. In their judgment Gibbs CJ and Wilson J stated at page 524: ‘As we have already pointed out, once a real question arises as to the voluntariness of a confession tendered in evidence, the judge has a duty to satisfy himself of the voluntariness of the confession, and usually must hold a voire dire for that purpose, even if the accused does not object to the evidence or seek a voire dire.’ 裁決︰ (1) 從謄本可見,辯方律師完全明瞭控方的案情及可舉的證 據。他明確地提出爭議會面紀錄的自願性,但卻沒有提及口頭招 認的自願性,並建議原審暫委裁判官當控方有需要呈遞會面紀錄 時,才就「案中案」作安排; (2) 當招認的自願性成為爭議點時,裁判官並非必須強制地採 取「案中案」程序的法律原。在Thongjai一案Lord Hutton沒有確 立一個一成不變的原則。R v MacPherson (1981) 47 CLR 512一 案也只認為在這情況下,法官一般應主持「案中案」程序。每件 案件有其獨特性,應否主持「案中案」視乎多項的考慮; (3) 在 HKSAR v Wong Wai Yip , 刑 事 上 訴 案 件 1995 年 第 287 號,上訴法庭法官在考慮過Thongjai一案及對Ajodha v The State (1982) AC 204一案詳加分析後指出: It would, of course, have been open to trial counsel to have conducted a voir dire with a view to obtaining a ruling on the admissibility of Exh. 8 in accordance with the usual practice. This was not, however, mandatory. In Ajodha’s case, Lord Bridge’s judgment helpfully set out four typical situations which practitioners would be most likely to encounter in approaching admissibility, where voluntariness was the issue that had to be decided. These points are set out at page 222A to E:

‘(1) The accused admits making the statement (orally or in 352

CCAB 2000

Statement/Admissions/Confession/Voire Dire writing) but raises the issue that it was not voluntary. This is a simple case where the judge must rule on admissibility and, if he admits the evidence of the statement, leave to the jury all questions as to its value and weight. (2) The accused, as in each of the instant appeals, denies authorship of written statement but claims that he signed it involuntarily. Again, for the reasons explained, the judge must rule on admissibility, and, if he admits the statement, leave all issues of facts as to the circumstances of the making and signing of the statement for the jury to consider and evaluate. (3) The evidence tendered or proposed to be tendered by the prosecution itself indicates that the circumstances in which the statement was taken could arguably lead to the conclusion that the statement was obtained by fear of prejudice or hope of advantage excited or held out by person in authority. In this case, irrespective of any challenge to the prosecution evidence by the defence, it will be for the judge to rule, assuming the prosecution evidence to be true, whether it proves the statement to have been made voluntarily. (4) On the face of the evidence tendered or proposed to be tendered by the prosecution, there is no material capable of suggesting that the statement was other than voluntary. The defence is an absolute denial of the prosecution evidence. For example, if the prosecution rely upon oral statements, the defence case is simply that the interview never took place or that the incriminating answers were never given; in the case of a written statement, the defence case is that it is a forgery. In this situation no issue as to voluntariness can arise and hence no question of admissibility falls for the judge’s decision. The issue of fact whether or not the statement was made by the accused is purely for the jury.’ In further helpful guidance, the judgment gave an indication as to how ‘practitioners might proceed to deal with these four situations, and it is (2) which is relevant for the present purposes. At page 223D, Lord Bridge said this: ‘Though the case for the defence raises an issue as to the voluntariness of a statement in accordance with the principles indicated earlier in this judgment, defending counsel may for tactical reasons prefer that the evidence bearing on that issue be heard before the jury, with a single cross-examination of the witnesses on both sides, even though this means that the jury hear the impugned statement whether admissible or not. If the defence adopts this tactic, it will be open to defending counsel to submit at the close of the evidence that, if the judge doubts the voluntariness of the statement, he should direct the jury to disregard it, or, if the statement is essential to sustain the prosecution case, direct an acquittal. Even in the absence of such a submission, if the judge himself forms the view that the voluntariness of the statement is in doubt, he should take the like action proprio motu.’ (Emphasis added). (4) 在 A jo d h a 一 案 所 舉 四 類 常 見 的 錄 取 會 面 紀 錄 情 況 中 的 第 二 類,即上訴人指紀錄為調查員所虛構,但他卻在不自願的情況下在 紀錄上簽名,Lo r d B r id ge也認為辯方律師可基於策略性理由不選用 「案中案」程序。這考慮更適用於本案沒有簽名作實的口頭招認情 況。 (5) 辯方律師選擇不採用「案中案」程序。這明顯是基於策略 性理由。他認為控方能否成功地把上訴人定罪則取決於關員90142 的可信性,所以採用「案中案」程序對上訴人沒有特別幫助。

353

CCAB 2000

Statement/Admissions/Confession/Voire Dire (6) 在 R v Jim Ch o n g - sh in g ,刑 事 上 訴 案 件 1 9 8 8 年 第 7 9 號 有 以 下判詞: We are quite satisfied that if, at the conclusion at the Crown’s evidence, or alternatively when all the evidence had concluded relating to the statement, the judge had been of the view in the absence of any submission that he was not satisfied it was voluntary he would have directed the jury to disregard it. We do not consider that in Ajodha, Lord Bridge meant that in all instances where the second alternative procedure is followed the judge must give a ruling on admissibility whether or not a submission is made, but to do so only when he considers, on the evidence, that he should. (Emphasis added). (7) 雖然原審暫委裁判官沒有選用「案中案」程序,但上訴人 已就針對該口頭招認的自願性舉證。所以原審暫委裁判官已得悉 上訴人作出口頭招認時的全部情況。上訴人已享有「案中案」程 序 的 一 切 保 障 。 鑑 於 Jim Ch o n g - sh in g 的 判 詞 , 本 席 認 為 原 審 暫 委 裁判官沒有在控方結案前就口頭招認的表面自願性作出裁斷並無 不妥。 上訴駁回。

[English digest of MA 939/2000] To DJ (18.11.2000) *Jonathan Man # W T So

LEE Choi-bun

No voire dire held on the voluntariness of the verbal admissions/Ruling on verbal admissions given at close of defence case/Circumstances under which voire dire should be held The Appellant was charged with possession for the purpose of trade or business with a view to committing any act infringing the copyright, an infringing copy of a copyright work without the licence of the copyright owner, contrary to s 118(1)(d) and s 119(1) of the Copyright Ordinance, Cap 528. He was convicted after trial and sentenced to 11 months’ of imprisonment. The facts showed that Customs officers 90142 and 8505 went to a shop in Shau Kei Wan. There were many customers inside the shop and the Appellant was standing behind the cashier counter. He was seen receiving some money from a man, and he gave some CDs to the man. Officer 90142 revealed his identity to the Appellant and asked what he was doing there. The Appellant said that he was the owner of the shop. The officer further asked for copyright proof of the CDs. The Appellant replied ‘no’ and said that he could not provide any proof for his inspection. A total of 3,965 CDs were then seized. During the trial, the defence counsel submitted: I had discussed with the prosecutor whether the record of interview would be relied on. Basically, the prosecutor said that if the C&E officers gave evidence which could make him feel satisfied, then he may not produce the record of interview. Whether you would agree with me, … if the prosecutor produced the record of interview, then I … When officer 90142 testified about the verbal admissions, the defence counsel did not challenge their involuntariness. The prosecution did not ask the officer to produce the record of the interview. Upon cross-examination, it was put to the officer that the Appellant did not make the verbal admissions as described by him. The officer was further cross-examined on the basis that the Appellant had been subjected to threats and inducements offered by his senior. The defence counsel, however, did not relate the threats and the inducements to the verbal admissions which the officer had described. 354

CCAB 2000

Statement/Admissions/Confession/Voire Dire On appeal against conviction, it was submitted, inter alia, that (i) the magistrate, having admitted as evidence what the Appellant had said above, failed to hold a voire dire in determining the issue of voluntariness of the verbal admissions. The Appellant was therefore deprived of the right to testify on the issue of voluntariness; (ii) the magistrate did not give a ruling on the voluntariness of the verbal admissions before the close of the prosecution case. It was after the close of the defence case, and when the magistrate gave the reasons for verdict, that he made a ruling as to the admissibility of the verbal admissions. The procedure adopted, so it was said, was rather unusual. In support of (ii) above, reference was made to Thongjai & Another v R [1997] HKLR 678: It is sufficient to raise the issue of voluntariness for consideration by the trial judge if an allegation of ill-treatment is put to a police officer in cross-examination, even if the defendant does not give evidence in support of the allegation. This is what occurred in MacPherson where the defendant made allegations of ill-treatment in his cross-examination of the police officers, but gave no evidence in support of the allegations and made no reference to the ill-treatment in his statement from the dock. In their judgment Gibbs CJ and Wilson J stated at page 524: ‘As we have already pointed out, once a real question arises as to the voluntariness of a confession tendered in evidence, the judge has a duty to satisfy himself of the voluntariness of the confession, and usually must hold a voire dire for that purpose, even if the accused does not object to the evidence or seek a voire dire.’ Held : (1) From what the transcript described, the defence counsel clearly understood the prosecution case and the evidence to be produced. He had clearly indicated that he had raised that he would challenge the voluntariness of the record of interview. However, he did not mention the voluntariness of the verbal admissions. He suggested to the magistrate that a voire dire might need to be arranged if the prosecution was to adduce the record of interview; (2) It was not mandatory for a magistrate to hold a voire dire whenever there was a challenge to the voluntariness of an admission. In Thongjai, Lord Hutton did not lay down a rigid principle. In R v MacPherson (1981) 47 CLR 512, it was held that a judge ‘usually must’ hold a voire dire. Each case had its uniqueness, and to decide whether or not a voire dire should be held depended on the circumstances of each case; (3) Thongjai and Ajodha v The State (1982) AC 204, were considered in HKSAR v Wong Wai-yip CA 287/1995. It was said in Wong Wai-yip that : It would, of course, have been open to trial counsel to have conducted a voir dire with a view to obtaining a ruling on the admissibility of Exh. 8 in accordance with the usual practice. This was not, however, mandatory. In Ajodha’s case, Lord Bridge’s judgment helpfully set out four typical situations which practitioners would be most likely to encounter in approaching admissibility, where voluntariness was the issue that had to be decided. These points are set out at page 222A to E: ‘(1) The accused admits making the statement (orally or in writing) but raises the issue that it was not voluntary. This is a simple case where the judge must rule on admissibility and, if he admits the evidence of the statement, leave to the jury all questions as to its value and weight. (2) The accused, as in each of the instant appeals, denies authorship of written 355

CCAB 2000

Statement/Admissions/Confession/Voire Dire statement but claims that he signed it involuntarily. Again, for the reasons explained, the judge must rule on admissibility, and, if he admits the statement, leave all issues of facts as to the circumstances of the making and signing of the statement for the jury to consider and evaluate. (3) The evidence tendered or proposed to be tendered by the prosecution itself indicates that the circumstances in which the statement was taken could arguably lead to the conclusion that the statement was obtained by fear of prejudice or hope of advantage excited or held out by person in authority. In this case, irrespective of any challenge to the prosecution evidence by the defence, it will be for the judge to rule, assuming the prosecution evidence to be true, whether it proves the statement to have been made voluntarily. (4) On the face of the evidence tendered or proposed to be tendered by the prosecution, there is no material capable of suggesting that the statement was other than voluntary. The defence is an absolute denial of the prosecution evidence. For example, if the prosecution rely upon oral statements, the defence case is simply that the interview never took place or that the incriminating answers were never given; in the case of a written statement, the defence case is that it is a forgery. In this situation no issue as to voluntariness can arise and hence no question of admissibility falls for the judge’s decision. The issue of fact whether or not the statement was made by the accused is purely for the jury.’ In further helpful guidance, the judgment gave an indication as to how ‘practitioners might proceed to deal with these four situations, and it is (2) which is relevant for the present purposes. At page 223D, Lord Bridge said this: ‘Though the case for the defence raises an issue as to the voluntariness of a statement in accordance with the principles indicated earlier in this judgment, defending counsel may for tactical reasons prefer that the evidence bearing on that issue be heard before the jury, with a single cross-examination of the witnesses on both sides, even though this means that the jury hear the impugned statement whether admissible or not. If the defence adopts this tactic, it will be open to defending counsel to submit at the close of the evidence that, if the judge doubts the voluntariness of the statement, he should direct the jury to disregard it, or, if the statement is essential to sustain the prosecution case, direct an acquittal. Even in the absence of such a submission, if the judge himself forms the view that the voluntariness of the statement is in doubt, he should take the like action proprio motu.’ (Emphasis added). (4) In situation (2) above in Ajodha i.e. where the accused denied authorship of the written statement but claimed that he signed it involuntarily, it was considered by Lord Bridge that, for strategic purposes, the defence could choose not to hold a voire dire. It could have been sought in the circumstances of this case where there was no signature to confirm whether or not the verbal admissions were in fact made; (5) It was clear that the defence counsel did not seek a voire dire for strategic purposes. The prosecution case depended on the credibility of officer 90142. Where or not a voire dire was held did not matter;

(6)

In R v Jim Chong-shing CA 79/1988, it was held that: We are quite satisfied that if, at the conclusion at the Crown’s 356

CCAB 2000

Statement/Admissions/Confession/Voire Dire evidence, or alternatively when all the evidence had concluded relating to the statement, the judge had been of the view in the absence of any submission that he was not satisfied it was voluntary he would have directed the jury to disregard it. We do not consider that in Ajodha, Lord Bridge meant that in all instances where the second alternative procedure is followed the judge must give a ruling on admissibility whether or not a submission is made, but to do so only when he considers, on the evidence, that he should. (Emphasis added). (7) Although the magistrate did not hold a voire dire, the Appellant had already adduced evidence on the issue of the voluntariness of the verbal admissions. The circumstances under which the verbal admissions were made were known to the magistrate. The Appellant had all the safeguards provided by a voire dire. In light of Jim Chong-shing, it was not improper for the magistrate to give a ruling on the admissibility of the verbal admissions after the prosecution case was closed. Result – Appeal dismissed.

Stay of Proceedings HCCC 191/99 Pang J (21.7.2000) *N Aiken SC, Ian Lloyd, G Goodman & Kelvin Lee #J Caplan QC, M Lunn SC, A King & Colin Wong (1) G Plowman SC & Keith Yeung (2)

(1) LEE Ming-tee (2) TSE Chu-fai, Ronald

Conspiracy to defraud/Effect of publicity from Inspector’s report on right to fair trial/Disclosure of material by Inspector to police an abuse/No express or implied gateway for Inspector to release information to third party/Undue delay not of itself basis for stay/Deliberate violation by authorities of accuseds’ right to fair trial 串謀詐騙 - 有關審查員報告的報道影響被告獲得公平審訊的權利 審查員向警方披露資料是濫用職權 - 並無明示或隱含的途徑可讓審 查員向第三者發放資料 - 不當延誤本身不足以構成擱置法律程序的 理據 - 當局蓄意侵犯被告獲得公平審訊的權利 The accused were jointly charged with two counts of conspiracy to defraud, contrary to common law, and four counts of publishing a false statement of account, contrary to s 21(1) of the Theft Ordinance, Cap 210. Prior to plea, the accused argued that there could not be a fair trial as they had suffered irreparable prejudice. That was on three bases. First, that the manner in which an Inspector, appointed by the Financial Secretary under the Companies Ordinance to investigate the affairs of the Allied Group Limited, (‘AGL’), of which the accused were, respectively, chairman (A1), and finance director (A2), conducted the investigation and handled the information gathered from the accused, was an abuse of his powers under the Companies Ordinance. It was further said that the inspection was conducted in such a way that it had violated the rights of the accused under the Bill of Rights Ordinance (‘BORO’) and/or their rights at common law. Second, it was submitted that the combined effect of the publicity of the companies inspection, the publication of the abridged report by the Financial Secretary, the police raid at the office of the AGL and the subsequent media coverage had been so prejudicial that a fair trial was no longer possible. In so far as those events were orchestrated by the authorities involved to bring about maximum publicity, that was an abuse of the authorities’ powers. Third, by reason of the delay in prosecuting the accused, a fair trial was no longer possible. Held : 357

CCAB 2000

Stay of Proceedings

(1) The power of the court to stay criminal proceedings should only be exercised in the most exceptional circumstances. The burden was on the accused to make out their case. To justify a stay, there must be a fundamental defect which went to the root of the case against the accused and which was of such a nature that nothing that a trial judge could do in the conduct of a trial could relieve against its unfair consequences. In an appropriate case, a court should exercise its power to suppress abuses of the judicial process and safeguard an accused from oppression or prejudice: R v Tan Soon-gin [1992] 1 HKLR 149; R v Horsferry Road Court exp Bennett [1994] 1 AC 42. The categories of abuse, by necessity, were never closed. Violation of an accused’s fundamental right in common law or a right which was recognised by the Bill of Rights Ordinance, must be a most compelling ground to stay any criminal proceedings brought against a defendant in violation of such rights; (2) The Inspector derived his powers under s 145 of the Companies Ordinance, which powers enabled him to gather information for the purpose of his investigation and to report to the Financial Secretary. Such powers did not extend to the investigation of criminal offences or gathering admissible evidence for possible future judicial proceedings. It was a well established principle of law that a statutory power was only to be used for the purpose for which it was conferred by the legislature and not for some collateral purpose: Marcel v Commissioner of Police for the Metropolis [1991] 2 WLR 1124. Since s 145 removed the privilege against self-incrimination for the purpose of the companies’ inspection, the accused were under a duty to answer the Inspector’s questions and to provide explanation and documents to him when required. If the Inspector came across any matter which tended to show that an offence had been committed, he should inform the Financial Secretary under s 146(4) of the Ordinance. There were no provisions in the Companies Ordinance under which the Inspector could pass on the information to anyone other than the Financial Secretary. Therefore, the Inspector was not entitled, as he did in the present case, to pass the documents to the police and the prosecuting authorities; (3) The prosecution submission of an express gateway did not bear scrutiny. There were no such provisions in the Ordinance. There were, however, provisions in s 152C for disclosure of materials acquired under ss 152A or 152B to a competent authority. But this did not include information and material obtained under s 145. Since there were express provisions under s 152C, the legislature must therefore have first considered such gateways but decided not to provide one for the information and material acquired compulsorily under s 145. That supported the contention of the accused that there were no express gateways through which the Inspector could pass on information under s 145 to a third party. That view was reinforced by the fact that, whereas the Hong Kong Companies Ordinance was modelled on the UK Companies Act of 1948, and, subsequently, on the 1985 Act, the UK Law was amended in 1986 to provide a gateway for the Inspector’s material acquired under the equivalent provision to s 145 (UK s 434). If such a gateway already existed expressly or by implication in the previous legislation, there would have been no need for the UK amendment in 1986; (4) There was no express provision in the Ordinance whereby the Inspector could pass compulsorily acquired information to any third party. The Inspector was, however, under a duty to inform the Financial Secretary and not anyone else under s 146(4) of matters tending to show that an offence had been committed. But that he had chosen not to do; (5) Although it was submitted that it could not have been the intention of the Ordinance, and nor was it in the public interest, that the prosecuting authority or police be denied access to the Inspector’s information and must start its own investigation from afresh, as this would produce an absurd duplication of effort and funding, the law was clear that gateways could not be implied because 358

CCAB 2000

Stay of Proceedings

statutory powers might only be used for the purpose for which they were expressly conferred. If there be a gateway, it must be created by express words in the legislation. The authorities cited did not sanction the existence of implied gateways, however important the collateral purpose might be. It was not an attractive proposition that public interest, whether in terms of financial savings or the public’s right to be informed, could justify the Inspector acting ultra vires. There was no implied gateway whether under the Companies Ordinance or in law to enable the Inspector to pass on the information to the police and the prosecuting authorities as he did in this case; (6) The right of any person charged with a criminal offence was recognised by A11(1) of the BORO. A11(2)(g), which provided that a person should not ‘be compelled to testify against himself or to confess guilt’, was partially removed under s 145(3A) of the Companies Ordinance. The right against selfincrimination could only be removed for the stated purpose of the legislation. As the stated purpose of s 145(3A) was to facilitate the Inspector’s enquiry, any suggestions that the answers could be used for any other purpose, or as in this case to facilitate police investigation, would be inconsistent with A11(2)(g) if a person was eventually charged with a criminal offence. It could not be the intention of the legislature that s 145(3A) could circumvent the provisions of A11(2 (g). Any kind of implied gateway for the Inspector to pass on the information to anyone other than to the Financial Secretary would render A11(2)(g) nugatory. It was based on the same information that the accused were subsequently arrested and charged. The activities of the Inspector were a blatant violation of the rights of the accused under A11(2)(g). On that basis, the proceedings should be stayed; (7) The publication of the Inspector’s report, the press conference and the police raid were co-ordinated moves by the authorities to achieve maximum publicity. Those involved included the Inspector, top government officials, prosecutors, and the police. It would be ironical for the prosecution to argue that the carefully orchestrated objective of the authorities to bring about maximum adverse publicity had not been attained at the time or had since faded with the passage of time. Having regard to what had been published as late as August, 1998, the accused had already been convicted by the Inspector and the subsequent media opinion. That must be prejudice in its worst form. There had been a deliberate violation of the right of the accused to a fair trial in order to justify the expenses incurred in the inspection. On a balance of probabilities, there could no longer be a fair trial of the issues; (8) Although there had been unreasonable delay in bringing the accused to trial, having regard to the time span, from investigation in 1992 until the scheduled trial in October 2000, the period of delay was only one factor to be considered in determining whether or not prejudice had been caused. The overriding consideration for the court was whether it was possible to hold a fair trial: R v Wilkinson [1996] 1 Crim App R 81, 85. There was no presumption of prejudice in a long delay, and, consequently, no shifting of the burden of disproving prejudice to the prosecution: George Tan Soon-gin v H H Judge Cameron and Another [1992] 2 HKLR 254. The burden remained with the accused. The application to stay the proceedings on the basis of undue delay failed. Result - Applications allowed. Proceedings stayed.

359

CCAB 2000

CA 210/99 Stuart-Moore ACJHC Keith & Woo JJA (23.8.2000) Andrew Bruce SC & Winston Chan # Graham Harris

Stay of Proceedings

LAW Yik-lun

Absent defence witness/Basis for permanent stay of proceedings/Prejudice must be substantial or serious/ Direction to jury on difficulties caused by lapse of time 辯方證人沒有出庭作證 - 永久擱置法律程序的理據 - 損害必須是重 大或嚴重的 - 就時間流逝所引致的困難向陪審團作出引導 The Applicant was convicted after a trial in the Court of First Instance in March 1999, of two counts of trafficking in a mixture containing salts of morphine, and one count of manufacturing salts of morphine. Prior to trial, an application was made to the trial judge to stay the prosecution permanently on the ground that a witness, Ng Wai-keung (‘Ng’), was no longer available to give evidence on behalf of the accused. The trial in prospect was a second trial, as the convictions resulting from the first trial had been overturned on appeal in May 1998. It was when the accused served a Notice of Alibi in the first trial that the name of Ng was disclosed as a witness in that Notice. The police interviewed Ng and obtained a statement dated 1 April 1995 from him. In the statement, Ng provided an alibi for the accused, and indicated that at the time police alleged he went to the flat in Kwun Tong where the drugs was manufactured, he was in fact at the Temptation Karaoke in Causeway Bay. At the time of the application for stay, there was an agreement between the parties as to the applicable law as to how a court should exercise its discretion to stay prosecutions. In Attorney General’s Reference (No. 1 of 1990) [1992] 1 QB 630, 643, Lord Lane CJ said: In principle, therefore, even where the delay can be said to be unjustifiable, the imposition of a permanent stay should be the exception rather than the rule. Still more rare should be cases where a stay can properly be imposed in the absence of any fault on the part of the complainant or prosecution. Delay due merely to the complexity of the case or contributed to by the actions of the defendant himself should never be the foundation of a stay. ….no stay should be imposed unless the defendant shows on the balance of probabilities that owing to the delay, he will suffer serious prejudice to the extent that no fair trial can be held: in other words that the continuance of the prosecution amounts to a misuse of the process of the court. In assessing whether there is likely to be prejudice and if so whether it can properly be described as serious, the following matters should be borne in mind: first, the power of the judge at common law and under the Police and Criminal Evidence Act 1984 to regulate the admissibility of evidence, secondly, the trial process itself, which should ensure that all relevant factual issues arising from delay will be placed before the jury as part of the evidence for their consideration, together with the power of the judge to give appropriate direction to the jury before they consider their verdict. That decision was followed in R v Holgate (No. 1) [1998] 3 HKC 315, where Yang CJ stressed that ‘the thrust is that the lost evidence would more likely than not tend to rebut some evidence of the Crown case or would more likely than not tend to assist the accused’ was incorrect, and re-stated the proposition, based on Attorney General’s Reference (No. 1 of 1990), that ‘the defendant has the burden of showing, on a balance of probability, that the absence of Pauline’s evidence was so prejudicial to his case that no fair trial could be held.’ Pauline was the defendant’s wife who had disappeared about two years before the trial but who otherwise could have provided confirmation of the defendant’s exculpatory explanations. The judgment continued: 360

CCAB 2000

Stay of Proceedings

We have no doubt the judge’s decision to refuse a stay was correct. It could not be shown that Pauline’s absence from the witness box prejudiced the applicant to the extent that ‘no fair trial could be held.’ This is especially so when the trial was by a judge alone. He could ensure that the applicant had a fair trial in spite of the absence, and potential importance, of the witness. In the present case, although the judge was satisfied that the prosecution had exhausted all reasonable measures to find Ng, and ensure his attendance, he refused the application. He observed: In all the circumstances of the nature of the potential evidence, and the fact that it is not direct evidence of the offences themselves, but evidence which may or may not go to the credit of the prosecution witnesses depending upon the view taken of it by the jury, I am not of the view that I ought to exercise any discretion to permanently stay this matter. The first ground of appeal was that the judge erred in refusing to stay the proceedings permanently. The error of law was said to be that the judge ‘failed to give sufficient regard to the unfairness and resulting prejudice to the Applicant caused by the non-availability of certain defence witnesses.’ The second ground of appeal asserted that the judge failed to give the jury adequate directions as to the prejudice caused to the Applicant by the lapse of time since the events in question both as a general matter and in particular in relation to the resulting non-availability of the witnesses referred to in the first ground. Held : (1) There was no error of law or principle in the judge’s refusal to exercise his discretion in favour of the defendant. He was correct to assess the quality and credibility of Ng as a witness, and his view was tantamount to saying that Ng’s evidence might not be sufficiently credible or reliable as to cast reasonable doubt on the police evidence that they found the defendant entering the flat at the time they asserted. The judge was right to take the view that the prejudice caused by the absence of Ng at the second trial was not substantial or really serious enough as to ground a permanent stay. The Applicant had failed to show the judge, on the balance of probabilities, that he would suffer serious prejudice to the extent that no fair trial could be held in consequence of the absence of Ng; (2) It was clear, from R v Dutton [1994] Crim LR 910, and R v John E [1996] 1 Cr App R 88, that a trial judge should in an appropriate case give directions to the jury on the difficulties faced or prejudice suffered, possible or real, by the defendant where an application for stay had been made by the defendant and rejected. The requirement of such a direction, however, depended on the circumstances of each case. The judge told the jury that they must consider the loss of opportunity to the defendant caused by the delay, but he refrained from intimating to the jury the value or quality of the evidence that was unavailable, and that was most proper. The judge used wording that was fair and balanced in the circumstances. Result – Application dismissed.

361

CCAB 2000 MA 663/1999 Beeson J (19.10.2000) *Cheung Waisun #Philip Wong

Stay of Proceedings LI Chi-shing

Appellant summoned for offences under the Import and Export Ordinance and Telecommunication Ordinance/Each summons based on same factual basis/Application made to stay the second summons under Telecommunication Ordinance/Whether magistrate should give reason for her decision to disallow the stay application 被告被票控《進出口條例》和《電訊條例》的罪行 - 每張傳票都是 基於相同的事實 - 上訴人就《電訊條例》控罪所發出的第二張傳票 申請擱置該控罪的法律程序 - 裁判官應否就她決定不接納擱置法律 程序的申請一事述說理由 The Appellant was summoned for an offence of attempting to export from Hong Kong 1297 sets of mobile phone otherwise than under a permit, contrary to sections 9 and 31 of the Telecommunication Ordinance, Cap 106 and s 159G of the Crimes Ordinance, Cap 200. He was convicted and fined $2000. The phones were ordered to be forfeited. The facts showed that the Appellant was charged first with attempting to export unmanifested cargo under the Import and Export Ordinance. The present summons under the Telecommunication Ordinance was issued after the trial on the first summons had ended, but before the verdict. The factual basis upon which the Appellant was prosecuted a second time under the Telecommunication Ordinance was exactly the same as that for the original summons. On appeal, it was submitted, inter alia, that (i) there was a material irregularity in the trial as the magistrate failed sufficiently, or at all, to consider an application to stay the proceedings and thereby erred in not exercising her discretion to stay and, (ii) the magistrate failed to consider the effect of s 159H of the Crimes Ordinance and wrongly concluded that she had jurisdiction to entertain an application for forfeiture of the mobile telephones. Held : (1) The transcript showed that the magistrate said ‘I now give a brief reason for why this matter should be proceeded with. I do not consider that this is a case that I should exercise my power to stay the proceeding.’ At no time did the magistrate set out the special circumstances which, in her view, made it just and convenient that the second trial should proceed. The absence of reasons made it difficult, if not impossible, for the court to deal with this appeal. The magistrate had a discretion as to whether or not to order a stay. She should have given reasons, however brief, to illuminate, or support, her ruling refusing the stay; (2) In Zhuo Cui Hao v Ting Fung Yee [1999] 3 HKC 634, it was held that “[t]he duty of a professional judge to give adequate reasons for any decision which he/she had made was a principle of paramount importance in the common law system”; (3) Where an important point like this was argued and rejected, the Appellant was entitled to know why it was rejected and what special circumstances enabled the magistrate to order the case to proceed, when it appeared there were strong grounds for ordering a stay. The Appellant needed to know whether there was any error in the reasons for the magistrate’s decision before deciding whether or not to appeal; (4) Regarding (ii) above, there were grounds which enabled the magistrate to order the forfeiture under s 36 of the Telecommunication Ordinance, irrespective of whether or not the second summons was stayed. The conditions in s 36 had been fulfilled. The forfeiture order made by the magistrate remained in force. Result - Appeal allowed. Application for stay allowed and proceedings stayed. Summons to lie on the court file. 362

CCAB 2000

Theft/Handling/Deception/False Accounting

Theft/Handling/Deception/False Accounting CA 402/99 Stuart-Moore VP Leong and Wong JJA (23.2.2000) *IC McWalters #J Marray

LI Ting-kit

Evasion of liability/Theft/Parameters of Preddy considered 逃 避 法 律 責 任 - 盜 竊 罪 - 考 慮 P red d y 一 案 的 適 用 範 圍 The original charge sheet preferred against the Appellant contained 13 charges of evasion of liability. At the end of the prosecution case, the judge granted an application by the prosecution to amend the charge sheet by adding 12 charges of theft along with the 13 charges of evasion of liability. The judge convicted the Appellant on the theft charges but acquitted him on the charges of evasion of liability. The Appellant was sentenced to concurrent sentences of 12 months’ imprisonment. The Appellant was a member of the District Board. He was aware that an extended market would be set up in Kwong Tin Estate. A group of hawkers approached the Appellant and expressed interest in having a stall. An agreement was reached under which the Appellant would make a bid to the Housing Authority to become the single market operator of the extended market and as such he could lease stalls to the hawkers who had shown interest. The Appellant required capital to satisfy the Housing Authority of his financial means. Agreements were signed under which the hawkers paid to the Appellant an acceptance fee each. The amounts paid ranged from $20,000 to $120,000 depending on the number of stalls and their size. It was stipulated in the agreements that the acceptance fee might be used as a deposit and/or rent after successful bidding, and if not successful the whole amount would be returned to the hawkers unconditionally within one month. The bid was unsuccessful. The Appellant never used the money paid to him by the hawkers for the purpose it was agreed. He sought to delay to refund the money to the hawkers as he was bound so to do under the agreements and issued personal cheques to the hawkers which were dishonoured. A sample of the charge was as follows: Li Ting-kit, between the 31st day of January 1996 and the 15th day of October 1996, in Hong Kong stole a chose in action, namely, a debt of $50,000.00 Hong Kong currency owed by Hua Chiao Commercial Bank to Lai Suk-wan, wife of Ng Ting, in the form of cheque number 182435 and dated the 31st day of January 1996 in the said Lai Suk-wan’s Account Number 064-782-00830714. The grounds of appeal were (1) relying on R v Preddy [1996] 3 WLR 255, that there was no property capable of being stolen because the choses in action had been extinguished by the time the judge found the thefts occurred or, alternatively, the Appellant should not have been convicted of the theft charges with imperfect particulars, (2) the acquittal of the evasion charges was inconsistent with the finding of dishonesty on the theft charges. The Respondent submitted that the case against the Appellant was his misapplication of monies which he had lawfully obtained and which he knew he was under an obligation to retain and deal with in a certain way. The chose in action was the property of the hawker by virtue of s 6(3) of the Theft Ordinance, Chapter 210 which provided: Where a person receives property from or on account of another, and is under an obligation to the other to retain and deal with that property or its proceeds in a particular way, the property or its proceeds shall be regarded (as against him) as belonging to the other.

363

CCAB 2000

Theft/Handling/Deception/False Accounting

Held : (1) In Preddy, the charges against the appellants were ones of obtaining or attempting to obtain property by deception. They were not charged with theft. Section 6(3) clearly had no application to the offence of obtaining property by deception because the element of a legal obligation to deal with the property or its proceeds in a particular way was not present. The present appeal did not fall within the parameters of Preddy and should be decided under s 6(3); R v Arnold [1997] 4 All ER 1 and R v Klineberg [1999] 1 Cr App R 427 followed; (2) There was no inconsistency between the judge’s finding on the two sets of charges. The judge had found the charges proved. On the evidence, the theft charges were clearly proved to the requisite standard; (3) Although the judge had not dealt with s 6(3) in his Reasons for Verdict, there was an abundant basis to bring it into play and he was bound to have found that it applied if he had considered it. Result - Appeal dismissed upon application of proviso.

CA 171/99 Stuart-Moore & Mayo VPP Keith JA (11.4.2000) *M C Blanchflower #I/P

NG Pak-yik

Framing of charge/Amendment of charges/Substitution of charges in the Court of Appeal 擬定控罪 - 修訂控罪 - 在上訴法庭以別的控罪取代 The Applicant (D2) organized a fraudulent scheme to obtain money through the unauthorized use of credit cards issued by the Standard Chartered Bank. He enlisted the assistance of D1 and D3 and of two other people. D3 worked for the bank. He would find out the particulars of some of the bank’s existing credit card holders who had cards which were about to expire. On the basis of that information, the bank would be informed that the credit card holders had changed their mailing addresses. The bank would then mail the new credit cards to the false mailing addresses which had been provided to the bank, and the new credit cards and accompanying mail would be retrieved from those addresses. The accounts of the credit card holders whose new credit cards had been retrieved from the false mailing addresses would be accessed by means of those new credit cards. Once accessed, either money would be withdrawn from cash dispensers or Jockey Club cash vouchers would be obtained from Jockey Club outlets which could then be redeemed in cash. The Applicant was originally charged with, inter alia, conspiring to steal (a) money (belonging to named account holders) and (b) cash vouchers (belonging to the Jockey Club). At the conclusion of the prosecution’s case, the prosecution applied to amend those charges. They were amended to conspiracies to steal money belonging to the bank. The Applicant was convicted and appealed on the ground that it was unfair for those amendments to have been made. Held : (1) There was no question of the Applicant being prejudiced in any way by the amendments. He was merely being deprived of the windfall of which he would unjustifiably have been a beneficiary if he had eventually had to be acquitted on the basis that the wrong charges had been preferred against him. The amendments did not require any new evidence or affect in any way the thrust of the Applicant’s defence;

364

CCAB 2000

Theft/Handling/Deception/False Accounting

(2) The amended charges on which the Applicant was convicted were still not properly drawn. The cash vouchers were not money and they did not belong to the bank. They were choses in action which were redeemable in cash and they had belonged to the Jockey Club. The charges would have been properly drawn if instead of referring to money belonging to the bank, they had simply referred to ‘property belonging to another’. Such a formulation could also have covered the cash to be obtained from the cash dispensers; (3) The combined effect of s 83A of the Criminal Procedure Ordinance, Chapter 221 and s 83 of the District Court Ordinance, Chapter 336 was to enable the Court of Appeal to substitute for the convictions on the amended charges convictions for other offences if on the findings of fact made by the judge the Applicant was guilty of those offences. Result - Save for substitution of reworded charges, application dismissed.

CA 491/99 Mayo VP Wong & Keith JJA (21.6.2000) *J Reading, SC & I Tam #Gerard McCoy, SC, Phillip Ross & Cherry Hui

GOH Swee-yan, Angelina

Drawing of inference from circumstantial evidence/Duty of court to consider all possible defences/Drawing of conclusion of dishonesty/Effect of regime created by s 20, Cap 8/Single count charging theft of separate sums of money on same occasion not duplicitous/Nature of chose in action/ Restitutionary claim arising 憑環境證據作出推論 - 法官須考慮所有具可能性的辯護理由 - 得出 不忠實行為的結論 - 第8章第20條建立的制度的影響 - 單一控罪控 告同一次盜竊不同金額的金錢並非包含多項罪行 - 據法權產的性質 - 引發復還申索 The Applicant was charged with three offences under the Theft Ordinance, Cap 210, namely, procuring the making of an entry in a record of a bank by deception, contrary to section 18D (the 1st charge), theft, contrary to section 9 (the 2nd charge) and evading a liability by deception, contrary to section of 18B (the 3rd charge). She was convicted of the first and second charges and sentenced to two years’ imprisonment. The background was that the Applicant and Amy Ng (‘Amy’) were close friends. For a while they lived with each other and shared the same bed, though Amy said that they were never lovers. Eventually, they fell out, and Amy went to the police complaining that the Applicant had taken money from her. The facts and grounds of appeal relating to the first charge The business of which Amy was the proprietor had an account at the Hang Seng Bank (‘the Bank’). On 14 January 1997, the Applicant telephoned the manager (‘Wong’) of the branch at which the account was held. She told him that Amy had gone to China on business, and that Amy had asked her to call him and request that a sum of $1.07 m be transferred to a particular account. She gave Wong the number of the account to which the money should be transferred and the name of the account holder. The Applicant added that Amy would lose the business deal if the money was not transferred. Believing what he had been told, Wong gave instructions for the money to be transferred. In fact, Amy had not told the Applicant to do anything of the kind. When Amy discovered what the Applicant had done and talked to her about it, the Applicant told her that she had used the money to repay gambling debts incurred in Macau. Upon the request of the Applicant, Amy agreed to treat the transfer of the money as a loan and signed the necessary documents retrospectively authorising the transfer. On appeal, it was submitted that the judge did not properly consider whether, in procuring the making of such an entry by deception, the Applicant 365

CCAB 2000

Theft/Handling/Deception/False Accounting

had acted dishonestly. Further, it was contended that the prosecution had not proved by admissible evidence that the entry in the Bank’s records had indeed been made. The facts and grounds of appeal relating to the second charge The Applicant was a director of Styland Holding Ltd (‘Styland’). She suggested to Kenneth Cheung (‘Mr Cheung’), another director of Styland, that she lend him $3 m to enable Styland to negotiate a business deal which would generate profits in which the Applicant could share. It was agreed that the loan should be repaid in three months’ time, and that the charge for such loan should be $300,000. As a result, the Applicant drew a cheque of 4 December 1996 in favour of Mr Cheung and, in return, Mr Cheung drew two cheques – one for $3 m and one for $300,000 – on his account in favour of the Applicant. Those two cheques were post-dated to 4 March 1997. However, the Applicant was not financing the loan herself but got Amy to put up the $3 m. Accordingly, Amy transferred the sum to the Applicant’s account, and the Applicant drew two cheques – one for $3 m and one for $300,000 – on her account in favour of Amy. The cheques were also post-dated to 4 March 1997. When the loan to Mr Cheung was about to mature, the Applicant told Amy that Mr Cheung wanted the loan to be extended for a further three months. Amy agreed but imposed two conditions, namely, interest should be increased to $396,000 and that the cheques had to be provided by Mr Cheung. Presumably the Applicant told Amy that Mr Cheung agreed and Amy was then provided three cheques for those amounts post-dated to 4 June 1997. The cheques were drawn on the account of a company ‘owned’ by the Applicant and bore signatures which were illegible save for the first two letters of each name, which were the initials of Mr Cheung. In fact, Mr Cheung knew nothing of what the Applicant had told Amy on his behalf. He had not wanted to extend the loan and his two cheques were presented for payment and the proceeds of the cheques had gone into the Applicant’s account. He had nothing to do with the three cheques drawn on the British Virgin Islands’ company’s account. What actually the Applicant had done was to trick Amy into not expecting the return of her money for another three months, and to have access to this $3.3 m in the meantime. On appeal, it was submitted that the property stolen was not cash but choses in action, and it was not established that it was property belonging to another. Held : (1) There was no direct evidence that the Applicant had thought that Amy would have consented to her using Amy’s money to settle a pressing gambling debt. She elected not to give evidence, as was her right, and when she was interviewed by the police, she had said that Amy had expressly authorised her to request the transfer of the money - an assertion which was rejected. However, there was a considerable body of circumstantial evidence from which it might have been open to the judge to infer that the Applicant had thought that. It was here that the relationship between Amy and the Applicant was important; (2) A judge trying a case with a jury was obliged to leave for the jury’s consideration all possible defences which were realistically raised by the evidence, even if a particular line of defence was not being relied upon. Similarly, a judge sitting alone should consider all possible defences which were realistically raised by the evidence, even if no reliance was being placed on a particular line of defence; 366

CCAB 2000

Theft/Handling/Deception/False Accounting

(3) The Applicant untruthfully told Wong that Amy had expressly authorised her to request the transfer of the funds. The question which the judge had to address was whether that was dishonest. Even if the Applicant thought that Amy would not have minded about the transfer of the funds, she nevertheless deceived Wong into thinking that Amy had expressly authroised the transfer. She had resorted to the lie because she would have thought that without making Wong believe that she had Amy’s express authority, there would have been little chance of Wong acceding to her request. The judge was entitled to draw the conclusion that the Applicant was dishonest; (4) The law recognised that the number of transactions which a bank daily carried out, and the use of computers to record them, made it quite impossible for first-hand evidence to be given about them. That was why section 20 of the Evidence Ordinance created a simple regime for the admission of evidence relating to such transactions which did not depend on either the recollection or the evidence of the employee of the bank who carried them out. But in order to minimise the risk of error, safeguards were built into that regime. They were not onerous, and compliance with them was not difficult. As the conditions were not complied with, the exhibits of a transaction advice on the Bank’s standard form which contained a computerised entry purporting to show the deposit of $1.07 m into an account and a customer’s computerised statement of account were not admissible. However, an inference could be drawn from the bank account withdrawal form which had been completed by a member of the staff of the Bank that the deposit entry referred to in the first charge had been proved; [Keith JA, dissenting: It could never be appropriate to apply the proviso to section 83(1) of Cap 221 if the admissible evidence relied on by the prosecution was not capable of proving one ingredient of the offence. Otherwise, a conviction would be upheld when proof of an essential element of the offence had not been established. If there had been other evidence from which the deposit entry could have been inferred, the proviso should be applied because that material irregularity had not resulted in any injustice to the Applicant. Since there was none, the question of the application of the proviso could not arise.] (5) If a defendant stole two sums of money on the same occasion in exactly the same circumstances, a charge was not duplicitous if it included in the one charge the aggregate of those two sums; (6) What the Applicant had stolen was not $3.3 m in cash belonging to Amy, but the proceeds of the two cheques which Mr Cheung had drawn in her favour. Once the cheques had been paid into the Applicant’s account, the cheques were converted into an increased credit balance in her account. That increased credit balance represented a debt which the Bank owed to the Applicant, which it had undertaken to repay on demand. That debt was a chose in action, which was the legal expression used to describe all personal rights which could only be claimed or enforced by action. Such form of property was of a kind which could not have belonged to another: R v Preddy [1996] AC 815, R v Klineberg [1999] 1 Cr App R 427; R v Graham [1997] 1 Cr App R 302 and HKSAR v Li Ting-kit CA 402/99 considered; (7) Section 6(3) of the Theft Ordinance could not assist the prosecution because the person from whom the defendant had to have received the property had to have been the same person to whom an obligation was owed to retain and deal with the property in a particular way. The two cheques were made payable to the Applicant by Mr Cheung. He knew nothing of the onward transmission to Amy of the money which the cheques represented. The Applicant was not under an obligation to Mr Cheung to retain and deal with the proceeds of the cheques in any particular way. In other words, the prosecution failed to prove that the Applicant had stolen anything which was capable of belonging to Amy; 367

CCAB 2000

Theft/Handling/Deception/False Accounting

(8) However, the Applicant was under a duty to Amy as her principal to account for the sums. A failure on the part of the Applicant to do so gave Amy a restitutionary claim against the Applicant: Para. 6-058, art. 53, Bowstead & Reynolds on Agency 16th ed. considered. Result

MA 912/99 Chu DJ

CHAN Hoi-ying

- Application for leave to appeal allowed. The appeal against conviction on the first charge was by a majority dismissed. The appeal on the second charge was allowed to the extent that the conviction for theft was substituted for a conviction under section 18B of Cap 210 of evasion of a liability by deception.

Magistrate’s findings/No direct evidence of offence /Need to identify primary facts and inferences drawn/Specific findings to be indicated 裁判官的裁斷 - 被指控的罪行欠缺直接證據 - 有需要指出基礎事實 及所作的推論 - 須述明具體的裁斷

(1.9.2000) The Appellant was convicted after trial of an offence of theft. *Vivien Chan #Eric Kwok

The particulars of offence alleged that the Appellant on 21 April 1999, at a Wellcome Supermarket in Yuen Long, stole a total of 10 items belonging to Wellcome Company Limited. On appeal, it was submitted, inter alia, that the magistrate failed to make a specific finding that the 10 items were property belonging to Wellcome, and that he did not appear to have properly drawn an inference that the 10 items were the property of Wellcome. Held : (1) There was no direct evidence that the Appellant took the 10 items from the goods racks inside Wellcome, and there was no direct evidence that they were the goods of Wellcome. Since it was a case of drawing inferences from primary facts, it was prudent and necessary for the magistrate to indicate and set out the primary facts as proved and then to state the inferences drawn from the proven facts. This had not been done in either the oral reasons or the statement of findings. The omission was not necessarily fatal where the facts were straightforward and the inferences to be drawn were apparent. However, it was the Appellant’s case that she purchased the ten items elsewhere and the burden was on the prosecution to prove beyond reasonable doubt the issues of appropriation and of property belonging to another. That required the magistrate to evaluate the evidence and to make specific findings on these issues; (2) It was not sufficient for the magistrate to state in his findings that in reliance on the evidence of the prosecution witnesses he found the prosecution had proved each ingredient of the offence of theft beyond reasonable doubt. That was insufficient because the credibility of the prosecution witnesses, including the supermarket cashier and manager, was not conclusive of the matter, particularly so since the manager testified under cross-examination that goods sold in other shops could still be registered on Wellcome cash registers when being scanned by the scanners at the supermarket cashier counters. This was not explored further, and it was not clear how goods sold by the supermarket could be distinguished from goods sold elsewhere. There was a reasonable doubt or, at least, a lurking doubt as to whether all or any of the 10 items were taken from Wellcome and whether they were the property of Wellcome; (3) The failure of the magistrate to make a specific finding on the issues of appropriation and property belonging to another, and his omission to set out the 368

CCAB 2000

Theft/Handling/Deception/False Accounting

process whereby he came to his findings, rendered the conviction unsafe and unsatisfactory. Result - Appeal allowed.

CA 91/1999 Stuart-Moore VP Leong JA & Keith JA

LAM Choi-man

False accounting/Whether invoices used as supporting documents for the issue of letters of credit were documents for accounting purpose/Inconsistent verdict based on wrongful acquittal/Extent of duty of District Judge to provide reasons 偽造帳目 - 為獲簽發信用證而用作證明文件的發票是否為會計用途 而製備的文件 - 錯誤裁定無罪因而造成不一致的裁決 - 區域法院法 官在列述理由方面所擔負的責任

(1.8.2000) *John Reading SC & Alex Lee #Daniel Marash SC

The Applicant was convicted of four charges of false accounting. All four charges involved LIANG Jin-wen, one of the co-conspirators who had absconded. LIANG was, at the material time, the general manager of CAC Brake Co., Ltd. (CAC). He had applied for the issue from the Guongdong Development Bank (GDB) of four letters of credit, without authority from CAC, with supporting documents which included the four invoices particularised in the four charges. These four invoices purported to show that there had been sales of goods to CAC but they were false in that there were in fact no such sales and no goods had been delivered. These letters of credits were discounted and the proceeds were distributed, some of which went to the Applicant’s account. He was acquitted of two charges of conspiracy to defraud. One of the grounds of the appeal was that the judge erred in convicting the Applicant in that there was insufficient evidence that the Applicant falsified any commercial invoice made or required for an accounting purpose of the issuing bank, the GDB or CAC. He submitted that the invoices were generated solely to deceive GDB and other banks and would not form part of the records of the CAC since it was not the prosecution case that the documents would form part of those records. There was also no evidence that the Applicant appreciated that GDB required an invoice for negotiating a letter of credit nor an invoice was always required for that purpose. The second ground was that the acquittals of the conspiracy charges were inconsistent with the convictions on the false accounting charges. Held : (1) There was ample evidence in each of the four charges that the Applicant had participated or concurred in the making of the four invoices which were false and he knew they were false because there were no sales and no goods had been delivered under the purported contracts of sales. The Applicant knew and intended that these invoices were to be used as supporting documents for the issue of the relevant letters of credit by GDB. These invoices were required to enable the bank to know the underlying transactions for the letters of credit. Without these supporting invoices, no bank would have issued them; (2) The invoices were required for accounting purpose even if they were not to be included as part of the accounts. The phrase ‘document made or required for any accounting purpose’ in s 19 of the Theft Ordinance, Chapter 210, should not be limited to accounting records or financial statements. It should include the documents that would be required for the preparation of accounting records or financial statements and an invoice would fall within that category: AttorneyGeneral’s Reference (No. 1 of 1980) [1981] 1 W L R 36 followed; (3) The purpose or motive for the creation of the false invoices was not relevant, and as long as the invoices were falsified by the Applicant as a party to the falsification for an accounting purpose, it was not necessary to prove that the 369

CCAB 2000

Theft/Handling/Deception/False Accounting

Applicant knew that they were for the accounting purpose of the GDB: R v Graham [1997] 1 Cr App R 302 considered; (4) The conspiracy offences and the false accounting offences were distinct in their elements that were required to be proved. It might well have been that the evidence of the prosecution witnesses supported a verdict of guilty of the conspiracy charges by the Applicant, and that acquitting him purely on the ground of lack of evidence from GDB was wrong. But a wrong acquittal was not a ground to disturb a correctly founded conviction on a different offence. There was ample evidence to support the conclusion of the judge; (5) The trial judge was not required when setting out his reasons, as required by s 80 of the District Court Ordinance, Chapter 336, to state precisely what evidence he accepted or rejected: R v Sheik Abdul Rahman Bux [1989] 1 HKLR 1 followed. Result - Application dismissed.

CA 577/1999 Wong Keith & Woo JJA (12.10.2000)

*John Reading SC & C Fung #Daniel Marash SC & J Vaughan (1) Gary Plowman SC & Po Wing-kay (2) L Lok SC & E Shum (3)

(1) CHIU Yu-man (2) WONG Siu-leung (3) YIP Wai

Conspiracy to procure execution of valuable securities by deception/Whether irrevocable letter of credit a valuable security/Status of letter of credit considered 串謀以欺騙手段促致有價產權書的簽立 - 不可撤銷信用證是否有價 產權書 - 考慮信用證的性質 The Appellants were convicted of offences of conspiracy to procure the execution of valuable securities by deception, contrary to common law and s 22(2) of the Theft Ordinance, Chapter 210. All the charges were in similar terms, with the exception of the names of the parties and the dates, which alleged that the named defendants, with a view to gain for themselves or another, conspired together to dishonestly procure from the Bank of America (Asia) Ltd. (BA), the execution of irrevocable documentary letters of credit by deception. The prosecution case was that in each case the relevant defendants conspired to obtain the issue of letters of credit by BA on the basis of nonexistent or bogus transactions. By such means credit for 90 days was obtained from BA. It was not in dispute that all letters of credit or trust receipt loans were settled and the bank suffered no loss. The issue of law raised at the trial and on appeal was whether a letter of credit was a valuable security for the purposes of s 22 of the Theft Ordinance, Chapter 210.

S 22(2) of the Theft Ordinance, Chapter 210 provided: Any person who dishonestly, with a view to gain for himself or another or with intent to cause loss to another, by any deception (whether or not such deception was the sole or main inducement) procures the execution of a valuable security shall be guilty of an offence and shall be liable on conviction upon indictment to imprisonment for 10 years. S 22(4) defined ‘valuable security’ as meaning any document creating, transferring, surrendering, or releasing any right to, in or over property, or authorizing the payment of money or delivery of any property, or evidencing the creation, transfer, surrender or release of any such right, or the payment of money or delivery of any property, or the satisfaction of any obligation. The Appellants argued, firstly, that no right was created, transferred, surrendered or released by the letters of credit and, secondly, if a right had been created, transferred, surrendered or released by the letters of credit, that right 370

CCAB 2000

Theft/Handling/Deception/False Accounting

was not a right ‘to, in or over property’. The trial judge had held, following R v Benstead & Taylor (1982) 75 Cr App R 276, that a letter of credit was a valuable security in terms of s 22. The Appellants argued alternatively that if an irrevocable letter of credit could be a valuable security within the meaning of s 22(2) of the Theft Ordinance, Chapter 210, the prosecution failed to prove that Letters of Credit 7071/95 and 7072/95 were executed as valuable securities. The other grounds of appeal related to the judge’s findings of fact. Held : (1) The argument put the cart before the horse because without the letter of credit the draft demanding payment would never have come into existence. The draft demanding payment had no independent status or existence and it was only an off-spring born out of the letter of credit. It was true that the bank’s obligation to pay did not crystallise until the draft demanding payment had been presented and accepted. But this did not mean that the obligation to pay was not created by the letter of credit. The draft demanding payment was only one of the conditions subsequent to be fulfilled before the bank paid out the money; (2) A letter of credit was a special kind of contract which imposed an absolute obligation on the bank to pay the seller of goods. No bank would grant a letter of credit to an applicant without, prior to the issue of the letter of credit, the applicant having paid the full amount or having provided sufficient security to the bank in satisfaction of the amount to be drawn on the letter of credit. This was accepted commercial and banking practice. In this case, a credit line or overdraft facilities would have been granted to the applicant companies before the letters of credit were issued. This satisfied the requirement that the property was already in existence at the time of issue of the letters of credit. An ordinary contract in writing was subject to the law governing ordinary contracts and the uncertainties and litigation that such a contract might bring. A letter of credit was no less a valuable security than a cheque. Although breaches of contract or bounced cheques were often heard about, there was seldom a case of breach of a letter of credit between a bank and the seller or still less a bounced letter of credit. There were clearly distinctions between an ordinary contract in writing and a letter of credit; (3)

Benstead was correctly decided and should be followed;

(4) The alternative argument was founded on the premise that even if the letters of credit were valuable securities, the prosecution had failed to prove that they were executed as valuable securities. This was a fallacious argument. If a person was defrauded or deceived into issuing a cheque, the fact that the cheque had not reached the hands of the fraudster or had not been presented to the bank for payment did not alter the nature of the cheque as a valuable security. It was plain from the evidence, and this was never in dispute, that BA made payments to the purported sellers pursuant to the letters of credit. The judge found that the underlying transactions were bogus and on the basis of that finding he was entitled to draw inferences from that finding that the letters of credit were duly and properly executed as they would have been but for the fact of the deceit practised by the Appellants; (5) Deception, like intention, was hardly an element of the offence to be proved by direct evidence. It was more often than not proved by indirect evidence or inferences to be drawn from primary facts. This was clearly the case here. There was an abundance of evidence from which the judge could draw the inference that the transactions underlying the applications for the letters of credit were non-existent and false. Once this was established, it followed, like day followed night, that the participants in the conspiracies were acting dishonestly to procure the execution of the letters of credit from BA by deception: R v Kassim [1991] 3 All E R 713, R v Kwan Ping-bong [1979] 371

CCAB 2000

Theft/Handling/Deception/False Accounting

HKLR 1 considered. There was no requirement for a District Judge to deal with every point or all the evidence in his reasons for verdict: HKSAR v Lam Chunsun [1998] 2 HKC 214, R v Sheik Abdul Rahman Bux and Others [1989] 1 HKLR 1 followed; Per Keith J A: (6) The obligation to pay was created by the letter of credit, even though the performance of that obligation was not triggered until the presentation of the draft. In other words, the obligations created by the letters of credit in the present case, to the extent that they related to the payment by the Bank of the sums due under them, were subject to the fulfilment of conditions subsequent, one of which was the presentation of drafts demanding payment. Accordingly, it was the letters of credit in the present case, rather than the drafts, which created in the beneficiaries the right to payment from the Bank; (7) In the present case, the rights of the beneficiaries to enforce payment from the Bank arose from the rights created by the letters of credit. The primary right was the right to payment. The secondary right was the right to enforce that primary right by action. The letters of credit in this case came within the first limb of the definition of ‘valuable security’. Result - Leave granted. Appeal dismissed. Obiter -It was a source of bewilderment that it was necessary to instruct two sets of senior and junior counsel to argue identical grounds when this could have been done by one set of counsel.

Triad CA 249/99 Stuart-Moore & Mayo VPP Woo JA (11.4.2000) *Peter Chapman, Jackson Poon & H Tse #P Cogrove (Ds1, 2, 4, 6, 8, 15 & 16) M Delaney (D3 & D9) W Allan (D5, D19 & D20) G J X McCoy SC & V Rajwani (D17) D Boyton (D18)

WONG Fung-tak and 15 others

Witness permitted to refresh memory/Reading from diary when testifying in witness box/Acceptable practice in cases of complexity to avoid mistake and confusion/Acting as member of triad society/Procuring others to act as members of a triad society sufficient basis for conviction under s 20(2), Cap 151 證人獲准重溫一些資料以助記憶 - 在證人台作證時照記事簿內容讀 出 - 在複雜案件中為避免錯誤及混淆可以接受這種做法 - 以三合會 社團成員的身分行事 - 促致他人以三合會社團成員的身分行事,是 被裁定第151章《社團條例》第20(2)條的罪行罪名成立的充分理據 The Applicants were convicted in the District Court after a 12-day trial of offences related to their connections with the Wo Hop To triad society. The charge sheet contained 29 charges against 23 defendants. For a period of about 6 months, the Wo Hop To triad society was infiltrated by a police agent, PW1. On appeal, it was submitted, inter alia, that the judge erred when he permitted PW1 to read his evidence from a diary prepared by him during the course of the investigation. On behalf of D17, who allegedly summoned triad troops to execute a revenge attack, it was submitted that the judge had wrongly interpreted the offence of acting as a member of a triad society to include his conduct in procuring other people to act as triad members. Held : 372

CCAB 2000

Triad

(1) The judge had a full grasp of the considerations to be borne in mind when approaching the commonly encountered situation, in cases of factual complexity, where witnesses requested to refer to and even to read from notes made at or shortly after the time of the incident to which they referred. Time and again, the courts had emphasised the importance of not turning the exercise of obtaining testimony given by witnesses from the witness box into a test of memory. The jury, or a judge sitting alone, were primarily concerned with the truthfulness and accuracy of witnesses and it bordered on the absurd to suggest that when a record was available from which the witness could refresh his memory as to complicated events he could not also be permitted to read directly and extensively from it: R v Da Silva (1990) 90 Cr App R 233, distinguished; (2) Section 20(2), Cap 151, did not exclude the application of the common law doctrine of secondary participation. D17 was fully aware of the nature of the case against him as a procurer of the troops and he had ample opportunity to defend himself against that precise allegation; (3) It was quite unnecessary to prove actual membership of a triad society so far an offence of acting as a member of a triad society were concerned. The common law doctrine related to secondary participation had been incorporated into s 89 of the Criminal Procedure Ordinance, which provided that: Any person who aids, abets, counsels or procures the commission by another person of any offence should be guilty of the like offence. It was open to the judge to convict D17 on the basis that he procured others to commit the offence of acting as triad members. He was, however, by acting as he did, himself acting as a member of a triad society by procuring others to so act. On either basis, however, the conviction was amply justified. Result – Applications dismissed.

373

CCAB 2000

Verdict

Verdict MA 838/99 Nguyen J (11.4.2000) *Cheung Waisun #J Hemmings & J McGowan

LO Chun-nam

Reasons for Verdict/Whether reasons sufficient/Basis for interference on appeal 裁決理由 - 理由是否充分 - 上訴時對裁決作出干預的理據 The Appellant was convicted after trial of an offence of theft and sentenced to six months’ imprisonment. On appeal, it was submitted, inter alia, that the reasons given by the magistrate for accepting the evidence of the prosecution witnesses and for rejecting the testimony of the Appellant were wholly inadequate. In particular, it was said that demeanour was of little assistance in evaluating the truthfulness, or otherwise, of a witness, especially over a short period of time and particularly when assessed by a magistrate, as opposed to a jury, who of necessity spent most of his time keeping a record of proceedings rather than observing the witnesses. Held : (1) The court could do no better than to quote from the judgment in R v Sheik Abdul Rahman Bux & Others [1989] 1 HKLR 1, where the Court of Appeal quoted with approval that said by Blair-Kerr J in Re R E Low [1961] HKLR 13: Similarly, if he [a district judge] chooses to review the evidence at length and it is clear from his statement that he has substantially misapprehended or misunderstood the true nature of that evidence, or any important part of it, it may well be that it would be open to an appellant to attack his conclusions on the facts before this Court. But it must be remembered that the district judge is himself the jury. He has heard the whole of the evidence and he is not duty bound to set down precisely what he accepts, what he rejects and what weight he attaches to every piece of evidence, or the arguments of counsel on the evidence, or the whole of the workings of his mind in arriving at his conclusion. Of course, to the extent to which he chooses to discuss the evidence, to that extent does he disclose how ‘the mind of the jury’ was working; and an appellate court is therefore in a stronger position to review his conclusions than it is in regard to a jury verdict. But an appellate court would not, except in the most exceptional circumstances, interfere with a finding which depended on the credibility of a witness; and, when the district judge draws inferences of fact, which inferences depend not only on an examination of documents and facts which are not in dispute but also depend partly on the credibility of witnesses and facts which were very much in dispute, then I think an appellate court should act with the greatest caution before interfering with the district judge’s findings if, having regard to the whole of the evidence, such findings appear reasonable. Having regard to the evidence adduced at the trial, and having regard to the fact that the magistrate was fully aware of what the issue was which he had to decide, the magistrate’s findings were unassailable and the required evidence to support them was there. Result - Appeal dismissed.

374

CCAB 2000

MA 763/99

Verdict

LAI Cheong Nang

Beeson J

Adequacy of Statement of Findings/Short written reasons making reference to oral reasons/No basis for assessment 裁斷陳述書內容是否充分的問題 - 簡短的書面理由提到參考口頭理 由 - 沒有理據可供評估

(30.3.2000) The Appellants were convicted after trial of the offence of common assault.

*Winston Chan

On appeal, the major complaint was that the magistrate’s Statement of Findings was inadequate.

#C Grounds

Held : (1) It was clear that the magistrate did not take any responsibility at all for preparing a proper Statement of Findings. It was well-known from the judgment in R v Chong Kam-san MA 859/85, that: The magistrate should, in a manner appropriate to the circumstances of the case, illustrate the salient points in the case and demonstrate that the evidence has been evaluated. Significant inconsistencies and conflicts should be dealt with in such a manner as to indicate how those matters were resolved. There is no simple formula for what ought to be in a judgment, as it must be related to the circumstances of the particular case. The magistrate should state his reasons, to such an extent as will inform the parties as to how and why the particular verdict was arrived at, and furthermore, will enable an appellate court to perform its duty. (2) The formal Statement of Findings was merely four simple sentences in which the magistrate referred those interested to his findings which were given in an open court at the end of the trial. However, when one turned to that material, it was a rambling mishmash of speculation and indirection, which certainly did not help an appellate court to assess what the case was about, or why these convictions were safe. Result - Appeal allowed.

MA 431/99 Stuart-Moore VP Wong JA & Beeson J (30.6.2000) *J Reading SC & Tam Sze-lok #C Y Wong SC & H Au Yeung [Reserved pursuant to s 118(1)(d) Cap 227]

TAN Say-seng

Magistrate delivering two sets of reasons for conviction/Comments on desirability of producing a single set 裁判官宣述兩套定罪理由 - 就宜宣述單一套定罪理由事作出論述 On an appeal against conviction, a ground of appeal concerned an apparent difference between the oral Reasons for Verdict given by the magistrate at the end of the trial and his more detailed Statement of Findings written some time later. In rejecting the submission and dismissing the appeal, the court observed : The fact that this ground of appeal could be advanced at all does provide magistrates with a salutary warning of the difficulties that can be caused by delivering an extempore judgment at the end of the trial and a later Statement of Findings which appears to, or does actually, differ from the original. If a magistrate perceives that the findings in a case need to be more detailed because of complex evidence or case law, the wiser course is to reserve judgment for an appropriate time to enable those findings to be put into orderly form and written down if necessary. [See also Sentencing in Hong Kong, 3rd ed., at p 364: Ed]. 375

CCAB 2000

MA 76/2000

Verdict

WU, David

(12.7.2000)

Magistrate providing separate sets of reasons for conviction/ Contradictory reasons cause for concern/Test of w het her difference in reasoning concerned form or substance 裁判官提交了多套定罪理由 - 互相矛盾的定罪理由引起關注 - 判斷 推論的差異是形式上還是實質上的差異

*John Reading SC & Maggie Yang

In the course of an appeal against conviction, it was submitted that the magistrate had erred in giving different reasons and findings in support of the conviction during his oral judgment at the trial, oral judgment at the review and written Statement of Findings.

Gall J

#C Y Wong SC & Peter Chow

CA 87/2000 Wong & Keith JJA Stock J (31.8.2000) *Peter Chapman #Wong Mankit

In rejecting this ground of appeal, the court observed: It is, of course, always a matter of concern if contradictory reasons are given in various rulings and reasons given at different times in the trial…. The differences between the various statements are more the manner in which the magistrate expressed himself rather than of substance. He employed different language to convey his meaning at different points during the trial, but I am satisfied that in overall terms his assessment of the evidence, his determination of the elements to be proved in the charge and his application of the law to the facts he found to be true has not been compromised in any measure by that difference of language.

CHAN Man-chung

Reasons for verdict/Extent of duty on court to resolve conflicts in evidence/Comments on resolution of differences by judge or magistrate 裁決理由 - 法庭在解決證據矛盾方面所擔負的責任 - 就法官或裁判 官解決分歧的做法作出評論 In the course of dismissing an application for leave to appeal against conviction for three offences of wounding with intent, which was made on the basis that the judge had not referred in his Reasons for Verdict to a number of differences in the evidence of the prosecution witnesses, and had not thereby complied with that said by the Court of Appeal in R v Lam Kam-ming Cr App 607/79, namely, that ‘significant inconsistencies and conflicts should be dealt with in such a manner as to indicate how these matters were resolved’, the court observed: It is appropriate, we think, for us to reiterate that it is not necessary for judges or magistrates to resolve in their Reasons for Verdict every single conflict in the evidence. They only need to decide those issues of fact which need to be decided in order to answer the ultimate question, which is whether the prosecution has proved each of the ingredients of the offence beyond reasonable doubt. That is not to say that peripheral factual issues never need to be resolved. What issues of fact need to be resolved in a particular case will depend on the circumstances of that case. But what we can say without reservation is that in the present case the judge was entitled to take the view that the differences in the evidence did not have to be resolved. It was open to him to conclude - as we must assume that he did - that, however those differences were resolved, the evidence of identification remained sufficiently reliable for the judge to have safely convicted the Applicant on the basis of it.

376

CCAB 2000 CA 91/1999 Stuart-Moore VP Leong JA & Keith JA

Verdict LAM Choi-man

False accounting/Whether invoices used as supporting documents for the issue of letters of credit were documents for accounting purpose/Inconsistent verdict based on wrongful acquittal/Extent of duty of District Judge to provide reasons 偽造帳目 - 為獲簽發信用證而用作證明文件的發票是否為會計用途 而製備的文件 - 錯誤裁定無罪因而造成不一致的裁決 - 區域法院法 官在列述理由方面所擔負的責任

(1.8.2000) *John Reading SC & Alex Lee #Daniel Marash SC

The Applicant was convicted of four charges of false accounting. All four charges involved LIANG Jin-wen, one of the co-conspirators who had absconded. LIANG was, at the material time, the general manager of CAC Brake Co., Ltd. (CAC). He had applied for the issue from the Guongdong Development Bank (GDB) of four letters of credit, without authority from CAC, with supporting documents which included the four invoices particularised in the four charges. These four invoices purported to show that there had been sales of goods to CAC but they were false in that there were in fact no such sales and no goods had been delivered. These letters of credits were discounted and the proceeds were distributed, some of which went to the Applicant’s account. He was acquitted of two charges of conspiracy to defraud. One of the grounds of the appeal was that the judge erred in convicting the Applicant in that there was insufficient evidence that the Applicant falsified any commercial invoice made or required for an accounting purpose of the issuing bank, the GDB or CAC. He submitted that the invoices were generated solely to deceive GDB and other banks and would not form part of the records of the CAC since it was not the prosecution case that the documents would form part of those records. There was also no evidence that the Applicant appreciated that GDB required an invoice for negotiating a letter of credit nor an invoice was always required for that purpose. The second ground was that the acquittals of the conspiracy charges were inconsistent with the convictions on the false accounting charges. Held : (1) There was ample evidence in each of the four charges that the Applicant had participated or concurred in the making of the four invoices which were false and he knew they were false because there were no sales and no goods had been delivered under the purported contracts of sales. The Applicant knew and intended that these invoices were to be used as supporting documents for the issue of the relevant letters of credit by GDB. These invoices were required to enable the bank to know the underlying transactions for the letters of credit. Without these supporting invoices, no bank would have issued them; (2) The invoices were required for accounting purpose even if they were not to be included as part of the accounts. The phrase ‘document made or required for any accounting purpose’ in s 19 of the Theft Ordinance, Chapter 210, should not be limited to accounting records or financial statements. It should include the documents that would be required for the preparation of accounting records or financial statements and an invoice would fall within that category: AttorneyGeneral’s Reference (No. 1 of 1980) [1981] 1 W L R 36 followed; (3) The purpose or motive for the creation of the false invoices was not relevant, and as long as the invoices were falsified by the Applicant as a party to the falsification for an accounting purpose, it was not necessary to prove that the Applicant knew that they were for the accounting purpose of the GDB: R v Graham [1997] 1 Cr App R 302 considered; (4) The conspiracy offences and the false accounting offences were distinct in their elements that were required to be proved. It might well have been that the evidence of the prosecution witnesses supported a verdict of guilty of the conspiracy charges by the Applicant, and that acquitting him purely on the 377

CCAB 2000

Verdict ground of lack of evidence from GDB was wrong. But a wrong acquittal was not a ground to disturb a correctly founded conviction on a different offence. There was ample evidence to support the conclusion of the judge; (5) The trial judge was not required when setting out his reasons, as required by s 80 of the District Court Ordinance, Chapter 336, to state precisely what evidence he accepted or rejected: R v Sheik Abdul Rahman Bux [1989] 1 HKLR 1 followed. Result - Application dismissed.

Witness CA 249/99 Stuart-Moore & Mayo VPP Woo JA (11.4.2000) *Peter Chapman, Jackson Poon & H Tse #P Cogrove (Ds1, 2, 4, 6, 8, 15 & 16) M Delaney (D3 & D9) W Allan (D5, D19 & D20) G J X McCoy SC & V Rajwani (D17) D Boyton (D18)

WONG Fung-tak and 15 others

Witness permitted to refresh memory/Reading from diary when testifying in witness box/Acceptable practice in cases of complexity to avoid mistake and confusion/Acting as member of triad society/Procuring others to act as members of a triad society sufficient basis for conviction under s 20(2), Cap 151 證人獲准重溫一些資料以助記憶 - 在證人台作證時照記事簿內容讀 出 - 在複雜案件中為避免錯誤及混淆可以接受這種做法 - 以三合會 社團成員的身分行事 - 促致他人以三合會社團成員的身分行事,是 被裁定第151章《社團條例》第20(2)條的罪行罪名成立的充分理據 The Applicants were convicted in the District Court after a 12-day trial of offences related to their connections with the Wo Hop To triad society. The charge sheet contained 29 charges against 23 defendants. For a period of about 6 months, the Wo Hop To triad society was infiltrated by a police agent, PW1. On appeal, it was submitted, inter alia, that the judge erred when he permitted PW1 to read his evidence from a diary prepared by him during the course of the investigation. On behalf of D17, who allegedly summoned triad troops to execute a revenge attack, it was submitted that the judge had wrongly interpreted the offence of acting as a member of a triad society to include his conduct in procuring other people to act as triad members. Held : (1) The judge had a full grasp of the considerations to be borne in mind when approaching the commonly encountered situation, in cases of factual complexity, where witnesses requested to refer to and even to read from notes made at or shortly after the time of the incident to which they referred. Time and again, the courts had emphasised the importance of not turning the exercise of obtaining testimony given by witnesses from the witness box into a test of memory. The jury, or a judge sitting alone, were primarily concerned with the truthfulness and accuracy of witnesses and it bordered on the absurd to suggest that when a record was available from which the witness could refresh his memory as to complicated events he could not also be permitted to read directly and extensively from it: R v Da Silva (1990) 90 Cr App R 233, distinguished; (2) Section 20(2), Cap 151, did not exclude the application of the common law doctrine of secondary participation. D17 was fully aware of the nature of the case against him as a procurer of the troops and he had ample opportunity to defend himself against that precise allegation; (3)

It was quite unnecessary to prove actual membership of a triad society so 378

CCAB 2000

Witness

far an offence of acting as a member of a triad society were concerned. The common law doctrine related to secondary participation had been incorporated into s 89 of the Criminal Procedure Ordinance, which provided that: Any person who aids, abets, counsels or procures the commission by another person of any offence should be guilty of the like offence. It was open to the judge to convict D17 on the basis that he procured others to commit the offence of acting as triad members. He was, however, by acting as he did, himself acting as a member of a triad society by procuring others to so act. On either basis, however, the conviction was amply justified. Result – Applications dismissed.

MA 352/2000

NUSRAT Bibi

Bigamy/Husband competent but not compellable to testify 重婚 - 丈夫有資格作證但不可被強迫作證

Tong DJ (22.8.2000)

The Appellant was charged with offences of bigamy and making a false declaration for the purpose of procuring a miscarriage. After trial, she was convicted of bigamy only, and fined $10,000.

*C Ko #M Panesar

On appeal, the substantial ground alleged a material irregularity in that the magistrate failed to advise PW1, the husband of the Appellant, that he was not compellable to testify against her. It was common ground between the parties that there was no problem as to competence. Held : (1) In Bruce and McCoy’s Criminal Evidence in Hong Kong (1995 ed), at paras [355]-[400], it was stated: Where a spouse is competent to give evidence for the prosecution, and is called to do so by the prosecution, the court must satisfy itself before the spouse gives evidence that he or she is aware that he is not compellable to do so: Hoskyn v Metropolitan Police Commissioner (above). The position should be explained to the witness before (the spouse) gives evidence: R v Pitt (1982) 75 Cr App R 254. If necessary the matter may be resolved by voir dire proceedings before the spouse gives evidence. It was further stated at para [301] that: ... If the prosecution calls a spouse to testify in a case in which he or she is neither competent nor compellable, this may be such an irregularity as to entitle an accused to have his conviction quashed on appeal: R v Yung Kit-mei Cr App 1025/81. The magistrate was not aware of this and as a result, she had failed to give the advice accordingly; (2) The court could not speculate what would have been PW1’s reaction if he had been advised that he was not obliged to give evidence against his wife. In any event, justice had to be seen to be done. The magistrate should herself have given the advice to PW1, if not before PW1’s evidence, at least at some stage during his evidence. Result - Appeal allowed.

379

CCAB 2000 HCCC 411/99 Gill DJ (20.7.2000)

Witness LEE Chi-hung

Witness testifying at earlier trial/Witness not available to testify at retrial/Discretion at retrial to have read to jury earlier testimony/Jury to be directed on weight 證人曾在早前的審訊中作供 - 證人在重審時未能出庭作供 - 在重審 時法官可酌情准許向陪審團讀出早前所作的證供 - 法官須就證供的 比重引導陪審團

*Catherine Ko #Phillip Ross

This was a retrial following the discharge of a jury after the judge ruled that prejudicial material was inadvertently made known to the jury. At the first trial, the prosecution wanted to produce evidence of admissions that were contested by the defence, and in the course of a voire dire proceeding the defence called a Madam Chan to testify as to police conduct at or prior to the defendant’s arrest. After that proceeding, the judge ruled the admissions to be inadmissible. At that trial, the defence had intended to call Madam Chan to give precisely the same account before the jury on the general issue but, in the event, the trial was terminated before she could testify. At the retrial, the prosecution indicated that it no longer wished to produce the alleged admissions. The trial proceeded to the stage where the defendant gave evidence and, although it wished to called Madam Chan, it could not do so as she could not be located and the witness summons could not be served. Although the defence applied to have the transcript of the evidence of Madam Chan from the first trial read to the jury as her evidence, the prosecution opposed this, firstly, on the ground that there would be no opportunity to crossexamine Madam Chan before the jury, and, secondly, on the ground that, without evidence, it could not be accepted that all had been done that could have been done to trace the witness and get her to come to court. Held : (1) Jurisdiction to deal with the application derived from section 22 of the Evidence Ordinance, which made provision, in certain circumstances, for a statement contained in a document to be admitted in criminal proceedings as prima facie evidence of any fact stated therein if the person could not be found after all reasonable steps had been taken; (2) Although the equivalent English legislation was differently worded, it had been held in R v Lockley and Corah (1995) 2 Cr App R 554, that a transcript of evidence taken at the first trial from a witness who had since disappeared was admissible and that the judge at trial should have considered the exercise of a discretion available to him; (3) Although s 22(2) of the Evidence Ordinance, which provided that ‘a statement made in connection with any criminal proceedings or with any investigation relating or leading to any criminal proceedings shall not be admissible under this section’, might be seen at first blush as operating to exclude a transcript, and support for that view was to be found in Lockley and Corah, the editors of Archbold 2000 were critical of that approach, and, at paragraph 10-57 they stated: The conclusion that section 26 applies is questionable; it involves a finding that a transcript is a statement ‘prepared for the purposes of pending or contemplated criminal proceedings or a criminal investigation’. It is submitted: (a) that this language is entirely inappropriate to a transcript; and (b) a transcript is plainly outside the mischief at which section 26 was aimed. That mischief was the possibility that a party in the course of preparation for proceedings might fashion the evidence to suit his own cause. Parliament could never have intended a transcript to come within this section: see the explanation of the common law rule in Hall. The most important 380

CCAB 2000

Witness considerations are, of course, that the transcript is evidence on oath with the opportunity for cross-examination: the common law rule depended on the parties being the same, the cause being the same, or substantially the same, and there being opportunity to object to the competency of the witness, to cross-examine him and to contradict him by other evidence. The judgment in Lockley and Corah leads to the bizarre result that the admissibility of a transcript in a retrial ordered under the 1968 Act is not subject to section 26, whereas at a retrial ordered in other circumstances, it is. Neither Hall nor Thompson are referred to in the judgment of the court, although apparently referred to in argument; it is submitted that the decision should be treated as having been made per incuriam. The court agreed with the editors of Archbold 2000; (4) The defence application would be acceded to, although the jury would need to be directed on the issue of weight in the manner contemplated by s 22B(3) of the Evidence Ordinance. Result -

Application allowed, subject to defence demonstrating through evidence that all reasonable steps had been taken to find Madam Chan.

香港特別行政區訴鄧冬玉 HKSAR v T ANG Do n-yuk 高等法院原訟法庭–高院裁判法院上訴1 9 9 9 年第9 3 1 號 *冼佩霞 Sin Pui-ha # 梅國強 Mui Kwokkeung

高等法院原訟法庭法官彭鍵基 聆訊日期:二零零零年七月十二日 宣判日期:二零零零年八月九日 COURT OF FIRST INSTANCE OF THE HIGH COURT MAGI ST RACY AP P E AL NO. 9 3 1 OF 1 9 9 9 P ang J Date o f Hear ing : 1 2 J uly 2 0 0 0 Date o f J ud gment : 9 August 2 0 0 0 丈夫指證妻子 - 法庭是否需要提醒丈夫有權不作證- 可否強迫丈夫 作證- 向丈夫發出的證人傳票的效力 上訴人被控襲擊致造成身體傷害的罪名,經審訊後被裁定 罪名成立。受害人是上訴人的丈夫。上訴人不服原判提出上訴。 上訴時,上訴人代表律師提出多項理由,包括: (i )

在普通法的法制下,被告人的配偶不能被‘強迫’ 在刑事審訊中指證被告人。在本案中,法庭向丈夫 發出證人傳票傳召他出庭作證,這等同於‘強迫’ 丈夫作證,所以屬於重大不當之處;

(i i )

原審裁判官沒有提醒丈夫他可以選擇不作供,亦沒 有指出法庭不能‘ 強迫 ’他作供。

381

CCAB 2000

Witness 裁決︰ (1) 證 人 傳 票 是 根 據 《 裁 判 官 條 例 》 表 格 14 簽 發 的 。 傳 票 要 求 證人於指定日期到裁判官席前作供。證人只要在指定日期到達法 院,便已履行證人傳票所定的責任。證人傳票並沒有規定證人必須 在裁判官席前作供。 (2) 在 R v Pitt (1982) 75 Cr App R 254一案中,法庭裁定如妻 子被傳召作控方證人,以便檢控她的丈夫,由於她是一個有資格但 不是可被強迫作證的證人,因此較理想的做法是法官於陪審團退席 後,並在她宣誓之前,向她解釋她有權拒絕作證。 (3) 根據案例顯示,如案件涉及證人作供指證其配偶,主審法 官必須提醒證人其權利這個做法,不能說是一項法律程序上的規 定。 上訴駁回。

[English digest of MA 931/99 above] Pang J (9.8.2000)

TANG Don-yuk

Husband testifying against wife /Whether warning to husband necessary/Whether husband compellable/Effect of witness summons issued to husband The Appellant was convicted after trial of assault occasioning actual bodily harm. The victim was the husband of the Appellant. The Appellant appealed against conviction. On appeal, it was submitted, inter alia, that:

*Sin Pui-ha

(i) under the common law a spouse of the accused could not be ‘forced’ to give evidence in a criminal proceeding against the other. The husband was served with a witness summons to attend court to give evidence. That meant that the husband was ‘forced’ to give evidence. It was a material irregularity;

#Mui Kwok-keung

(ii) the magistrate failed to give a warning to the husband that he could elect not to give evidence, and the court could not ‘force’ him to testify. Held : (1) The witness summons was issued in accordance with Form 14 of the Magistrates Ordinance. It required a witness to attend before a magistrate on a certain date to give evidence. Once a witness had attended court on a specified date, his duties under the witness summons were discharged. The witness summons did not require that a witness must give evidence before a magistrate; (2) In R v Pitt (1982) 75 Cr App R 254, it was held to be desirable that where a wife was called as a witness for the prosecution of her husband - she being a competent but not a compellable witness - the judge should explain to her, in the absence of the jury and before she took the oath, that she had a right to refuse to give evidence; (3) On the authorities it could not be said that there was a procedural requirement that such a warning must be given in cases involving a witness who gave evidence against his/her spouse. Result - Appeal dismissed.

END 382

Smile Life

When life gives you a hundred reasons to cry, show life that you have a thousand reasons to smile

Get in touch

© Copyright 2015 - 2024 PDFFOX.COM - All rights reserved.